처음 처음 | 이전 이전 | 1 | 2 | 3 |다음 다음 | 마지막 마지막

12

최근의 동향

 

이번 에서는 비교적 최근에 언어철학자들에 의해 주로 논의되어온 다음 다섯 가지 영역을 간략하게 살펴보고자 한다(그 과정에서 앞선 들에서 탐구된 개념 및 이론들이 종종 활용될 것이다): (1) 주장(主張)(단언(斷言), 긍정적 서술)assertion, (2) 맥락-상대성context-relativity, (3) 허구적(虛構的) 실체fictional entity, (4) 추론주의(推論主義)inferentialism, (5) 슬러slur. 다만 이 주제들에 관한 모든 논의들을 철저하게 살펴보기보다는 독자의 흥미를 고취하는 정도로만 개관할 것이다.

 

 

주장

 

Frege판단(判斷)judgement한 명제를 참이라고 받아들임acceptance of a proposition as true으로, 그리고 주장(主張)(단언(斷言), 긍정적 서술)assertion판단을 언어적으로 표출(表出)(현시(顯示))verbal manifestation of judgement으로 특징지었다. 그는 주장이라는 표현 대신 말하기to say, 주장하기proclaim, 진술(陳述)하기state, 긍정(肯定)하기affirm, 언명(言明)하기declare, 선언(宣言)하기proclaim, 공표(公表)하기announce, 제언(提言)하기put forward, 공언(公言)하기avow 등등 미묘하게 다른 개념들을 표현하는 여타 용어들을 활용할 수도 있었을 것이다. 하지만 대부분의 철학자들은 Frege의 선례를 따라 주장 개념을 핵심적인 것으로 간주하였다.

7에서 우리는 Austin을 따라 주장이라는 화행(話行)speech-act이 규약(規約)convention에 지배된다는 생각을 받아들인 바 있다. 지면상의 이유로 주장 화행이 주장하는 사람asserter의 심리psychology 내지 의도(意圖)intention의 측면에서 -규약적으로 설명될 수 있다는 관점은 여기서 차치해두기로 한다. 이러한 견해는 Grice의 영향력 있는 두 논문 의미Meaning(1957) 화자 의미와 의도Utterer’s Meaning and Intention(1969)에 개진된 것으로서, 두 글에서 Grice는 다른 무엇보다도 주장을 화자의 의사소통적 의도communicative intention의 측면에서 정의하고자 시도하였다(이에 대한 맛보기로서 이번 말미의 탐구문제를 참조할 것). 하지만 우리는 이러한 견해를 도외시한 채 주장 화행을 최소한 부분적으로 규약적인 종류의 행위로, 즉 사회적으로 구성되는 관습행위socially constituted practice로 가정할 것이다. 이러한 논점을 Michael Dummett은 다음과 같이 분명하게 내비친 바 있다:

 

언어적 행위linguistic act는 내적 상태interior state의 외적 표현으로 분류될 게 아니라 규약적 행위conventional action로 분류되어야 한다. 예를 들어 주장이라는 행위는 한 사람이 해당 문장을 참이라고 판단하는 내적 행위(혹은 믿는다는 내적 상태)를 표현하려는 의도에서 그 문장을 발화하는 행위로 설명될 게 아니라, 주장적 효력assertoric force을 지닌 것으로 받아들여지는 문장의 사용을 지배하는 규약의 측면에서 설명되어야 한다.

 

(Dummett 1973[Frege: 언어철학Frege: Philosophy of Language], 311.)

 

주장이 규약적이라는 점을 받아들이고 나면 우리가 물어야 할 질문은 이것이다: 이러한 특정 화행을 특징짓는 그 규약이란 정확히 무엇인가?

본격적인 논의에 앞서 (Elizabeth AnscombeJohn Rawls를 따랐던) Searle을 따라 규제적(規制的) 규칙regulative rule구성적(構成的) 규칙constitutive rule이라는 구분을 도입하고자 한다. 먼저 규제적 규칙이란, 가령 영국 도로에서 통용되는 좌측통행운행규칙과 같이, 이미 이루어진활동pre-existing activity을 사후적(事後的)으로 규제하는 규칙이다. 영국 도로에서 좌측통행 규칙을 어긴다고 해서 그 행위가 자동차 운행이 아니게 되는 것은 아니다(다만 이는 운전자의 목숨을 그의 것이 아니게 만들 것이다). 이와 대조적으로 구성적 규칙이란 그 규칙의 지배를 받는 해당 활동 내지 행위가 성립하기 위해 그 규칙 자체가 반드시 필요한 규칙이다. 가령 체스나 풋볼을 정의하는 규칙들은, 단순히 체스말을 배치하거나 공을 차서 네트에 넣는 방식을 규제하기만 하는 게 아니라, 체스에서 말들을 운용하거나 풋볼에서 득점하는 것 자체를 가능케 하는 규칙이다. 좀 더 일반화해서 말하자면 체스나 풋볼에서 통용되는 규칙이 없다면 우리가 아는 바로서의 체스나 풋볼 게임 역시 존재하지 않는 셈이다. [가령 어떤 게임에서 체스말과 체스판이 사용되더라도 우리가 아는 체스 규칙에 따라 게임이 진행되지 않는다면 이는 어쨌든 체스게임이 아니다.] 에서 살펴본바 삶의 형식들forms of life이 구성적이라는 말의 의미는 바로 이것이다. 물건을 사고파는 것, 재산을 소유하는 것, 혼인하는 것 등과 같은 다양한 사회제도(制度)social institution 내지 삶의 형식들이 가능한 이유는 그것들이 존립할 수 있게 해주는 특정한 구성적 규칙들이 존재하기 때문인 것이다.

규제적/구성적 규칙 구분은 많은 비판을 받아왔지만, 여기서 우리는 그 구분을 일단 문제없는 것으로 받아들이고자 한다. 이러한 구분을 도입하는 요지는 주장이라는 언어행위가 구성적 규칙의 지배를 받는다는 것이다. 이러한 착상을 염두에 둔 채, 이제 Moore의 역설(逆說)Moore’s paradox(이는 Russell과 대략 비슷한 시기에 활동했던 영향력 있는 철학자 George Edward Moore의 이름을 따서 붙여진 것이다)이라 알려진 역설의 한 사례를 생각해보자. Gia가 다음과 같이 말한다:

 

냉장고에 치즈가 있지만, 나는 그것을 믿지 않는다.

 

Gia의 발언은 어딘가 잘못된 것처럼 여겨진다. , Gia에 의해 발화된 문장 자체는 (‘p&p’ 형식과 같은) 논리적 모순이 아니라는 점을 명심해야 한다. 냉장고에 치즈가 있는데도 Gia가 냉장고에 치즈가 없다고 믿는 것은 명백히 논리적으로 가능한 사태이기 때문이다.1) 여기서 문제는 그녀가 어떤 의미에서 스스로를 표상하는(드러내는)represent 방식, 냉장고에 치즈가 있다의 내용을 받아들임으로써 그것을 믿는 동시에, 그 내용을 믿지 않는다고 명시적으로 말하고 있다는 데에 있다. 요컨대 위 발화의 부적절성infelicity은 문장의 진리치와 관련되는 의미론적인 층위에 있는 게 아니라, 문장의 발화와 관련되는 화용론적인 층위에 있다. 자신이 믿고 있는 것만을 말해야 한다는 것은 주장의 본성에 속하는 사안이다. 따라서 다음 규칙은 주장에 대한 구성적 규칙으로서, 이를 위반하는 것은 주장을 지배하는 규약을 위반하는 것처럼 여겨진다:

 

(믿음-규칙Belief-rule) p라고 믿는 경우에만 p라고 주장해야 한다.

 

1) ‘x는 치즈이다‘Cx’, ‘x는 냉장고에 있다‘Rx’, ‘sp라고 믿는다‘Bs[p]’로 각각 기호화한 뒤, 본문의 예시문을 술어논리의 언어로 번역하면 다음과 같다(지표사 와 연관된 구문론적의미론적 문제는 차치한 채 단순히 ‘i’로 기호화하기로 한다):

 

(x)(Cx&Rx)&Bi[(x)(Cx&Rx)].

 

첫 번째 연언지 ‘(x)(Cx&Rx)’의 부정문은 두 번째 연언지에서 믿음 연산자의 영향권 내에서 나타나고 있는바, 이 문장은 ‘p&p’ 형식의 모순이 아니다


 그런데 주의할 사항이 있다. Gia가 냉장고에 치즈가 있다고 믿지 않으면서 단지 냉장고에 치즈가 있다고만 주장한다면(어쩌면 그녀는 실제로도 냉장고에 치즈가 없다는 것을 알고 있을 수 있으며, 그 경우 Gia는 거짓말을 한 셈이다) 위 규칙을 위반하는 것이지만, 그 경우에도 어쨌든 그녀는 냉장고에 치즈가 있다는 주장을 한 셈이다. 즉 믿음-규칙은 주장행위가 이뤄질 때마다 자동적으로 적용되지만, 가령 지켜지지 않을 약속을 하는 경우처럼, 그러한 규칙 내지 규범(規範)norm이 준수되지 않는 경우가 가능한 것이다. 약속의 경우 약속을 한 당사자는 그것을 이행할 의무(義務)obligation를 반드시 떠안게 된다. 즉 약속한 바를 이행할 의무가 부여된다는 것은 약속 행위에 대해 구성적이다. 이와 마찬가지로 주장하고 있는 내용을 믿어야 한다는 것은 주장 행위에 대해 구성적이다. 믿음-규칙을 위반하는 주장행위는 신뢰할 수 없는 불성실한 행위이다.

지금까지의 관찰은 우리를 Grice적인 영역으로 이끄는 듯하다. 믿음-규칙을 위반하는 것은 Grice가 제시한 협조원리들 중 질의 준칙을 위반하는 것 아니겠는가? 물론 그렇긴 하다. 그런데 Grice의 준칙들은 규제적인 규칙인 동시에 상황에 따라 조금씩 다르게 적용될 수 있는 대강의 규칙rules-of-thumb일 뿐이다. 하지만 지금 우리는 어떤 구성적 규칙이 가능할 수 있는지, 말하자면 주장을 정의해 줄 구성적 규칙이 무엇인지를 논의하고 있다.

그럼에도 Grice의 질의 준칙을 가만히 살펴보면 다른 가능성이 발견될 수 있을 듯하다. 우선 질의 준칙 자체는 다음과 같이 말한다: ‘거짓이라 믿는 바를 말하지 말라’, ‘적절한 증거가 결여된 바를 말하지 말라’. 이에 두 항목을 각기 적절히 활용하여 다음과 같은 두 규칙을 정립할 수 있겠다:

 

(-규칙Truth-rule) p인 경우 그리고 오직 그 경우에 p라고 주장해야 한다.

(정당화-규칙Justification-rule) p라는 정당화된 믿음을 갖는 경우 그리고 오직 그 경우에 p라고 주장해야 한다.

 

유의할 사항은, 믿음-규칙과 마찬가지로 위 두 규칙 역시 무언가를 긍정적으로 주장하는 데 대한 충분조건을 명시하고 있지는 않다는 점이다. 두 규칙은 적절한 주장이 되기 위한 필요조건만을 명시하고 있다. 다르게 말해 두 규칙은 우리가 무언가를 주장할 때 준수할 것이라 기대되는 제약조건만을 나타낸다.

이제 믿음-규칙과 위의 두 규칙을 모두 결합하면 다음을 얻는다:

 

(JTB-규칙JTB-rule) p라는 정당화된 참인 믿음justified true belief that p을 갖는 경우 그리고 오직 그 경우에 p라고 주장해야 한다.

 

정당화된-참인-믿음(약칭 JTB)은 전통적으로 지식(知識)knowledge에 대한 정의항으로 간주되어왔다. 그런데 Edmund Gettier는 유명한 논문에서 JTB 조건을 충족하면서도 지식이 아닌 사례를 제시하였다. 간단한 예를 들어보자. 나는 우리 집 울타리 건너편에 개가 한 마리 있다고 믿는다. 울타리가 워낙 빽빽한 정원수로 뒤덮여 있기에 내가 개를 본 적은 없지만, 늘상 그 방향에서 개 짖는 소리가 들려오기 때문에 나의 믿음은 충분히 정당화된다. 하지만 사실 평소 내가 들었던 소리는 실제 개가 내는 소리가 아니라 녹음된 개소리였다. 그런데, 비록 나는 모르고 있지만, 울타리 건너편에서는 개 한 마리가 늘상 조용히 어슬렁거리고 있다. 이런 상황을 감안하건대, 나는 울타리 건너편에 개가 있다는 정당화된 참인 믿음을 갖고 있긴 하지만, 울타리 건너편에 개가 있다는 것을 내가 안다고 할 수는 없을 듯하다. 내가 이러한 기이한 상황에 놓인 것은 순전한 우연에 불과한 것처럼 보인다. 왜냐하면 나의 믿음을 정당화해 주는 요인(녹음된 개소리)이 내 믿음을 참이게 만들어 주는 요인(조용히 어슬렁거리는 개)와 아무런 관련이 없기 때문이다. [이렇듯 나의 믿음은 지식에 대한 JTB 조건을 만족함에도 이는 우연의 일치에 지나기 않기에 지식이 아닌 것처럼 보인다. 따라서 JTB 조건은 지식에 대한 필요충분조건이 아니다.]

지식에 대한 소위 Gettier 반례가 처음 제기되었을 때, 많은 사람들은 지식에 대한 정의인 JTB 조건을 약간만 수정하면 Gettier 사례를 처리할 수 있을 것이라 생각했다. 하지만 이후 전개된 현대 인식론의 역사를 보면, 최초 Gettier 사례의 미묘한 변형태들이 수정된 JTB 정의의 반례로서 계속 제기되었으며, 최종적으로 JTB 정의를 만족스럽게 수정하려는 시도는 지난한 일로 판명되었다. 이에 Timothy Williamson은 지식이 이런 식으로 분석될 수 없으며 지식을 원초적(原初的)primitive(정의불가능한indefinable) 것으로 받아들여야 한다고 주장하기에 이른다. 따라서 그는 주장에 대한 제약사항으로 다음을 제안한다:

 

(지식-규칙Knowledge-rule) p라는 것을 아는 경우 그리고 오직 그 경우에 p라고 주장해야 한다.

 

이는 구성적 규칙이다. 그리고 앞서 살펴본 규칙들과 유사하게, 지식-조건이 만족되지 않는 주장들은 매우 많다. 그럼에도 불구하고, 무언가를 약속할 때 약속 당사자가 약속을 이행해야 한다는 규범의 지배를 받는다는 것과 정확히 동일한 의미에서, 무언가를 주장할 때 주장하는 사람은 지식-규칙의 지배를 받는다.

이러한 제안에 따르면 다음과 같은 화행은 부적절한 것으로 판명된다(이 예시는 Moore의 역설을 확장시킨 것이다):

 

냉장고에 치즈가 있지만, 나는 그것을 모른다.

 

Williamson의 제안은 많은 지지를 얻음과 동시에 많은 비판을 받기도 하였다. 우선 긍정적인 측면을 들자면, 지식-규칙은 주장행위가 지식을 전달하는transmitting 하나의 방식이라는 생각 및 무언가를 주장하는 사람은 주장한 바를 방어하는defending 입장에 개입하게 된다는 생각과 잘 부합하는 것처럼 여겨진다. 하지만 지식-규칙보다 약한 규칙들, 가령 참-규칙 내지 심지어 믿음-규칙 역시 이러한 작업을 수행하기 위한 것으로서 주장되었다. 그리고 주장 개념을 언어철학 내지 화용론에서 핵심적인 사안으로 간주하는 것부터가 애초에 잘못이었다는 주장이 제기되기도 하였다. 우리가 현실에서 실제로 수행하는 서술적 화행declarative speech-act들이 언어철학에서 생각되는 것만큼 균일하고 통일적이지는 않기 때문이라는 것이다. 말하기, 상기(想起)하기, 경고하기, 유발하기, 조언하기, 상담하기, 표현하기, 지도(指導)하기, 기술하기, 보고하기 등의 다양한 화행들이 지닌 차이점들은 매우 미묘하겠지만, 어쨌든 그 각각은 나름의 고유한 규범들 집합을 지니고 있을 것이다.

 

 

맥락-상대성

 

독자들은 이제 지표사(指標詞)indexical에 익숙할 것이다. 지표사에 속하는 표현들로는 ’, ‘’, ‘우리’, ‘he/him’, ‘그녀’, ‘지금’, ‘그 때’, ‘오늘’, ‘어제’, ‘여기’, ‘저기’, ‘저것’, ‘이것등이 있다. 이러한 단순한 형태의 지표사뿐만 아니라 너희 어머니가 기르시는 고양이처럼 여러 표현들이 혼합된 형태의 지표적 표현도 있다. 이러한 표현들은 맥락-상대적이다context-relative 내지는 맥락에-민감하다context-sensitive고 말해진다. 즉 지표적 표현들은 오로지 특정한 맥락(脈絡)context에서만 지시체를 가지며, 그것들이 사용되는 맥락(이러한 맥락으로는 시간, 장소, 화자 및 청자의 동일성, -지시화된demonstrated 대상의 동일성 등이 있다)에 따라 각기 다른 대상을 지시한다.

일찍이 지표사에 관한 에서 우리는 크다’, ‘부유하다’, ‘가깝다등 단어들을 언급한 바 있지만, 이에 관해 상세히 논하지는 않았다. 이러한 형용사들 역시 맥락에 민감한 것처럼 보인다.2) 예를 들어 누군가 쥐 한마리를 가리키면서 다음과 같이 말한다 해보자:

 

(1) 저거 엄청 크다!

 

이 경우 이 문장에 의해 할당되는 사이즈의 범위는, 그와 동일한 문장이 하마를 가리키면서 말해질 경우 그 하마에게 할당되는 사이즈의 범위와 분명 다를 것이다. 그 하마가 평균적인 하마에 비해 현저히 작다면 전자는 참이더라도 후자는 거짓일 것이다. 물론 아무리 작은 하마라 하더라도 쥐보다는 크다. 하지만 그럼에도 그 하마는 하마치고는작은 셈이다. [즉 전자에서 할당되는 사이즈가 크다고 말해질 수 있다면 후자에서 할당되는 사이즈 역시 당연히 크다고 말해질 수 있어야 하겠지만, 이는 크다가 사용되는 맥락이 무시될 수 있는 경우에만 올바르다.] 이렇듯 크기에 대한 표준(標準)standard 내지 적절한 비교집합comparison calss은 맥락에 따라 달라지는 듯하다.


2) (原註) 이외에도 맥락-상대성을 갖는다고 추정되는 표현들로는 다음을 들 수 있다:

(1) 한 용어가 자동사 형태로 나타나지만 암묵적으로 타동사 역할을 하는 경우. 가령 난 준비됐어라는 문장은, 어떤 행위나 사건을 나타내는 용어 𝜙에 대해 나는 𝜙할 준비가 되었다라는 문장의 줄임말인 셈이다. [그리고 𝜙가 나타내는 행위/사건이 정확히 무엇인지는 이 문장이 사용되는 맥락에 의해 결정된다.] 신나는 파티에 가기 위해 이제 막 외출하기 직전 상황에서 당신이 ‘Berta는 준비 안됐대라고 말한다. 통상 이러한 상황에서는 무슨 준비?’ 하는 물음에 무리 없이 답할 수 있겠지만, 당신이 말한 문장 그 자체만으로는 Berta가 무슨 준비가 되지 않았는지를 알 수 없다.

(2) Putnam[]’과 같은 자연종(自然種) 용어natural kind term가 암묵적인 지표적 요소를 지니고 있다고 논증한다. 자연종 용어가 지시하는 대상은 그 용어가 사용되는 가능세계에 따라 달라지기 때문이라는 것이다. 앞서 2차원주의 의미론two-dimensional semantics을 논할 때 살펴보았듯이, 현실세계와는 다른 가능세계non-actual possible world에서 이 발화될 경우, 표현되는 개념 내지 의미는 동일하겠지만 그 지시체는 분명 다르다. 예를 들어 현실의 발화맥락context of utterance과는 다른 발화맥락에서 로 칭해지는 물질은 H2O가 아니라 XYZ일 수 있다.

(3) 통상적인 고유명ordinary proper name에는 지표성이 결부되어 있거나 혹은 그 자체가 지표적이라고 주장될 수 있다. 한 학생이 ‘John Smith’를 발화한다면 이는 누구를 지시하겠는가? 답은 다음과 같다: ‘John Smith’라는 이름을 가진 실존하는 수많은 사람들 중 그 발화가 지시하는 이는, 그 이름에 대한 화자의 사용과 연결된 인과적 연쇄의 기원에 있는 한 사람이다. 따라서 화자와 연관된 적절한 인과연쇄를 골라내는 매개변수 x를 취하여, ‘John Smithx와 같은 식으로 표기할 수 있다


부유하다에 대한 표준 역시 이와 마찬가지이다. 우리는 샌프란시스코에 거주하는 중산층 이하의 사람을 보고 부유하다고 말하지는 않는다. 참고로 샌프란시스코 시민의 중위소득은 78,378달러이다. 그런데 중위소득이 590달러인 에티오피아에 사는 사람이 앞의 샌프란시스코 거주민과 동일한 소득을 번다면, 우리는 그 에티오피아 사람이 부유하다고 말할 것이다. ‘가깝다는 형용사 역시 마찬가지이다. 저녁을 먹기 위해 괜찮은 식당을 물색하면서 꽤 가깝네하고 말하는 경우와, 여행할 나라를 고르기 위해 지구본을 돌려보면서 꽤 가깝네하고 말하는 경우, 각각에 적용되는 근접성에 대한 표준은 분명 다르다.

이러한 비교급 형용사 및 비교급 전치사의 경우에 중요한 것은 표준 내지 비교집합에 대한 암묵적 상대성implicit relativity이다. 이에 대한 한 가지 설득력 있는 가설은 크다와 같은 단어들에는 각기 다른 표준 내지 비교집합을 지시하는 암묵적 지표사가 수반되어 있다고 보는 것이다. 그리고 그러한 암묵적 지표성을 명시적으로 드러내기 위해 크다x와 같은 식으로 표기할 수 있다(종종 지표사는 특정 규칙에 의해 논의영역이 제한된 자유변항free variables처럼 간주되고는 한다).

맥락-민감성이 부각되는 또 다른 종류의 사례는 양화사가 포함된 문장 및 모든 사람과 같은 양화사quantifier-phrase이다(이는 6첫 번째 말미에서 잠시 언급된 적이 있다). 다음 문장을 보자:

 

(2) 모든 사람들이 전부 다 무례했다.

 

Sally가 파티장을 빠져나오면서 이 말을 중얼거렸다고 해보다. 이 경우 (2)에 대한 Sally의 발화는, 세상에 존재하는 모든 사람이 무례한 경우 그리고 오직 그 경우에 참인 게 아니라, 그 파티에 참석했던 모든 사람(혹은 그 파티에서 Sally와 대화하면서 무례하게 굴었던 모든 사람)이 무례한 경우 그리고 오직 그 경우에 참이라고 해야 한다. 이러한 사례에서 발화와 연관된 맥락은 부분적으로 화자의 의도intention에 의해 결정되는 듯하다. 다른 맥락에서 발화된다면 (2)에 대해 의도된 일반화의 범위는 이 이상으로 얼마든지 달라질 수 있다.[?] 가령 모든 사람Sally가 뉴욕에 머무르는 동안 그녀와 대화했던 모든 사람들을 아우를 수도 있다.

비교급 형용사에는 비교집합을 지시하는 암묵적 지표사가 수반된다고 생각해야만 하는 것과 유사하게, 양화사에는 대상들의 집합을 지시하는바 사실상 암묵적 지표사인 논의영역 제한자domain restrictor가 수반된다고 생각해야만 한다. 한정 기술구가 사용되는 방식을 Russell의 기술구 이론에 따라 생각해본다면 그럴 수밖에 없다는 점이 확연히 드러난다. 우선 Russell에 따르면 F’는 사실상 양화사이다. ‘FG하다는 정확히 하나의 F존재하고 모든 FG인 경우 그리고 오직 그 경우에 참이다. 정확히 하나의 F가 존재할 경우 한정 기술구 F’, Russell의 용어법에 따르면 특정 대상을 지칭denote하며, 우리의 느슨한 용어법에 따르면 특정 대상을 지시한다. 그런데 예를 들어 그 고양이라는 한정 기술구에 대해 이 세상에 존재하는 모든 고양이가 그 논의영역으로 취해진다면 (고양이가 딱 한 마리만 남아있는 슬픈 세계가 아닌 바에야) 지시체는 특정되지 않는다. 하지만 통상적으로 그러한 표현은 분명 지시체를 갖는다. 굳이 고양이가 딱 한 마리만 남은 슬픈 세계가 아니더라도, 이 현실세계에서 우리는 종종 그 고양이는 침대 위에 있다는 말을 무리 없이 사용한다. 따라서 그 말은 그 고양이’(그리고 목전에 있는 침대를 가리키는 그 침대’)에 아래 첨자가 부가된 그 고양이x는 침대y 위에 있다와 같은 식의 발화를 나타내는 것으로 간주될 수 있다. 여기서 xy에 대한 논의영역 제한자는 가령 화자가 기르는이 방에 있는 따위가 될 것이다.

암묵적인 맥락-상대성이 드러나는 또 다른 사례로서 일상에서 친숙한 것을 들 수 있다. 비근한 일이지만 비가 온다가 에딘버러에서는 참이면서도 아테네에서는 거짓일 수 있다. 동일한 한 진술의 진리치가, 발화의 시점이 주어진 경우 발화가 이뤄지는 장소에 따라 달라질 수 있으며, 마찬가지로 발화의 장소가 주어진 경우 발화가 이뤄지는 시점에 따라 달라질 수 있는 것이다(에딘버러에 있는 사람이 명시적으로 아테네에는 비 와하고 말함으로써 아테네의 날씨에 관해 말할 수도 있다). 마찬가지로 바람이 분다’, ‘시끄럽다’, ‘자스민 향기가 난다’, ‘밝다등의 진리치 역시 발화가 이뤄지는 시점 및 장소에 따라 달라질 수 있다.

 

철학적 개념: 가치(價値)value. 익숙하지만 다소 논쟁거리인 또 다른 사례는 일부 가치진술value statement들이 지닌 암묵적 상대성이다. Joe‘Beatles가 제일 위대한 가수야!’ 하고 말하고 Karen‘Rolling Stones가 제일 위대한 가수야!’ 하고 말한다. 두 사람의 의견 불일치는 진정한 불일치인가? 두 입장은 상호 모순되는가? 물론 두 사람은 이에 대해 논쟁을 벌일 수도 있을 것이다. 하지만 이 경우 두 사람은 각자의 진술에 암묵적 지표사가 포함되어 있다는 점을 받아들여야만 한다. 그래서 JoeBeatles그에게for him 가장 위대한 가수라고 말한 셈이며, KarenRolling Stone그녀에게 가장 위대한 가수라고 말한 셈이다. 그 경우 두 사람의 입장은 동시에 참일 수 있다.

이와 동일한 종류의 트릭은 윤리적 진술, 미학(美學)aesthetic 진술, 취향(趣向)taste에 관한 진술 등 가치에 관한 그 어떤 진술에서든 일어날 수 있다. 가치평가와 얽힌 논쟁에 숨은 이러한 혼동은 JoeKaren 사례에서 충분히 해명되었을 것이다. 좀 더 뚜렷한 경우를 들자면, 민트초코가 맛있는지 여부에 대한 논쟁과 같이 미각적 취향에 관한 논쟁에서 이러한 혼동이 빈번하게 발생한다. 민트초코는 어떤 사람에게는 맛있게 느껴지고 그와 다른 사람에게는 그렇지 않다. 오로지 극단적인 객관주의자라든가 음식 스노비즘에 취한 사람만이, 둘 중 어느 취향이 옳은지를 결정해줄 객관적인 사실이 반드시 있어야만 한다고 주장할 것이다. 어떤 사람은 민트초코를 좋아하고 그와 다른 사람들은 그것을 싫어하는 것일 뿐, 그게 전부다. 이와는 대조적으로, 가령 임신중절이 올바른지 여부와 같은 무겁고 심각한 윤리적 사안에 관한 논쟁에서는 의견대립이 이런 식으로 해소되는 일이 드물다. ‘하지만 임신중절은 어쨌든 나한테는 나쁜 게 아냐!’ 하고 말한다 해서, 낙태를 공공연히 긍정하는 입장에 가담하지 않게 되는 것은 결코 아니다. 이는 다소 논쟁의 여지가 있는 중간지대로서, 외견상 분명해 보이는 논쟁이 진정한 논쟁인지, 혹은 n항 술어를 n+1항 술어로 변환하여 지표사를 삽입시킴으로써 그 논쟁이 진정 축소될 수 있는지 여부는 진지하게 따져보아야 할 사안이다.[?]

상대화된relativised 형태의 가치진술에서 적절한 관계항이 될 수 있는 것은 개별 화자에만 국한되지 않는다. ‘Ax에게 가치가 있다라는 형식의 진술에서 x는 개별 화자뿐만 아니라 특정 인종, 국가, 사회집단의 구성원들과 같은 복수의 사람들로 이뤄진 한 집합일 수도 있다.

 

철학적 개념: 지식. 당신이 다음 문장을 말한다고 상상해보자:

 

(3) 나는 그녀가 욕실에 있는 것을 안다.

 

(당신은 방금 그녀가 욕실에 들어가는 모습을 보았으며 그 이후로 욕실 문은 줄곧 닫혀 있었다.) 그런데 당신은 철학 강의 시간에 다음과 같은 말을 한 적이 있다:

 

(4) 나는 이 모든 게 꿈이 아니라는 것을 알지 못한다.

 

두 진술은 동시에 참일 수 있는가? 우선 한편으로, (4)가 참이라면 (3)은 참일 수 없다고 생각할 것이다. 당신이 꿈꾸고 있는 게 아님을 전혀 알지 못한다면, 당신의 친구가 욕실에 들어가는 모습을 본 경험이 꿈이 아니었다는 것 역시 알지 못한다. 그렇다면 당신은 그녀가 욕실에 있다는 것을 알지 못하는 셈이다. 다른 한편으로, 지식에 대한 기준이 맥락에 따라 달라진다고 볼 수도 있다. 가령 회의론을 논하는 철학 강의 시간에서라면 지식에 대한 엄밀한 기준을 가정하는 것이 올바른 반면, (3)과 같은 문장을 말하는 일상적인 상황에서라면 그 기준이 다소 완화시킬 수 있다. 이 경우 위 두 진술은 동시에 참이 될 수 있다.

인식론적 맥락주의epistemic contextualism에 따르면 'Sp라는 것을 안다' 형식의 주장에 의해 표현되는 명제는, S가 누구이고 p의 내용이 무엇인지에 따라 달라질 뿐만 아니라, 발화의 시점에 어떤 기준이 적용되고 있는지에 따라서도 달라진다. 다시 한 번 말하지만 지식은 숨겨진 지표사를 지니고 있는바, ‘알다x와 같이 발화의 맥락에 의해 고정되는 변항을 암묵적으로 포함하고 있다. 여기서 매개변항 ‘x’의 자리는 특정 맥락에 따라 그 맥락에서 우세하거나 의도된 지식의 기준 내지 종류를 기술하는 어구로 채워진다. 따라서 우리는 그녀가 욕실에 있다는 것을 절대적으로simpliciter 알 수는 없지만, 그녀가 욕실에 있다는 것은 일상적인 목적에서 수는 있으며, 그 모든 게 꿈이 아니라는 것을 알지는회의론을 논박하려는 목적에서 못한다. 우리는 자신이 꿈을 꾸고 있지 않다는 것을 알지만일상적인 목적에서, 자신이 꿈으로 꾸고 있지 않음을 알지는회의론을 논박하려는 목적에서 못한다.

이보다 더욱 급진적인 견해로서 인식론적 맥락주의보다는 인식론적 상대주의epistemic relativism로 알려진 견해가 있다. 이에 따르면 예컨대 '나는 바닥이 평평하다는 것을 안다'가 발화될 경우 설사 그 발화맥락과 관계된 모든 사실들이 고정되더라도, 그 발화에 의해 표현되는 명제가 갖는 정확히 하나의 진리치 같은 것은 없다. 발화에 의해 표현되는 명제의 진리치는 진리치를 귀속시키는 [즉 명제의 진리치를 평가(評價)evaluate하는] 사람의 관점(觀點)perspective과 목적purpose에 따라서도 달라지는데, 대체로 진리치 평가자는 발화가 이뤄졌던 맥락에 포함되지 않기 때문이다. 예시로 든 발화가 동일한 특정 바닥에 관한 것이었다고 가정해보자. 그 경우 표현되는 동일한 명제는 일상적인 목적에 따르자면 참이겠지만 물리학 실험에 요구되는 엄밀한 기준에 따르자면 거짓일 것이다. 일반적으로 인식론적 상대주의자에 따르면, ‘As라는 것을 안다에 대해 As와 발화맥락이 결정됨으로써 특정 명제 내지 내용content이 고정되더라도, 그 명제 내지 내용의 진리치는 평가되는 관점에 따라 달라질 수 있다. 이러한 입장이 결코 주관주의(主觀主義)subjectivism는 아니다. 주관주의에 따르면 특정 표준들 집합이 선택되더라도 진술의 진리치에 대한 평가는 여전히 틀릴 수 있다(그리고 선택된 표준들이 올바른지 여부는, 항상 그렇지는 않더라도 대체로 지식-주장을 평가하는 사람이 처한 맥락의 특징에 따라 합리적으로 제약된다).

 

철학적 개념: 사실적 조건문. 5장에서 우리는 사실적 조건문counterfactual conditional을 간략히 살펴본 바 있다. 사실적 조건문은 진리-함수적으로 잘 설명되지 않는다. pq의 진리치는 통상적인 (‘직설법적indicative’) 조건문 ‘p이면 q이다의 진리치를 결정해준다. 실질적 조건문의 진리표에 따르면 () 일단 전건 p일 경우, 후건 q가 참이면 조건문 ‘p이면 q이다전체는 참이고 q가 거짓이면 조건문 전체는 거짓이며, () 일단 전건 p거짓일 경우, 후건 q가 어떤 진리치를 갖든 조건문 전체는 참이다. 따라서 실질적 조건문은 ‘p가 아니거나 q이다[pq]’와 동치이며, 이는 p가 거짓이거나 q가 참일 경우 참이다. [그래서 실질적 조건문의 진리-조건을 간단히 말해보자면 전건이 거짓이거나 후건이 참일 경우 조건문 전체는 참이다.]

물론 실질적 조건문을 진리-함수적으로 설명하는 데 대해서는 논쟁의 여지가 있기는 하다. 하지만 이보다 더욱 심각한 문제는, ‘p라면 q일 것이다If p were so, then q would be so’ (혹은 ‘p였더라면 q였을 것이다If p had been so, then q would have been so’)와 같은 형식의 사실적 조건문 내지 가정법적subjunctive 조건문의 경우에는 명백히 진리-함수적으로 설명되지 않는다는 것이다. 일상에서 사실적 조건문을 발화하는 경우 우리는 p가 사실이 아님을 (그리고 보통 q 역시 사실이 아님을) 이미 알고 있긴 하지만, 그렇다고 해서 그 사실적 조건문이 참인지 여부가 분명하게 결정되지는 않는다. 예를 들어 다음 문장을 생각해보자:

 

(5) 만약 Ronald Amundsen이 최초의 남극 탐험가가 아니었다면, Robert Scott이 최초의 남극 탐험가였을 것이다.

 

실제로 최초의 남극 탐험가는 Scott이 아니라 Amundsen이었지만, 이 사실은 (5)의 진리치를 결정해주지 않는다.

사실적 조건문에 대한 영향력 있는 설명으로서 David Lewis의 설명에 따르면 이러한 조건문의 진리치는 다음과 같은 절차에 따라 결정된다: 전건이 참이라는 점만 제외하고는 현실세계actual world와 가장 유사한 가능세계를 생각해보자. 위 사례의 경우 그 가능세계에서 Amundsen은 최초의 남극 탐험가가 아니다. 나머지 모든 가능한 사항들이 현실세계와 정확히 동일한 그 세계에서, 후건은 참인가? 만약 그렇다면 조건문 전체는 참이며 그렇지 않다면 조건문 전체는 거짓이다.

하지만 이러한 설명마저 진리치를 결정하는 데에 충분하지 않은 사실적 조건문의 사례들이 있다. 다음 두 문장을 보자(이는 Quine이 제시한 것으로서, 이 문장에서 배경으로 깔린 전쟁은 1950-3년까지 일어났던 한국전쟁이다):

 

(6) 만약 Caesar가 총사령관이었다면, 그는 원자폭탄을 사용했을 것이다.

(7) 만약 Caesar가 총사령관이었다면, 그는 투석기(投石機)를 사용했을 것이다.

 

일견 두 문장은 동시에 참일 수 없는 것처럼 여겨진다. 하지만 Lewis에 따르면 이 두 조건문은 발화의 맥락에 따라 각기 참일 수 있다. 우선 우리가 Caesar의 무자비한 성격을 고정한 채, 전쟁무기에 대한 그의 지식을 기원적 1세기에 이용할 수 있었던 것에만 제한하는 게 아니라 그가 20세기의 전쟁무기 또한 알고 있다고 가정한다면, 우리는 그에 상응하는 가능세계를 선택할 것이며 그에 따라 (6)은 참이고 (7)은 거짓으로 결정될 것이다. 하지만 그의 정신적 상태나 성향들을 더욱 많이 반영하여 그의 성격 뿐 아니라 그가 과거에 지녔을 전반적인 지식(과 무지)마저 그대로 유지시킨다면, 우리는 (7)이 참이고 (6)은 거짓이라 판정할 것이다. 이렇듯 사실적 조건문의 진리-조건은 부분적으로 발화의 맥락에 의존적인바, 특히 조건문의 주제, 암묵적인 가정, 대화에서의 관심사 등에 따라 달라진다.

 

철저한 맥락주의radical contextualism. 우리는 명시적 지표사를 포함하고 있지 않으면서도 맥락-의존성을 띠는 많은 사례들이 있음을 확인해왔다(이번 각주1)(原註) 참조). 이쯤 되면 독자들은 맥락-의존성이 도대체 어디까지 뻗칠 수 있는지 궁금해 할 것이다. 그리고 어떤 견해에 따르면 모든 언어표현들이 맥락에 민감할 수도 있다! 7장에서 만났던 인물인 Austin은 이러한 견해를 지니고 있었다. 유명한 구절에서 그는 다음과 같이 말한다:

 

특정 언어에서 온전하게 형성된 여러 다양한 문장들을 생각해본다면 그것들을 참인 것과 거짓인 것으로 단순하게 분류할 수는 없다. 왜냐하면 참과 거짓의 문제는 문장이 무엇이며 무엇을 의미하는지에 따라서만 좌우되는 게 아니라, 대략적으로 말하자면 문장이 발화되는 환경에 따라서도 좌우되기 때문이다. 그러한 문장들은 참도 거짓도 아니다.

 

(Austin, 1962[단어를 사용하여 어떻게 행위가 이뤄지는가How to Do Things With Words], 110-11.)

 

그러면서 그는 다음과 같은 사례를 제시한다:

 

다음 문장을 생각해보라: ‘Raglan 경은 알마 전투에서 승리했다’. 여기서, 알마 전투는 설령 단 한 명만이 있었다 하더라도 어쨌든 병사가 수행한 전투였으며, Raglan 경의 명령이 일부 부하들에게 제대로 전달되지 않았다는 사실을 유념하자. 그렇다면 Raglan 경은 알마 전투에서 승리한 것인가, 아닌가? 물론 특정 맥락에서라면, 가령 학교에서 사용되는 역사교과서에서라면 그렇게 말하는 게 충분히 정당화될 것이다. 이는 약간 과장된exaggerated 것이긴 하지만, 어쨌든 Raglan에게 승리의 메달을 부여하는 데에는 이의가 없을 것이다. ‘Raglan 경은 알마 전투에서 승리했다[많은 세부사항들이 생략된 채 나머지 일부가] 과장된 문장으로서, 어떤 맥락에서는 적합하겠지만 다른 맥락에서는 그렇지 않다. 그러니 그 문장이 참이거나 거짓이라고 한사코 고집하는 것은 무의미하다.

 

(143-4.)

 

Austin의 이러한 기본 착상에 동의하는 인물들로는 後期 WittgensteinJohn R. Searle 및 비교적 최근의 인물들인 Charles TravisAvner Baz 등을 들 수 있겠다. 그 어떤 문장의 진리-조건도 맥락으로부터 자유롭지 않다는 급진적인 맥락주의적 견해는, 다음과 같이 단순하면서도 문제될 것이 없는 명백한 사례를 통해서 살펴불 수 있다(이는 Travis에 의해 논의된 사례를 약간 변형한 것이다): 당신이 새로 산 값비싼 명품 신발에 관해 내가 , 니 신발 니 침대 아래 있더라하고 말한다. 근데 사실 그 신발은 당신의 방 안에 있지 않다. 그러니 대부분의 발화맥락에서라면 내 말은 거짓일 것이다. 하지만 그 신발이 당신의 방 아래층에 있다고 해보자. 즉 그 신발은 당신의 침대와 지구표면 사이의 공간에 있는 셈이다. 그리고 당신의 침대는 물침대인 데다가, 이전에 한 번 터져서 아래층에까지 물난리가 난 적이 있다 해보자. 그러니 나는 그 비싼 신발을 포함하여 무엇이 되었든 당신의 침대보다 아래에 놓아진 물건들을 조심해야 한다고 경고하고자 그렇게 말한 것이다. 여기서 핵심은 이런 맥락에서라면 나는 참된 무언가를 말한 셈이라는 것이다.

 

 

허구적 대상

 

Frege의 의미론에서는 문장에 포함된 이름이 아무 것도 지칭하지 않을 경우 문장 전체는 사고(思考)though(온전한 뜻)를 표현하더라도 진리치를 갖는 데에는 실패한다(, 그 이름이 라고 믿는다와 같은 명제태도 연산자의 범위 내에 나타나는 경우는 제외). 그런데 이에 따르면, 일례로 Odyssey가 꾸며낸 이야기이고 그 등장인물들이 허구적(虛構的)fictional이라는 점(이러한 허구적 대상이 이번 에서 논의될 주제이다)을 감안했을 때, ‘Odysseus는 존재하지 않는다라는 명백히 참인 문장은 참도 거짓도 아닌 것으로 판명된다. Russell의 이론은 이 문제와 관련하여 좀 더 나은 위치에 있다. 그에 따르면 일상적인 고유명ordinary proper name은 위장된in disguise 한정기술구이다. 일상적인 고유명 ‘Odysseus’, 특정 발화를 가정하건대 대략 ‘Cyclops를 눈멀게 만든 그리스의 전사정도의 한정 기술구가 축약된 것이다. 그리고 이 한정기술구는 Russell이 제시한 기술구 이론에 따라 분석될 것이다(3, ‘기술구 이론의 적용참조). 그렇다면 가령 ‘Odysseus의 아내는 20년 동안 그를 기다렸다‘Odysseus의 아내는 20년 동안 그를 기다리지 않았다(이름 ‘Odysseus’를 대체하는3) 한정기술구에 대한 넓은-범위 해석에 따르면) 똑같이 거짓인 것으로 판명된다.


3) 사실 좀 더 정확히 말하자면 여기서 대체된 뒤 분석되어야 할 표현은 이름 ‘Odysseus’만이 아니라 ‘Odysseus의 아내라는 한정기술구인 듯하다.


(특정 사용 맥락 내에 있는) 이름이 실제로는기술구라는 생각은 다소 작위적이게 여겨진다. 하지만 그보다 더욱 심각한 문제는, Russell의 추정에 따르면 가령 다음과 같은 한 쌍의 문장들을 거짓이라 해야 한다는 점이다:

 

(8) Odysseus의 아내는 20년 동안 그를 기다렸다.

(9) Odysseus의 아내는 그가 떠난 지 3개월 만에 그를 기다리길 포기하고 구혼자들 중 가장 잘생긴 사람과 결혼하였다.

 

기술구 이론에 따르면 두 문장 모두 거짓이다. 하지만 우리가 생각하기에는 분명 (8)이 옳고 (9)는 뭔가 잘못되었다. 실제로 대부분의 사람들, 특히 언어철학을 접해본 바 없는 사람이라면 (8)을 주장하고 (9)는 부인하고자 할 것이다. (8)인 반면 (9)는 거짓이라 말할 것이다.

 

가능적인 대상possible object. 이 문제에 대한 한 가지 자연스러운 답변은, Odysseus가 현실적인 인물은 아니지만 가능적인 인물이라고 말하는 것이다. Homer의 이야기가 실제 역사인 가능세계가 있어서 그 세계에는 Odysseus가 실존하며 그 OdysseusHomer가 이야기해준 일들, Calypso에게 유혹을 당하고 외눈박이 괴물 Cyclops를 눈멀게 하고 전쟁과 모험이 끝난 뒤 20년 만에 자신의 신실하고 총명한 아내에게 돌아오는 등의 일들을 하였다. 위 두 문장의 앞부분에 ‘Odysseus가 실존하는 가능세계들에서는 라는 구가 암묵적으로 덧붙여져 있다고 상상한다면, (8)은 참이고 (9)는 거짓이 될 것이다. 그렇다면 이러한 제안은 허구적 실체fictional entity들이 가능적 실체possible entity들의 부분집합이라는 견해로서, 그 대부분은 -현실적인 가능세계에만 존재한다(그리고 Odysseus와는 달리 결코 생각되어본 적도 없는 허구적 실체들은 엄청나게 많을 것이다).

이러한 견해가 여기서 제시된 상태 그대로 지니고 있는 한 가지 문제점은 허구적 대상에 대한 과소-기술under-described의 문제이다. Odysseus에 관한 명제로서 Homer의 이야기와 일관적이지만 그 거짓 역시 Homer의 이야기와 일관적인 그러한 명제 p를 취해보자. 그 명제가 덧붙은 이야기에서는 가령 OdysseusCyclops를 눈멀게 하던 날 아침에 열일곱 개의 올리브 열매를 먹었다는 점 이외의 모든 것이 Homer의 원래 이야기와 정확히 동일하다고 해보자. Homer의 원래 이야기와 p가 둘 다 참인 세계를 Wp라 하고, Homer의 원래 이야기는 참인 반면 p는 거짓인 세계를 Wp라 해보자. 둘 중 어느 세계에 존재하는 Odysseus가 진짜real Odysseus인가? Wp-존재하는-Odysseus-유형인가 아니면 Wp-존재하는-Odysseus-유형인가?4) 현실에 존재하는 실제 인물에 관해서라면 그 사람이 특정 어느 날 아침에 올리브 열일곱 개를 먹었는지 여부와 같은 문제에 대한 명백한 사실이 존재한다. 하지만 Odysseus의 경우엔 그러한 사실이 결정되어 있지 않다.


4) ‘Wp로 칭해지는 가능세계, Odysseus가 본문에서 말해진 여차여차한 일을 한 가능세계는 많으므로, 그 중 특정 하나의 세계에만 존재하는 Odysseus-개항이 아니라 그러한 가능세계들을 통틀어 존재하는 Odysseus-유형이 문제시된다. Wp들에 존재하는 Odysseus에 대해서도 마찬가지이다.


이 문제에 대한 한 가지 그럴 듯한 대응은 우리가 Odysseus에 관해 말할 때 실제로는 가능세계들의 한 집합에 관해, OdysseusHomer에 의해 명시적으로 귀속된 속성들 (및 그렇게 Homer에 의해 귀속된 속성들 집합으로부터 논리적으로 따라 나오는 속성들)을 갖고 있는 그러한 가능세계들의 집합에 관해 말하고 있다고 가정하는 것이다. 하지만 Homer에 의해 작품 Odyssey에서 Odysseus에 관해 표현된 명제들이라 할지라도 절대적이고 신성불가침한 것은 아니다. 우리는 HomerOdysseus에 관해 기술한 여타 이야기들을 그대로 받아들이면서도, 가령 그가 Siren의 노래를 들었을 때 뱃머리가 아니라 돛대에 묶였다고 상상해볼 수 있다. 그렇게 한다고 해도 우리가 생각하는 인물은 여전히 Odysseus이다. 하지만 이 경우 우리는 HomerOdysseus에게 연관시킨 속성들 중 얼마나 많은 것들을 다르게 상상해볼 수 있겠는가? 어느 정도라야 그것이 여전히 Odysseus의 특징을 이루는 이야기라고 인정될 수 있겠는가? 이는 얼마나 많은 천사들이 바늘 위에서 춤출 수 있는가?’ 하는 고전적인 문제와 비슷하다. 이렇듯 명백히 쓸모없는 물음에도 분명한 해답이 있어야 한다고 제안하는 이론이라면 그저 제쳐두어도 좋을 것이다.

 

-Meinong주의Neo-Meinongianism. 이는 19세기 말 Alexius Meinong에 의해 제안되었던 도식의 현대화된 버전이다. 허구적 실체가 통상적인 대상과는 달리 결정적indeterminate이라는 점을 받아들임으로써 허구적 실체를 그 어떤 의미에서든 확정적인definite 가능적 대상과 동일시하길 거부한다면, 앞서 살펴본 문제로부터 다소 벗어날 수 있다. 이러한 관점에서 보자면 Odysseus는 실존하지 않는다. 그는 Cyclops를 죽이는 등등의 일을 하긴 했지만, Cyclops를 죽이던 날 아침에 올리브 열매 열일곱 개를 먹지도 않았고 안 먹지도 않았다. 이 점에 있어서 Odysseus는 가능적인 대상들과 달리 결정적이다.

-Meinong주의는 두 가지로 나뉜다. 첫 번째 변형은 허구적 대상이 통상적인 대상과 마찬가지로 구체적(具體的)concrete이라고(하지만 여전히 실존하지는 않는다고) 주장한다. 이러한 주장은 OdysseusCyclops를 눈멀게 하는 따위의 물리적 행위를 했기 때문에 그가 어떤 식으로든 구체적이어야 한다는 생각과 잘 부합한다. 두 번째 변형은 허구적 대상이 구체적인 대상이 아니라 모종의 추상적인abstract 대상이라고 주장한다. 다음과 같이 특정 속성들 조합에 의해 개별화(個別化)individuate되는 포괄적(일반적) 개별자들generic individuals을 생각해보자: 예를 들어 샌 프란시스코 49er 팀의 쿼터백이라는 것은 여러 속성들이 조합된 것으로서 시간을 걸쳐 다수의 구체적 개별자들에 의해 현실화된다. 이를 Russell이 제안하듯이 한정기술구로 간주하기보다는, 시간을 걸쳐 동일성을 유지하는 추상적 대상으로 간주해볼 수도 있다(그래서 가령 Joe Montana가 이 역할에 대해 맺는 관계는 특정 기간 동안 그 역할을 수행함이라는 관계이다). Odysseus 역시 이와 마찬가지이다.

허구적 대상이 추상적이든 구체적이든 -Meinong주의가 주장하는 바의 핵심은, -실존적인nonexistent 대상들이 지니는 속성들에는 대체로 상당한 여유가 있다quite spare in their properties는 것이다. 그래서 매우 적은 수의 속성이라도, 심지어 단 하나의 속성이라도 -실존적 대상이 되는 데에는 충분하다.

 

창조주의Creationism. 그런데 허구적 대상에 대한 -Meinong주의적 관점은, 가령 Odyssey2,800년 전에 지어진 작품이기에 Odysseus2,800살 먹은 우연적인contingent 대상이라는 생각과 잘 부합하지 않는다. Kripke에 따르면 전형적인 고유명의 지시체는 한 대상에 대한 최초 명명(命名)식에까지 이어져 있는 의사소통의 연쇄 내지 지시 의도referential intention의 연쇄에 해 결정된다. 대체로 그 최초 명명식은 칭명하고자 하는 대상을 목전에 둔 채 이 대상을 ‘N’이라 칭하자와 같은 식으로 말함으로써 이뤄진다. Kripke는 허구적 이름이 이와 비슷한 방식으로 도입된다고 생각해볼 것을 제안한다. Odysseus보다는 훨씬 최근의 예시로서 Jane Austen의 소설 오만과 편견의 등장인물인 Darcy를 생각해보자. Austen은 그 작품을 저술함으로써, 가령 Elizabeth Bennet의 마음을 사로잡기 위해 여러 행위들을 한 특정 인물 Darcy에 관해 말하는 하나의 관습practice을 도입한 셈이다. 그녀가 Darcy를 창조한 것은 하나의 추상적 대상을 창조한 것이긴 하지만, 이 대상은 마치 지구의 적도와도 같은 우연적 대상으로서 시간 내에서 시작점을 가지며, 부분적으로는 언어적개념적지향적intentional 실체이다. 그리고 그것은 필연적으로 그녀의 창조이다. 그러므로 이 견해는 앞서 가능적 대상 이론에 가해졌던 반박을 다음과 같은 방식으로 물리칠 수 있다: 어떤 하나의 기술구가 Jane Austen의 창조에 대해 참이고 인과적으로 올바른 방식에 따라 Darcy와 연결된 경우, 그 기술구는 오로지 Darcy에 대한 것이다. 따라서 -실존과 얽힌 퍼즐은 아무것도 없다. DarcySnoopyOdysseus와 마찬가지로 정말로 실존하되 살과 피로 이뤄진 대상으로서 존재하는 게 아니라, 다양한 기술구의 형태로서, 지향된 것으로서, 이미지로서, 혹은 영화에서 배우에 의해 연기된 역할로서 존재할 뿐이다.

 

허구성 연산자fictional operator. 또 다른 견해는 우리가 Odysseus에 관해 말할 때 그 화행을 지배하는 암묵적인 연산자가 존재한다고 가정하는 것이다. 그 연산자를 언어적으로 명시하자면 다음곽 같은 이야기에 따르면 〔…〕내지는 통상 Homer가 지었다고 알려진 이야기에 따르면 〔…〕와 같은 식이 될 것이다. 이 연산자는 -내포적인hyper-intensional 연산자로 간주될 수 있는바, 그 경우 허구적 대상에 관한 화행은 그 연산자의 영향권 내에 나타나는 문장(및 그 문장이 함의(含意)implicature하는 바와 그 문장에 선제(先提)presupposition된 바)의 뜻 내지 개념적 내용에 기반한다. 이러한 접근법에 속하는 한 가지 흥미로운 관점으로서 가식(假飾) 이론pretense theory으로 알려진 견해에 따르면, 허구성 연산자를 특정 심리적 태도인 가장하기(인 척하기)pretending라는 측면에서 이해해볼 수 있다. 즉 허구성 연산자는 우리로 하여금 그러한 태도를 취하게끔 지시한다instruct. 이는 어린아이들이 허구적인 이야기를 들을 때 그 이야기가 실제 일어난 일이 아니라 하더라도 개의치 않고 진지하게 들으며 즐긴다는 평범한 사실과 잘 부합한다.

허구성 연산자를 도입하는 견해들이 공통적으로 직면하는 난점은 참과 허구가 혼합된 진술들, 가령 ‘Snoopy는 종종 자신의 개집 위에 누워있고는 한다. 그런 이미지는 매우 널리 알려져 있다라든가 ‘PlatonOdysseus에 대해 잘 알고 있었지만 당연히 그를 만나본 적은 없다와 같은 진술들을 설명하는 데에 어려움을 겪는다는 점이다. 이러한 진술들에서 허구성 연산자가 작동하고 있다고 간주한다면, 특정 단일 문장 내에서는 그 연산자가 외견상 임시방편적인ad hoc 방식으로 소멸되어야 하는 것처럼 여겨진다. [이번 단락에서 제시된 사례들에서는 두 번째 문장들에서 그 연산자가 소멸되어야 하는바, 그 문장들은 허구적 이야기 내의 사태들에 관한 문장이 아니라 그 허구적 이야기 자체에 관한 현실의 사태를 말하는 문장이기 때문이다. 문제는 이런 식으로 허구성 연산자의 범위를 부분적으로 제한하는 것이 임의적으로 이뤄질 수 밖에 없는 듯하다는 점이다.]

 

 

추론주의

 

의미에 관한 전통적인 이론가들은 언어의 사용에 관한 화용론이 의미론에 토대를 두고 있다고 여기고는 한다. 의미론은 다음과 같이 기본적으로 지시 개념과 연관되어 있다: 단칭용어는 전형적으로 대상을 지시하며 술어는 속성 및 관계를 지시한다. 일단 의미론을 학습하고 나면 우리는 갖가지 단어들을 다양한 화행에서 사용할 수 있는바 이에는 추론(推論)하기drawing inference라는 화행 역시 포함된다.

이러한 전통적 관점과 대조적으로, Wilfrid Sellars(1912-89)에 의해 표명되고 최근 Robert BrandomJaroslav Peregrin에 의해 옹호된 견해에 따르면, 언어에서 근본적인 것은 화용론 특히 추론하기라는 화행이다. 이러한 견해는 추론주의inferentialism이라 불린다. ‘그리고라는 논리상항을 생각해보자(여기서 말해지는 그리고‘LaurelHardy’처럼 주어나 명사를 결합시키는 역할이 아니라, 온전한 문장들을 연결시키는 역할을 하는 논리적 연결사로서의 그리고이다). 우선 12.1그리고의 진리표로서, 여기서 ‘P’‘Q’는 임의의 서술문을 나타낸다:

 

12.1 그리고의 진리표

P

Q

P 그리고 Q

T

T

T

T

F

F

F

T

F

F

F

F

 

이 표에 따르면 PQ 양자 모두가 개별적으로 참일 경우 ‘P 그리고 Q’ 역시 참이며, 그렇지 않을 경우 ‘P 그리고 Q’ 전체는 거짓이다. ‘그리고에 대한 표준적인 추론규칙inference rule, 그리고가 형식적인 추론reasoning증명(證明)proof도출(導出)(유도(誘導))derivation에 도입되는 경우의 규칙은 12.2와 같다:

 

12.1 그리고의 추론규칙

P, Q

P 그리고 Q

P 그리고 Q

P 그리고 Q

P

Q

 

이 표에 따르면 PQ 각각이 전제로 주어진 경우 ‘P 그리고 Q’를 추론하는 것이 허용되며, ‘P 그리고 Q’가 전제로 주어진 경우 PQ 각각을 추론하는 것이 허용된다.

전통적인 이론가들은 논리적 연결사에 대한 진리표를 기본적인 것으로 간주한다. 즉 우리가 단어를 사용하는 방식을 지도하는 지침으로서의 추론규칙은 진리표에 토대를 두고based 있는바, 진리표는 그리고의미 내지 의미론을 명시한다. 하지만 추론주의자에 따르면 실상은 그 역방향으로 이루어진다. 우리의 언어사용에서 근본적인 것은 오히려 추론규칙이다. 우리가 그리고라는 낱말[의 의미]을 숙달mastery했다고 장담할 수 있기 위해서는 그에 관한 추론규칙[즉 추론에서 그리고를 사용하는 방법]을 먼저 암묵적으로라도 알고 있어야만 한다는 것이다. ‘그리고에 관한 추론규칙에 숙달되어 있는 한 그 낱말의 의미를 파악하기 위해 그 이상의 것은 더 이상 요구되지 않는다. ‘그리고에 대한 추론규칙에 의해 명시된 바에 따라 그 낱말을 사용할 줄 아는 한 우리는 그 단어를 이해하고 있는 셈이다. 추론규칙은 한 단어가 관습적으로 사용되는 방식, 즉 그 단어에 대한 명제적 지식(무엇-에 대한 지식)know-what과 대비되는 실천적 지식(어떻게-에 대한 지식)know-how을 명시한다.

상술한바 그리고에 대한 추론규칙은 언어-규칙inter-linguistic” rule이다. 반면 언어-intra-linguistic규칙은 언어-진입linguistic-entry규칙과 언어-이탈linguistic-exit규칙으로 구성되는데, 이들 규칙들은 논리-적인non-logical 단어들의 사용에서 중요한 역할을 담당한다. 예를 들어 빨갛다라는 단어를 설명하는 경우 그 단어의 지시가 특정 속성이라고 말하는 대신, 우리는 다음과 같이 그 단어의 사용을 지배하는 규칙들을 명시할 수 있다: 한 대상이 빨갛게 보인다면 그것은 빨갛다고 주장하는 것이 허용되며[(언어-진입 규칙)], ‘빨간 거 가져와라는 명령을 받는다면 빨간 것을 가져와야 한다should[(언어-이탈 규칙)]. (그리고 이 경우 다음과 같은 언어-) 규칙 역시 존재한다: ‘그것은 빨갛다로부터 그것은 색깔을 갖고 있다’, ‘그것은 파랗지 않다등을 추론하는 것이 허용된다.)

이러한 접근법은 지시 진리개념보다는 상술한 바와 같은 규칙들에 대한 진술을 더욱 근본적인 것으로 간주한다. 추론규칙은 특히 우리가 단어들을 갖고 무엇을 해야 하는지obligated 그리고 무엇을 하는 게 허용되는지may를 말해준다. 요컨대 추론규칙은 의미론적 개념이 아니라 규범적normative 개념이다. ‘를 지시하다은 참이다라는 낱말들 자체 역시 이런 식으로 다뤄져야 한다. 아주 거칠게 말하자면 우리는 P라고 주장할 자격이 있는entiled to assert경우에만 P는 참이라고 주장할 자격이 있다. 마찬가지로 b = A라고 주장할 수 있는 경우에만 ‘b’의 지시체 = A라고 주장할 수 있다.

물론 우리가 할 수 있는 추론들 중 단어에 대한 우리의 숙달에 의해 정당화된다고 합당하게 설명될 수 있는 것들은 일부에 지나지 않는다. 예를 들어 어떤 사람이 자신의 가방에 샌드위치가 있다는 것을 알고 있다 해보자. 그 경우 그 사람이 여기 내 가방이 있다로부터 여기 점심거리가 좀 있다를 추론하는 것은 정당화되지만, 이는 그 사람이 몇몇 단어들에 숙달해 있기 때문에만 정당화되는 것은 아니다. 하지만 일부 추론들은 이런 식으로 단어의 숙달로 인해 정당화된다고 할 수 있다. 예를 들어 ‘AB의 북쪽에 있다‘BC의 북쪽에 있다로부터 ‘AC의 북쪽에 있다를 추론하는 경우가 이에 해당된다. 추론주의의 요지는 이런 종류의 추론들의 총체가 언어적 숙달 및 의미에 대한 지식을 구성한다constitute는 점이다.

 

 

슬러

 

여기서 우리가 논의하게 될 슬러slur5)란 보통 특정 인종, 성별, 성적 지향, 사회집단 등에 속하는 사람들을 폄하하기 위해 사용되는 단어들을 말한다. 남부 유럽, 특히 이탈리아인에 대한 멸칭표현인 ‘wop’이라든가, 여성을 비하하는 ’, 동성애자를 낮잡아 부르는 속어 ‘fairy’ 등이 이에 해당한다. 이번 절에서 주의할 점은, 슬러 표현의 경우 그것을 실지로 사용하는 게 아니라 가령 인용 부호로 처리함으로써 그 표현을 단지 언급한다고 해도, 그에 담긴 모욕감이나 무례함이 무효화되지는 않는다는 점이다. 사용되든 언급되든 슬러가 그 자체로 모욕적이게 받아들여진다는 것은 명백한 사실이다. 다만 우리는 슬러를 논함에 있어 마치 끔찍한 질병의 표본을 연구하는 과학자와 같은 자세로 그 불쾌감을 무릅써야 한다. 여기서는 짱깨라는 단어를 예시로 삼아 논의를 진행하고자 한다.6)


5) ‘slur’란 특정 대상이나 대상들 집단에 대한 멸칭표현 내지 경멸적 비속어를 총칭하는 용어로서, 우리에게 친숙한 역어로는 혐오표현을 들 수 있겠다. 그런데 혐오표현혐오발언으로 번역되는 ‘hate speech/utterance’와 더불어 경멸적인 느낌을 지닌 모든 형태의 언어적 표현 및 언어적 행위까지도 총칭하는 용어로 통용되는 편이다. 반면 이번 을 읽다 보면 알 수 있듯이, ‘slur’는 기본적으로 대상 내지 대상들 집합을 지시하는 단칭용어 혹은 일반용어 형태의 언어표현만을 일컫는바, 슬러의 뜻 및 지시와 같은 의미론적 속성은 무엇인지, 슬러가 문장에서 나타나는 경우 진리-함수적 상항들과의 관계에서 어떻게 작동하는지, 슬러가 사용되는 경우 화용론적으로 어떤 특성을 보이는지 등이 이번 의 주된 논의사항이다. 이런 구문론적의미론적화용론적 사안을 감안하여, ‘혐오표현이라 번역하는 대신 원단어를 굳이 번역하지 않고 한글로 슬러라 표기하였다. (참고로 William G. Lycan, 언어철학: 현대적 입문Philosophy of Language: A Contemporary Introduction, 서상복 , 책세상, 2012에서는 중상/비방으로 번역되었다.)

6) 원문의 사례는 앞서 언급된 ‘wop’으로서, 저자는 이 표현을 사례로 삼은 이유로 친숙하면서도 작금엔 잘 쓰이지 않는 구식 단어라는 점을 들고 있다. 하지만 한국 사회에서 짱깨는 지금도 일상에서 아주 빈번히 사용되는 단어이다.


슬러는 (과연 의미라는 것을 갖고 있다면) 대체 어떤 종류의 고유한 의미를 지니고 있는가? 우선 자연스럽게 떠오르는 생각은, 슬러 표현의 슬러적-의미slurry-meaning란 그 표현과 동일한 외연을 가지되 모멸적이지는 않은 용어의 뜻 내지 내용에 부가되는 내지 내용이라는 것이다. 따라서 짱깨중국인’(또는 중국 혈통’)과 동일한 외연을 갖지만 중국인과 달리 모욕적이다. 짱깨단지 중국인이라는 것만으로 경멸을 받을 만하다와 같은 식의 내용이 부가된 슬러이다.

하지만 혹자는 슬러가 -슬러적인 대응표현non-slurry counterpart과 동일한 외연을 갖는다는 주장을 불쾌하게 여길 수도 있다. 분명 다음 문장이 이라는 주장에는 논쟁의 여지가 매우 많다고 하겠다:

 

(10) 모든 중국인, 그리고 오로지 중국인만이 짱깨이다.

 

어떤 사람은 적어도 일부 중국인들, 그리고 아마도 대부분의 중국인들은 짱깨가 아니라고 말할 것이다. 또 어떤 사람은 사실상 아무도 짱깨가 아니라 말할 것이며, 어떤 사람은 필연적으로 아무도 짱깨가 아니라고 주장할 것이다.

어쨌든 슬러가 뜻 내지 내용의 층위에서 불쾌감을 유발하도록 작동한다는 다소 일반적인 관점에는 한 가지 중요한 문제가 발생한다. 만일 슬러가 그런 식으로 작동하는 게 사실이라면, 그러한 불쾌한 작용이 부정될negated 수 있어야 하기 때문이다. 그래서 '그 어떤 짱깨도 하지 않다'짱깨의 모멸함을 무효화시켜야만 한다. 어떤 인종차별주의자가 다음과 같이 말한다 해보자:

 

(11) 짱깨 하나가 파티에 올 것이다.

 

이 화행에 포함된 불쾌감은 다음을 주장함으로써 거부될 수 있어야 한다:

 

(12) 아니다, 그 어떤 짱깨도 파티에 오지 않을 것이다.

 

하지만 많은 사람들이 보기에 (12)를 주장하는 것은 여전히 인종차별적 용어를 묵인하고 있으며 따라서 인종차별적 화행을 수행하고 있는 셈이다. 다음 문장 역시 마찬가지이다:

 

(13) 짱깨가 파티에 온다면, 나는 다른 맥주를 마실 것이다.

 

설사 이 조건문의 전건이 참이 아닌 것으로 받아들여진다 하더라도, 이 문장을 발화하는 것은 명백히 인종차별적이다. 요컨대 슬러에 담긴 모멸감은 아니다라면 이다와 같은 진리-함수적인 수단에 의해 격리quarantine하고자 하여도 그 시도를 벗어난다leap out’.

뿐만 아니라 슬러slurriness은 타인의 발화에 대한 간접보고indirect report, 그녀는 S라고 말했다said that S형식의 문장을 통해 분리confinement하고자 하는 시도 역시 벗어나버린다. (14)(15)에 비해서는 조금이나마 덜 무례한 발언으로 여겨질 것이다:

 

(14) Jolinda는 다음과 같이 말했다: ‘짱깨 하나가 파티에 올 것이다.’

(15) Jolinda는 짱깨 하나가 파티에 올 것이라고 말했다.

 

두 문장은 각각 직접보고와 간접보고의 형태를 띠고 있는바, 전자에서는 단어들이 발화되었던 그대로 보고되는 반면 후자에서는 반드시 그럴 필요가 없다. 그렇다보니 (14)와 달리 (15)(11)에 포함된 인종차별적 느낌을 고스란히 이어받고 있는 듯하다. [간접보고에서는 발화된 슬러를 그대로 사용할 필요가 없는데도 (15)는 그렇게 하고 있기 때문이다.] 그러므로 (15)를 말하는 사람은 중국인을 대하는 Jolinda의 태도를 공유하는 것처럼 여겨질 것이다.

이런 식으로 고찰하다 보면 슬러를 화용론적으로 설명하는 것 역시 불가능해지는 듯하다. 슬러가 지닌 경멸적 내용이 Grice인 대화적 함의(含意)conversational implicature에 속하는 사안이라 해보자. 그 경우 슬러취하될 수cancellable 있어야 한다(7, ‘함의 개념의 적용참조). 친구가 나를 저녁 파티에 초대했는데 내가 나 지금 과제 중이야하고 말한다면, 나의 말은 파티에 가지 못한다를 대화적으로 함의한다. 하지만 내가 곧바로 근데 한 시간이면 끝날 듯?’ 하고 덧붙인다면 앞선 발화에서 함의된 바는 취소된다. 이와 마찬가지로, 슬러가 발화될 때 진행되는 모든 일들이 단지 함의에 속하는 사안이라면, 다음과 같이 말하는 것은 분명 모욕적이지 않게 받아들여져야 한다:

 

(16) 짱깨 하나가 파티에 오더라도, 나는 그 사람을 존중할 것이다.

 

하지만 나는 (16)(11)만큼이나 인종차별적이라고 생각한다.

다음으로, 역시 7, ‘Russell의 기술구 이론에 대한 StrawsonDonnellan의 반박에서 살펴본 바 있는 전문적인 의미의 선제 개념을 활용하여, 인종차별적 태도가 슬러의 사용에 선제(先提)되어presupposed 있다고 가정하더라도 문제는 해결되지 않는다. 어떤 내용이 대화에 선제되어 있다면 그 선제된 바는 취소될 수 있어야 한다. 가령 냉장고에서 맥주를 가져올 사람을 정하는 상황에는 냉장고에 맥주가 있다가 참이라는 것이 선제되어 있다. 그런데 그 상황에서 내가 , 그거 알아? 여기서 누가 갔다 올지 정해봤자 아무 의미도 없어. 사실 냉장고에 맥주 없음하고 말한다면, 우리의 대화에 선제된 바는 취소되고 냉장고에 갔다 올 사람을 정하는 일 자체가 중단될 것이다. 하지만 슬러의 경우 그것이 지닌 경멸적 내용은 특정 화행을 통해 일단 표현되고 나면 결코 소멸될 수 없다.7) 스컹크가 방귀를 뀌면 그 냄새가 자연히 퍼지듯, 슬러은 일단 표현되고 나면 되돌릴 수가 없다.


7) 앞선 냉장고 맥주 사례의 경우, 사실은 냉장고에 맥주가 없다는 것이 밝혀짐으로써 냉장고에 다녀올 사람을 정하는 우리의 대화 자체가 소용없어지게 되었다. 즉 선제된 바가 취소됨으로써 그 선제에 토대를 둔 화행이 무효화되었다. 이는 7에서 살펴본바 임의의 문장 PQ에 대해, PQ를 선제할 경우, Q가 거짓이라면 P는 진리치를 결여한다는 선제 개념의 분석과 일치하는 결과이다. 반면 슬러의 경우 적어도 슬러에 대해서는 이런 식의 결과가 나오지 않는다. 내가 , 오늘 파티에 짱깨 한 명 온다더라.’하고 말한다. 여기서 말해지는 중국인을 a라 한다면, 나의 발화에는 ‘a는 짱깨이다가 참이라는 것과 더불어 짱깨에 결부된 경멸감이 선제되어 있다. 그런데 곧바로 내가 근데 전에 봤는데 걔는 짱깨는 아니더라. 깔끔하고 얌전해.’ 하고 덧붙인다면, 설령 ‘a는 짱깨이다가 참이라는 선제는 취하될지 몰라도, 앞선 발화에 결부되었던 경멸감마저 없던 일이 되는 것은 아니다. 내가 어떤 식으로든 중국인을 비하하는 발언을 했다는 사실에는 변함이 없는 것이다.


슬러에 관한 논의의 주된 영역을 이루는 것은 바로 이런 사안들이다. 슬러를 바라보는 다양한 시각들과 더불어 이 문제에 대한 많은 해결책들이 제안되어왔다. 언어철학 입문서로서 그 모든 것들을 낱낱이 살펴보기에는 지면이 부족하다. 다만 여기서는 David Kaplan이 제시한 해결책의 대략적인 윤곽 정도를 살펴보고자 한다.

 

슬러에 관한 Kaplan의 견해. Frege는 뜻과 지시 이외에도 단어의 어조(語調)tone내지 색채colouring에 관해 언급한 바 있다. 가령 똥개cur와 다르다는 점을 설명할 때 그가 염두에 둔 것이 바로 이러한 어조 내지 색채이다. 두 단어는 동일한 지시 및 동일한 뜻을 지니고 있지만, 발화에서 사용될 경우 그가 칭한바 각기 다른 주관적심리적 이미지가 환기(喚起)된다. 이와 마찬가지로 ‘p 그리고 q’‘p 그러나 q’와 다르다. 전자와 달리 후자의 연언문은 청자로 하여금 첫 번째 연언지(連言枝)와 대조적인 내용이 두 번째 연언지로 말해질 것이라 기대하게끔 유도한다. [하지만 연언문 전체의 진리-조건은 둘 다 동일하다. 이렇듯 언어표현의 인지적 내용과 분명 구분되는 특성을 포착하기 위한 것이 바로 어조 내지 색채 개념이다.]8)


8) “Frege는 표현의 사고와 그가 말한 색채라는 것을 구분한다. 과학적 언어는 사고를 명확하게 나타내고자 한다. 반면 인문학의 문장들은 아뿔싸!’맙소사와 같은 단어나 구를 중간에 삽입함으로써, 혹은 대신 개새끼와 같이 비하하는 말을 사용함으로써, 특정 느낌을 표현하는 색채의 옷을 걸치고 있는 경우가 있다. 문장의 그런 특성이 문장의 참에 영향을 주지는 않는다. ‘가 사용된 자리에 개새끼를 대체한 문장의 경우, 설사 그 문장을 발화하는 화자가 그 말이 표현하는 적대감을 지니고 있지는 않다고 하더라도, 그것만으로 그 문장이 거짓이 되는 것은 아니다. (PW[遺稿集Posthumous Writings, Blackwell, 1979], 140.) (中略) 자연언어의 문법에는 논리적인 것과 심리적인 것이 혼합되어 있다. 그렇지 않다면 모든 언어는 동일한 문법을 지닐 것이다. 언어의 미묘한 색채는 번역을 어렵게 만드는 요인이긴 하지만, 다른 언어를 학습할 때 그 뼈대가 되는 논리적 뜻과 그 언어가 지닐 수 있는바 뜻과는 다른 종류의 요소를 구분하는 데에 도움이 된다는 것을 시사하기도 한다. 색채가 문장의 아름다움에 중요할 수는 있겠지만, 아름다움이 진리는 아니며 그 역도 아니다.” (Anthony Kenny, Frege, 현대 분석철학의 창시자에 대한 소개Frege: An Introduction to the Founder of Modern Analytic Philosophy, 최원배 , 서광사, 2002, 255-6. 본디 해당 저서에서는 ‘colouring’묘미로 번역되었다.) 이에서 알 수 있듯 Frege가 어조나 표현의 색채 개념에 주목한 본래 목적은 언어의 객관적인 뜻(및 지시)과 주관적심리적 느낌을 구분하기 위함이었다.


그래서 우리는 슬러을 이러한 어조에 속하는 것으로 생각해볼 수 있다. 왜냐하면 슬러과 비슷하게 어조의 경우에도, 단어가 사용됨으로써 그 어조가 일단 작동하고 나면 그것을 격리하고자 하는 시도로부터 영향을 받지 않는다는 특성을 갖기 때문이다. 예를 들어 조건문 형식의 니가 그 똥개를 데려온다면, 우리 고양이는 달아나버릴 거다는 말은, ‘너 그 똥개를 기어이 데려왔구나!’ 못지않게 비방적인 말로 받아들여질 것이다.

하지만 어조 개념이 슬러을 적절히 설명해줄 수 있을 만큼 안정적인지 여부는 다소 의심스럽다. ‘그녀는 흡연자이고 술은 마시지 않는다그녀는 흡연자이지만 술은 마시지 않는다의 어조 차이, 즉 후자 연언문의 경우 첫 번째 연언지에서 기대되는 바[(그녀는 흡연자이니 자연스레 음주도 할 것이라는 예상)]가 두 번째 연언지에서는 좌절될 것이라는 어감의 차이가 환기된다는 사실은 관습(慣習)convention의 문제인 것처럼 여겨진다. 그런데 어조가 관습에 속하는 사안이라면, 어조는 그저 의미의 일부로 편입되어버리는 그 무언가인 셈이다. 다시 말해 어조는 Frege가 예상한 식으로 단지 주관적 세계에 속하는 게 아니라 언어의 객관적 사용의 일부가 되어버리는 것이다. 그러니 짱깨의 슬러역시 언어에 관한 객관적 사실이 되어버린다.

이 지점에서 Kaplan이 제안하는 방책은, Frege적인 어조를 본질적으로 객관화objectivise하는 규약적 의미convention meaning라는 범주를 도입하는 것이다.9) 이에 따르면 적어도 일부 단어들의 경우 지시 및 진리-조건과 같은 기술적(記述的) 기능descriptive function 이외에도, Kaplan이 칭한바 표현적(表現的)expressive 의미 내지 기능을 지니고 있다. 단어의 표현적 의미란 화자의 상태state 내지 태도attitude로서, 이는 기술적으로 전달descriptively convey되지는 않지만, 즉 문장의 진리-조건에는 아무런 영향도 미치지 않지만, 그 용어의 사용에 관습적으로 결부된다conventionally tied. 예를 들어 저기 빌어먹을 Smith 씨가 오고 있네저기 Smith 씨가 오고 있네와 기술적으로 동등descriptively equivalent하지만 표현적으로는 동등하지 않다expressively inequivalent. 또한 표현적 의미는 진리-함수적 연산자 내부에 삽입되지 않는다. ‘저기 빌어먹을 Smith 씨가 오고 있다저기 빌어먹을 Smith 씨가 오고 있다는 것은 사실이 아니다는 상호모순이지만, 양자의 표현적 의미는 동일하다. 즉 표현적 의미 역시 진리-함수적 수단으로 분리시키고자 하는 시도를 벗어난다. 그렇기에 만일 당신이 Smith 씨에 대한 화자의 태도를 공유하고 있지 않다면 [‘아냐, 저기 빌어먹을 Smith 씨가 오고 있다는 건 사실이 아니야라고 말하기보다는] ‘! 뭔 말을 그렇게 해!’ 하는 식으로 반응할 것이다.


9) (原註) 물론 윤리학 이론에서 암묵적으로든 명시적으로든 정서주의(情緖主義)emotivism를 옹호하는 사람들 역시 이러한 견해에 개입한다.


표현적 의미를 도입하는 이론이 그 자체로 합당하게 여겨지는 이유들 중 하나는, ‘아야!’, ‘젠장!’, ‘안녕!’과 같은 표현들이 문장 내에 적절히 삽입될 수 없음에도 왜 유의미한지를 잘 설명해낸다는 점이다. 그런 표현들은 기술적으로 무의미하기에 의미론의 구성적 규칙에 따라 적절히 연산될 수 있는 항목을 지니고 있지는 않지만, 표현적으로는 유의미하다. 하지만 각기 다른 언어로 된 인사말들이나 감탄사들에서 볼 수 있듯이 그런 표현들은 모두 관습적이다.

이런 설명에 따르면 짱깨중국인의 기술적 기능은 동일하지만 양자의 표현적 기능은 극적으로 다르다. ‘짱깨의 표현적 기능은 중국인들에 대한 혐오나 경멸감을 표현하는 것to express이다. 이를 화행론(話行論)speech-act theory의 관점에서 설명하자면, 슬러가 사용될 때 이뤄지는 발화수반행위는 모욕하기 내지 욕설하기라는 행위이다(이런 점에서 슬러는 V자 표시나 손가락 욕설과 표현적으로 다소 비슷하다). 그리고 이것이 슬러가 불쾌한 말로 받아들여지는 이유이다. 보통 상황에서라면 짱개라는 단어를 사용하면서도 중국인에 대한 경멸감을 표현하지 않을 수는 없다. ‘짱깨 하나가 파티에 올 것이다라고 말한다면 당신은 인종차별을 하고 있다는 비난을 면할 수 없다. ‘아무 짱깨도 파티에 오지 않을 것이다라고 해도 사정은 마찬가지이다. 그것은 그 자체로 인종차별적인 단어racist word이다.

실제로든 가짜로든 화자의 태도를 표현하는 것은, ‘빌어먹을 Smith 라든가 고결한 Smith 와 같은 별칭표현들이 일반적으로 지니는 표현적 기능이다. 여기서 한 가지 있을 수 있는 오해를 불식할 필요가 있겠다. 한 태도를 표현하는 것과 그 태도를 지니고 있다고 말하는 것은 다르다. 왜냐하면 ‘Smith 씨가 오고 있다저 빌어먹을 Smith 씨가 오고 있다의 진리-조건이 동일하다면, 분명 둘 중 어느 것도 화자에 관해 무언가를 말하는 진술과 동등할 수는 없기 때문이다. [상술했듯이 표현적 의미는 진리-조건과 같은 기술적 의미에는 아무런 영향을 미치지 않기에 두 문장의 진리-조건은 동일하다. 그런데 특정 태도를 표현하는 것과 화자가 그 태도를 지니고 있다는 진술이 동일하다고 가정한다면 후자 문장은 화자의 태도에 관한 진술인 셈이고, 따라서 전자 문장과 다른 진리-조건을 지니게 되어버린다. 이러한 귀결은 (동일성의 이행성 원리에도 어긋날 뿐만 아니라) 표현적 의미가 기술적 의미와 무관하다는 기본 논제와 배치되는바, 이를 막기 위해서는 태도를 표현하는 것과 화자의 태도를 진술하는 것 간의 동일성 가정이 거부되어야 한다.] 뿐만 아니라 단어가 전용(轉用)appropriation되는 현상과 같은 경우에서 볼 수 있듯이, 시간이 지남에 따라 기술적 내용에는 영향을 끼치지 않으면서도 표현적 의미가 변하기도 한다. 대표적인 사례로 퀴어queer라는 단어를 들 수 있겠다. 이는 과거에 동성애자들을 비하하고 모욕하려는 의도로 사용되었지만, 작금에는 동성애자나 양성애자뿐만 아니라 이성애자들 사이에서도 모욕적이지 않은 용어로 통용되는 추세이다.

 

 

이번 의 요약

 

우리는 주장이 규칙에 지배되는 행위, 특히 규제적 규칙과 대조되는 구성적 규칙들 집합에 의해 성립하는 행위라고 가정하였다. 구성적 규칙이란 그 규칙이 없이는 특정 행위가 존재할 수 없는 규칙이다. 예를 들어 체스에서 한 게임말을 특정 방식으로 움직이는 수가 룩의 운용법으로 간주되는 이유는 체스의 규칙에 따른 것이라는 점에서 체스의 규칙은 구성적이다. 반면 속도제한과 같은 도로교통법은 규제적 규칙이다. 도로교통법이 준수되지 않는다고 해서 차량운행행위 자체가 존재하지 않게 되는 것은 아니다. 논의 초입에 우리는 주장을 구성하는 규칙으로서 p라고 믿는 경우에만 p라고 주장해야 한다는 규칙을 고찰해보았다. 무언가를 주장하는 사람은 필연적으로 이 규칙의 지배를 받는다. 주장을 이런 식으로 생각할 경우 Moore의 역설을 해결할 수 있는 것처럼 보인다. 그 역설에 따르면, ‘p이지만 나는 p라고 믿지 않는다형식의 주장은 논리적 모순은 아니지만, 이러한 문장을 말하는 사람은 모종의 논리적 곤경에 처한다. 여기서 감지되는 부적절성은 A를 이행할 의도가 없으면서도 ‘A를 하겠다고 약속한다고 말하는 경우와 유사하다. A를 하겠다고 약속하는 사람은 약속을 지배하는 규범, A를 이행해야만 한다는 규범의 지배를 받는다. 이와 마찬가지로 p를 주장하는 사람은 주장을 지배하는 규범, p를 믿어야한 한다는 규범의 지배를 받는다. 그리고 약속의 경우 진실되지 못한 약속을 하는 게 가능한 것과 마찬가지로, 주장의 경우 거짓말(혹은 실제로 참이지만 주장하는 사람 자신은 거짓이라 생각해서 의도한 거짓 말)을 하는 게 가능하다. 주장을 비재하는 규칙의 후보로서 믿음-규칙 이외에도 진리-규칙, 정당화-규칙, 지식-규칙 등을 들 수 있다.

 

우리는 맥락-의존성이 언어에서 명시적 지표사 이외의 부분들로 확장되어야 하는지 여부를 탐구하였다. 다음 사례들을 고찰해보면 그렇게 확장되어야만 하는 듯하다: (1) ‘크다와 같이 암묵적 상대성이 포함된 표현들. 이러한 표현들의 경우 맥락-의존성과 연관된 지표는 정도의 표준 내지 비교집합이다. (2) 일상에서 사용되는 양화사. 가령 담임선생님이 학급 학생들에게 다들 준비 됐니?’ 하고 묻는 경우, 그 발화에 사용된 보편 양화사의 속박범위는 세상에 존재하는 모든 것이 아니라 그 학급 학생들로만 암묵적으로 제한된다. (3) 발화의 시간 및 공간이 암묵적으로 제한되는 경우. 일상에서 사용되는 지금 비 와같은 문장은 여기라는 지표사에 의해 암묵적으로 제한되어 있다. (4) 가치에 관한 진술. ‘그것은 좋다와 같은 가치진술은 실상 그것은 나에게 좋다와 같이 암묵적으로 상대적인 경우가 많다. 여기서 암묵적인 관계항은 단일 화자가 아니라 여러 사람들 집합일 수 있다. 이외에도 지식-주장이 맥락에 민감하다는 견해를 살펴보면서 인식론적 맥락주의인식론적 상대주의를 구분하였다. 그리고 사실적 조건문과 얽힌 퍼즐, 즉 적형well-formed이면서 동일한 전건을 갖되 상충하는 후건들을 지닌 두 사실적 조건문의 진리치를 결정하는 문제에 대한 Lewis의 맥락주의적 해결책을 살펴보았다. Lewis의 관점에서 보자면 그러한 조건문들은 둘 중 하나를 선택하고자 의도되는 맥락에 상대적이다. 마지막으로 보통 Austin에 의해 주창된 것으로 간주되는 견해로서 사실상 일상언어의 모든 문장들이 맥락-민감성을 갖는다는 제안을 고찰해보았다.

 

허구적 이름에 대한 FregeRussell의 설명을 거부한 채 그러한 이름들이 진정한 고유명 즉 지시적 이름referring name이라고 가정해보자. 따라서 우리는 그런 이름들이 지시하는 적절한 대상을 찾아내야 한다. 우선 한 가지 가능한 방책은 허구적 이름이 단지 가능적 대상들을 지시한다는 것이다. 이 견해는 많은 비용이 들지 않는다는 장점이 있다. 이를 위해서는 여타 목적을 위한 가능자(可能者)possibilia 내지 가능세계들만이 요구되기 때문이다. 문제는 가능적 대상이 [속성의 측면에서] 충분히 결정되어 있는 데 반해 허구적 대상은 그렇지 않다는 점이다. 예를 들어 Snoopy와 동일한 대상이 되는 후보는 한 배에서 태어난 특정 수의 강아지들 중 하나여야 한다. 하지만 Snoopy 만화의 작가 Charles SchultzSnoopy와 함께 태어난 강아지들이 정확히 몇 마리인지 언급한 바가 없다. 그러니 우리는 Schultz의 이야기를 참이게 하는 대상들 부류에 개의치 말고 그 가능적 대상을 생각해야 한다.하지만 이조차도 너무 엄격한지 모른다. 세상 사람들 전부 SnoopyCharlie Brown의 개라고 알고 있더라도, SnoopyLinus의 개라고 생각하는 것도 충분히 납득 가능한 일이기 때문이다. 또 다른 전략은 허구적 대상이란 매우 기이한 종류의 대상으로서 특정 속성들을 지니는지 여부에 대해서는 여하한 사실도 없다는 점을 받아들이는 것이다. 그래서 Snoopy의 형제 강아지들이 몇 마리였는지에 관한 문제를 결정해줄 사실이란 일절 없다. 이러한 -Meinong주의에 따르면 허구적 대상은 실존하지도 않고 가능적 대상을 통해 설명되지도 않는바, 단지 존재론적 결정성ontological indeterminacy이라는 기이한 특성을 갖는다. 마지막으로 살펴본 방책은 상당히 범용한 관점으로서 SchultzSnoopy를 창조했다는 평범한 사실을 진지하게 문자 그대로 수용하는 것이다. 그래서 Snoopy는 만화에 그려진 그림으로서, Schultz가 지어낸 이야기에 따른 여차저차한 기술구들로서, 혹은 텔레비전에 방영되는 애니메이션 프로그램 등의 형식으로서 실존한다. 요컨대 Snoopy는 우연적 대상이자 문화적 산물의 형태로 실존한다.

 

의미론 및 언어표현의 지시적 힘은 표현의 사용 및 특히 추론에서의 사용에 우선하는 것으로 여겨지고는 한다. 추론주의는 이러한 그림을 역전시켜 추론을 지시에 우선하는 것으로 간주한다. 실제로도, ‘그리고라는 논리상항이 사용된 특정 추론 패턴을 올바르거나 그릇된 것으로 식별하는 일 자체가 바로 그 논리상항의 의미를 아는 것이다. 이는 여타 논리상항들 역시 마찬가지이다. 추론 개념을 확장하여, 가령 빨강색 공의 색깔에 관해 질문을 받을 경우 빨갛다라고 답하는 것, 그리고 빨간 공 가져와라는 명령을 받을 경우 실제 빨강색 공을 가져오는 것 역시 추론에 포함시킨다면, 이런 식의 추론주의적 설명은 언어 전체에 적용될 수 있으며 그에 따라 기존에는 명백해 보였던 지시 개념의 필요성은 상당 부분 축소된다(하지만 추론주의적 설명은 우리가 지시하다와 같은 단어에 숙달되는 것에도 적용될 수 있다). 추론주의자에게 핵심적인 개념은 규칙 혹은 규범이라는 일반적인 개념이다. 언어란 근본적으로 대상이나 사태를 표상하는 것이 아니라 규칙들의 체계system of rules이며, 전자는 원리적으로 후자에 의해 설명될 수 있다.

 

슬러 현상은 일견 슬러 표현의 뜻 내지 내용 측면에서 설명될 수 있는 것처럼 여겨진다. 추정컨대 모든 슬러에 대해, 그와 동일한 외연 혹은 지시를 지니되 그 뜻을 달리하는 -슬러적인 표현이 존재한다. 가령 짱깨에 대응하는 슬러적 표현은 중국인/중국 출신의정도가 될 것이다. 심각한 문제는 이러한 견해가 모든 중국인 그리고 오로지 중국인만이 짱깨이다가 참임을 함축한다는 것이다. 그렇다면 일부 중국인은 짱깨라는 것인가? 아니면 아무 중국인도 짱깨가 아닌가? ‘그 어떤 중국인도 필연적으로 짱깨가 아니다는 어떠한가? [이 모든 질문에 어떻게 답하든, ‘짱깨라는 단어가 갖는 모욕감은 소멸되거나 상쇄되지 않는다.] 진짜 난점은 그 어떤 목적으로 짱깨사용하더라도 불쾌하게 받아들여진다는 사실이다. 가령 부정문의 경우에도 마찬가지로서 여기엔 아무 짱깨도 없다여기 짱깨가 있다만큼이나 무례하게 받아들여진다. 슬러 현상을 선제라든가 대화적 함의와 같은 화용론적 개념을 통해 설명하더라도 사정은 나아지지 않는다. 비교적 최근에 이르러 슬러 현상에 대한 수많은 설명들이 제시되었는데, 그 중 우리는 상술된 문제를 아주 간편히 처리하는 관점 한 가지를 간략히 살펴보았다. Frege는 슬러가 지닌 무례함을 슬러 표현이 환기하는 주관적심리적 태도의 측면에서 고찰한다. 이를 Frege는 슬러 표현의 어조혹은 색채라 칭하였다. 하지만 색채가 언어표현을 사용함으로써 작동하게 된다면, 이는 표현에 의해 환기되는 경향이 있는 주관적 연상의 문제라기보다는 관습에 속하는 사안이며, 따라서 대략적으로 말하자면 의미와 연관된 사안인 셈이다. Kaplan은 기술적 의미 이외에 의미의 또 다른 차원으로서 그가 칭한바 표현적 의미라는 것을 상정한다. 표현적 의미는 기술적 의미에 영향을 미치지 않기에 언어표현의 지시 내지 그것이 기여하는 문장 전체의 진리-조건에도 아무런 영향을 미치지 않는다. ‘저기 빌어먹을 Smith 씨가 오고 있다‘Smith 씨가 오고 있다의 진리-조건은 동일하지만, 전자는 대체로 화자가 Smith에게 좋지 않은 태도를 지니고 있는 경우에만 사용된다(그 태도가 정당한지 여부는 무관하다). 표현적 의미는 객관적이고 규약적인 사안이라는 점에서는 기술적 의미와 일치하지만, 의미론적으로 구성적이지는 않다는 점에서 기술적 의미와 다르다. 가령 언어표현이 부정문에 나타나거나 조건문의 전후건에 삽입되더라도 그 단어의 표현적 의미에는 아무런 영향도 가해지지 않는다.

 

 

탐구문제

 

1 주장에 대해 Williamson이 제시한 지식-규칙은 너무 엄격하지 않은가? 일상에서 볼 수 있는 주다양한 사례들을 통해 생각해보라.

 

2 Grice는 주장이 그보다 넓은 범주로서 다음과 같은 -자연적 의미non-natural meaning의 한 형태라고 생각했다: S는 다음의 경우 그리고 오직 그 경우에 A의 행위에 의해 p라는 것을 -자연적으로 의미한다non-naturally mean: 특정 청자 H에 대해 (1) AS를 통해 H로 하여금 p라는 믿음을 형성하도록 의도한다intend. (2) SH로 하여금 (1)을 알아채도록recognise 의도한다. (3) SH로 하여금 (1)을 근거로 p라는 것을 믿도록 의도한다. 다소 거칠게 말하자면 무언가를 -자연적으로 의미한다는 것은, 화자가 청자로 하여금 화자의 행위에 근거하여 무언가를 믿도록 의도하고 있음을 청자가 알아차리게끔 행위하는 것이다. Grice가 제시한 유명한 사례는 다음과 같다: 내가 YX의 부인과 부적절하고 지나치게 친밀해 보이는 그림을 그려서 X에게 보여줄 경우, 그럼으로써 나는 YX의 부인과 부적절한 관계에 있다는 것을 -자연적으로 의미한다. Grice의 견해의 반례로서 -자연적 의미를 지니지 않는 주장의 사례가 있는가?

 

3 Le VerrierVulcan 행성(수성의 공전궤도 안쪽에 있다고 믿어졌던 실존하지 않는 작은 행성)의 존재를 믿었다. Vulcan허구적 대상인가? 우리가 살펴본바 실존하지 않는 대상에 관한 다양한 설명방식들을 이 사례에 적용해보라.

 

4 -실존적 대상 내지 허구적 대상에 대한 또 다른 아이디어는 그러한 대상이 실제로는 단지 마음--관념idea-in-one’s-mind에 불과하다는 생각이다. 이에 대해서는 다음과 같은 반박들이 가해질 수 있다: (1) 동일한 허구적 대상인 Santa Claus에 대해 사람들이 갖는 관념은 분명 각기 다르지만, 위 관점에 따르면 이러한 일은 불가능해야 한다. (2) Le VerrierVulcan이 실존하는 이유가 그가 특정 사고를 갖고 있기 때문이라는 생각을 거부했을 것이다. 그는 Vulcan이 실존하는 것은 천문학적 사실에 근거한다고 말할 것이다. 이러한 논박은 효력이 있는가?

 

5 자연언어로 이뤄진 거의 모든 문장들이 맥락에 민감하다는 생각은 정말로 합당하게 받아들여질 수 있는가?

 

6 Dummett(Brandom)은 슬러적 현상이 추론주의에 의해 설명될 수 있다고 제안한 적이 있다. 예를 들어 옛날에 독일인에 대해 쓰였던 인종차별적 용어 ‘Boche’는 다음과 같은 추론규칙에 의해 그 슬러적 의미를 지니게 되는 것으로 설명된다:

 

‘Boche’ 도입규칙: ‘x는 독인인이다로부터 ‘xBoche이다추론한다.

‘Boche’ 제거규칙: ‘xBoche이다로부터 ‘x는 다른 유럽인들보다 상스럽고 잔혹한 경향이 있다추론한다.

 

이는 ‘x는 녹색이다로부터 ‘x는 색깔을 지니고 있다로의 추론이 허용되는 것과 유사하다. 이러한 전략은 합당한가? 이 전략은 ‘Boche’가 부정 연산자의 범위 내부나 조건문의 전후건에서 나타나는 경우를 설득력 있게 처리해낼 수 있는가? 여타 슬러 역시 이런 식으로 설명될 수 있는가?

 

7 다음 추론을 생각해보자:

 

() Smith가 오고 있다. 따라서 저 빌어먹을 Smith가 오고 있다.

() 저 빌어먹을 Smith가 오고 있다. 따라서 Smith가 오고 있다.

 

두 추론은 올바른가? 타당성(妥當性)validity진리-조건의 보존preservation으로 간주한다면, 위 추론들은 타당한가? 타당성 개념을 정보information의 보존으로 간주한다면 어떻겠는가?

 

 

주요 읽을거리

 

이번 장에서 다뤄진 주제들에 관한 주요 문헌 및 추가적인 문헌 목록은 다소 역사적시간적인 순서에 따라 작성되었다. 실상 일반적으로 우리 논의에 더 적합한 것은 추가적인 문헌들이지만, 일부 고전적인참고자료들은 1차적인 주요 문헌으로 분류될 법하다. 각 목록은 우선 주제별로 나뉜 뒤 한 주제 내에서 연대순으로(즉 해당 문헌이 저술된 시기에 따라) 작성되었다.

 

주장: Charles Sanders Pierce, 믿음과 판단Belief and Judgement」〔1877; Gorrlob Frege, 사고(思考)Thought」〔1918, Frege 選集The Frege Reader(1997), 325-45쪽에 수록; Herbert Paul Grice, 의미Meaning」〔1957화자의미와 의도Utterer’s Meaning and Intentions」〔1987, 단어 사용에 관한 연구Studies in the Way of Words(1989), 213-23쪽 및 86-116쪽에 각각 수록; Michael Dummett, 의미이론이란 무엇인가?()What Is a Theory of Meaning?()」〔1975, 언어의 바다The Seas of Language(1993b), 34-93쪽에 수록: John Rogers Searle, 화행: 언어철학 小論Speech Acts: An Essay in the Philosophy of Language(1969); Donald Davidson, 의사소통과 규약Communication and Convention, 진리와 해석에 관한 탐구Inquiries into Truth and Interpretation(1984), 268-80쪽에 수록, 그리고 그의 Davidson 주요 選集The Essential Davidson(2006)도 참고할 것.

맥락-상대성: John Langshaw Austin, 단어를 사용하여 어떻게 행위가 이뤄지는가, 2(1962); Ludwig Wittgenstein, 철학적 探究Philosophical Investigation』〔1953, 4.

허구적 대상: Gottlob Frege, 뜻과 지시에 관하여On Sinn and Bedeutung」〔1892, Frege 選集The Frege Reader(1997), 151-71쪽에 수록; 논리학Logic, 같은 책, 230-31쪽에 수록; 논리학 입문Introduction to Logic, 遺稿集Posthumous Writings(1979), 191-2쪽에 수록; Alexius Meinong, 대상에 관한 이론The Theory of Objecs, 실재론과 현상학의 토대Realism and the Backgrounds of Phenomenology(1981), 76-117쪽에 수록; Bertrand Russell, 지칭에 관하여On Denoting(1905), 479-93.

추론주의: Wilfred Sellars, 경험주의와 심리철학Empiricism and the Philosophy of Mind(1956), 253-329; Robert Brandom, 이유를 명시하기: 추론주의 序說Articulating Reasons: An Introduction to Inferentialism(2000)

슬러: 어조 혹은 색채에 관한 Frege의 견해는 뜻와 지시에 관하여, Frege 읽기, 151-71쪽 중 155쪽 및 논리학, 같은 책, 227-50쪽 중 240쪽 참조.

 

 

추가적인 읽을거리

 

주장 개념을 다루는 최근 문헌들 중 가장 핵심적인 것은 Timothy Williamson, 아는 것과 주장하는 것Knowing and Asserting(1996), 489-523쪽이다. 또한 그의 지식과 그 한계Knowledge and Its Limits(2000)는 인식론과 연계하여 주장 개념을 고찰한다. 이 노선에서 영향력 있는 논문은 Keith DeRose, 주장, 지식, 맥락Assertion, Knowledge and Context(2002), 167-203쪽이다. 주장 개념을 다루는 논문들을 모은 탁월한 選集으로는, 편자들의 글 역시 포함된 Jessica Brown, Herman Cappelen , 주장: 최근의 철학적 小論(2011)이 있다. Sanford Goldberg , 옥스포드 핸드북: 주장The Oxford Handbook of Assertion(출간예정)도 참고할 것.

허구적 대상을 가능대상으로 간주하는 견해는 David Lewis, 허구에서의 진리Truth in Fiction에서 탐구되고 있으며, 이 논문은 철학논문 選集 卷Philosophical Papers Volume(1983), 261-75쪽에 수록되었다. -Meinong주의는 Terence Parsons, 허구적 대상에 대한 Meinong주의적 분석A Meinongian Analysis of Fictional Objects(1975), 73-86쪽 및 -실존적 대상Nonexistent Objects(1980)에서 옹호되었다. 창조주의 견해에 대해서는 Saul Kripke, 1973[: Oxford대학교에서 행한 John Locke 강연], 지시와 실존Reference and Existence(2013)와 더불어, Anthony Everett, Thomas Hofweber , 공허한 이름과 허구, 그리고 -실존에 관한 퍼즐Empty Names, Fiction and the Puzzles of Non-existence(2000) Graham Priest, -존재에 대하여: 지향성의 논리학과 형이상학Towards Non-Being: The Logic and Metaphysics of Intentionality(2005)도 참조할 것.

맥락-상대성을 옹호하는 문헌으로는 Charles Travis, 상황-민감성: 選集Occasion-Sensitivity: Selected Essays(2008), Avner Baz, 단어가 요구되는 때-일상언어철학을 옹호하여When Words Are Called For-In Defense of Ordinary Language Philosophy(2012), Anne Bezuidenhout, 맥락주의의 정합성The Coference of Contexualism(2006)을 보라. 맥락-상대성을 공박하는 문헌으로는 Herman Cappelen, Erine Lepore 共著, 둔감한 의미론: 의미론적 최소주의와 화행 다원주의에 대한 옹호Insensitive Semantics: A Defense of Semantic Minimalism and Speech-Act Pluralism(2005) 및 좀 더 최근의 것으로는 Emma Borg, 의미를 추구하기Pursuing Meaning(2012)를 보라. 맥락주의와 대조되는 상대주의에 대해서는 John MacFarlane, 민감성 평가: 상대적 참과 그 적용Assessment Sensitivity: Relative Truth and Its Applications(2014)를 보라.

추론주의에 관해서는 Jaroslav Peregrin, 추론주의: 왜 규칙이 중요한가Inferentialism: Why Rules Matter(2013)을 보라.

Slur에 관한 Kaplan의 예비적 논의에 대해서는 David Kaplan, 아야와 이런의 의미The Meaning of Ouch and Oops(2004)를 보라. Christopher Hom인종차별표현의 의미론The Semantics of Racial Epithets(2008), 416-40쪽에서 슬러에 관한 영향력 있는 설명을 개진하는데, 그에 따르면 슬러은 내용 층위의 현상으로서 사회적이고 역사적인 사실에 의해 제공된 인종차별주의가 지닌 경멸적인 내용이다. Elizabeth Camp슬러적 관점(觀點)Slurring Perspectives(2013), 330-49쪽에서 흥미로운 관점적perspectival설명을 제시하는데, 그에 따르면 우리는 슬러를 사용함으로써 청자로 하여금 사회적으로 구성된 관점perspective을 지니도록, 슬러의 대상 및 그와 연관된 집단을 대하는 모멸적 태도에 동참하도록 유도한다. Daniel Whiting그건 당신이 말한 게 아니라 당신이 말한 방식이다: 슬러와 규약적 함의It’s Not What You Said, It’s the Way You Siad It: Slurs and Conventional Implicature(2013), 364-77쪽에서 매우 통찰력 있는 착상의 윤곽을 제시한다. Luvell AndersonErnie Lepore슬러적 단어Slurring Words(2013), 25-48쪽에서 개진하는 견해에 따르면, 슬러의 해악성은 아무리 넓게 해석하더라도 내용의 측면에서 설명될 게 아니라 금지(禁止)prohibits의 측면에서 설명되어야 한다. 마지막으로 David Sosa가 편집한 나쁜 말Bad Words은 매우 고무적인 책이 될 것 같다(이 책을 집필하던 당시에는 출간 예정이었다).

 


댓글(0) 먼댓글(0) 좋아요(1)
좋아요
북마크하기찜하기
 
 
 

11

後期 Wittgenstein

 

Ludwig Wittgenstein이 공식적인 출판을 염두에 두고 쓴 저서는 논리-철학 論考Tractatus Logico-Philosophicus(1921)철학적 탐구Philosophical Investigations(死後 1953년에 출간) 단 두 권이다. 두 저서는 표면적으로는 매우 비슷하다. 두 권 모두 대체로 언어철학에 관해 논하고 있으며, 논증을 통해 특정 결론을 납득시키는 연속적인 산문 형식이 아니라 짤막한 글들이 모여 단속적으로 배열된 형식으로 저술되었다. 번뜩이는 단상들이 풍부하고 다양한 모습으로 결합된 이러한 스타일은, 최종 목적을 지향하는 일반적인 선형적 글쓰기 방식에 저항하는 듯하다.

하지만 내용 측면에서 두 저서의 차이점은 이루 말할 수 없이 크다. 논의의 목적상 우리는 論考의 저자로서의 젊은 Wittgenstein을 언어의 기능에 대한 Frege-Russell적인 관점의 핵심적인 부분을 받아들였던 인물로 간주할 수 있다4에서 살펴보았듯 적어도 論考에서 제시되었던 원자문장에 대한 그림이론은 언어, 언어의 객관성, 마음과 세계 간의 관계 등이 바로 지시reference 개념에 토대를 두고 있다는 Frege-Russell적인 관점에 입각하여 구상되었다. 물론 論考의 저자로서의 Wittgenstein은 보다 심층적인 차원에서는 FregeRussell의 견해에 동의하지 않는다, 그런데 탐구의 저자로서 後期 Wittgenstein의 관점에서 보자면 그러한 불일치는 더이상 중요한 사안이 아니다. 後期 Wittgenstein은 지시reference가 언어를 이해하는 데에 핵심적인 사항이 아니라고 생각하며, 언어의 헤아릴 길 없는 다양성과 다층성을 기술할 수 있는 체계적이고 최종적인 방법이란 존재하지 않는다고 단언한다. 언어에서 체계적인 일반성generality을 찾아내려 시도하는 것은 마치 중세 이탈리아의 도시를 인위적인 격자판에 억지로 끼워 맞추려는 것과 같다. 그 도시에는 구불구불한 도로들, 막다른 골목, 규칙적인 형태의 광장들, 터널, 다리, 심지어 길 위에 나 있는 길들도 있다. 물론 굳이 하고자 한다면 그러한 시도를 할 수는 있겠지만, 이는 그 도시 본연의 모습을 작위적으로 왜곡하는 처사에 지나지 않으며 이런 식으로 얻어낸 정보는 그 도시를 실제로 답사해서 얻어낼 수 있는 정보와 판이할 것이다. 이것이 이번 에서 보게 될 後期 Wittgenstein의 더욱 급진적인 언어관에 대한 비유이다.

 

 

언어게임

 

FregeRussell 및 젊은 Wittgenstein을 포함하여 대부분의 언어철학자들에 따르면 지시 내지 명명(命名)naming하기라는 개념은 언어에서 매우 근본적인fundamental 것이다. 論考의 언어를 활용하여 말해보자면 지시는 언어가 실재(實在)reality 즉 세계와 접촉하는 지점이다. 언어에 대한 고전적인 구상에서 근본적인 역할을 담당하는 또 다른 단어는 이해(理解)understanding의미이다. 後期 Wittgenstein에 따르면 이러한 단어들은 명료성에 대한 환상을 불러일으킬 뿐이다. 문장을 이해하는 것이란 그 의미파악grasp하는 것이라고 말할 때, 우리는 그저 종교적인 기도문을 암송하고 있는 데 지나지 않는다. 반면 우리가 현실의 언어를 좀 더 가까이에서 들여다본다면 언어에 매우 다차원적인 사태들이 있음(가령 한 단어를 이해한 것으로 간주되는 바가 다른 단어를 이해하는 일과는 상당히 다를 수 있다는 것)을 알게 될 것이며, 인간이 갖는 여타 기술(技術)skill이나 기능들로부터 특별히 언어적 능력만을 따로 떼어내어 탐구할 방도란 없음을 알게 될 것이다. 무언가를 이해한다는 것은 다양한 현상들이 지닌 세부 사항에 따라 달라지는바 이해를 구성하는 단일한 그 무엇이란 존재하지 않으며, 이해했느냐 아니냐 하는 양자택일적인 것이 아니라 정도(定度)degree의 문제이다.

이러한 점은 탐구의 초입에서 Wittgenstein이 묘사한 언어게임language game을 생각해보면 보다 분명하게 드러난다.

 

내가 어떤 사람에게 심부름을 시킨다. 나는 그 사람에게 빨간 사과 다섯 개라고 적힌 쪽지를 준다. 가게에 간 그 사람은 점원에게 쪽지를 건네고 점원은 사과라 적힌 수납장을 연다. 표에 적힌 빨강색이라는 단어를 본 점원은 그 반대편에서 색깔 샘플을 찾아낸다. 그는 일련의 기수를 다섯에 이르기까지 세면서(아마 이를 마음속으로 세었을 것이다) 그에 맞춰 앞의 샘플과 동일한 색상의 사과를 수납장에서 꺼낸다. 우리가 단어들을 사용하는 일은 이런 식으로 이뤄진다.

(Wittgenstein 1953, 1)

 

이는 무척이나 평범하고 하릴 없이 따분한 일이 묘사된 것이긴 하지만, 이러한 게임을 하기 위해 점원과 심부름꾼은 기실 엄청나게 다양한 기술들에 숙달해 있어야 한다. 이러한 활동에 참여하기 위해 우리가 숙달해야 할 단 하나의 기술이란 없으며, 숙달되어야 할 다양한 기술들 중 일부를 언어적 기술이라 칭하는 것은 매우 자의적인 것처럼 여겨진다. 언어적인 기술과 그 외의 기술들을 깔끔하고 명확하게 구분할 수는 없다. 쇼핑목록을 특정 장소에 가져가서 건네는 것, ‘사과라고 적인 수납장을 찾아내는 것, 필요한 색상 샘플을 찾아내어 그에 걸맞게 대응하는 것, 정확한 개수의 사과를 수납장에서 꺼내는 것 등, 이 모든 기술 내지 활동들은 Wittgenstein이 칭한바 삶의 형식forms of life들을 구성한다constitute. 이것들은 부분적으로 언어적인 측면과 언어적인 측면을 모두 지니고 있으며, 이러한 행위들이 유의미해질 수 있는 이유는 그것들이 전반적인 삶과 문화의 좀 더 넓은 영역에서 구현되고 있기 때문이다. 이것들 중 언어적인 현상이라 할 만한 것을 외따로 떼어내어 조사하는 것은 이 활동들이 유의미해지는 맥락을 일절 무시하는 처사이다. 이는 마치 쓸개를 신체에서 떼어내어, 그것이 신체에서 어떤 기능을 하는지는 완전히 무시한 채 그 기능이나 특성을 조사하는 것마냥 터무니없는 일이다.

언어게임이라는 명칭은 이러한 활동들을 폄하하거나 경시하고자 붙여진 것이 아니다. 이러한 활동들을 언어게임이라 부르는 요지는 현실의 삶에서 사용되는 단어들과 게임에서 쓰이는 말piece 간의 유사성을 강조하기 위함이다. 특정 말을 rook이라고 식별하는 것은 특정한 틀framework에 따라, 즉 체스 게임에서 통용되는 전반적인 규칙과 관행에 따라 유의미해지는바, 이러한 규칙 및 관습의 총체는 게임 참여자가 룩을 갖고 둘 수 있는 특정한 수()move를 기술한다. 뿐만 아니라 체스에서 게임 참여자는 규칙을 그저 무작정 따르지만은 않는다. 정해진 규칙 내에 주어진 수많은 기회와 경우의 수를 활용하여 창조적이고 극적으로 게임을 이끌어가기도 하는 것이다. 언어게임 역시 이와 마찬가지이다. 다만 언어게임이 이뤄지는 전반적인 틀이란 것이 지시 내지 의미에 대해 통상 요구되는 기준에 비해서는 훨씬 더 넓고 덜 결정적으로 제한되어 있다는 점만이 다를 뿐이다. 이해될 수 있는바 적절성을 갖춘 언어행위에 대한 필요조건이 언어게임의 규칙에 의해 일단 결정되고 나면, 실제 언어현상에서 그러한 제한사항들은 다양하게 변형되는 주제로 취급되기도 하고 다른 목적을 위해 얼마든지 변경될 수 있는 수단으로 취급되기도 한다.

이러한 관점 자체는 통상적인 언어적 기술(記述)linguistic description을 전적으로 잘못된 것으로 치부하는 게 아니다. ‘다섯이 다섯을, ‘빨강이 빨강을, ‘사과가 사과를 지시한다고 말하는 것은 분명 올바르며, 누군가 그렇게 말한다면 우리는 그 요지를 이해할 것이다. 다만 그렇게 말한다고 해서 앞서 살펴본 언어게임에 무언가 더해지는 것은 아무것도 없으며, 실제로 무슨 일이 진행되고 있는지에 대해 더 명료한 설명을 제공하게 되는 것도 아니다. 언어현상을 그런 식으로[의미론적으로] 기술하는 것이 어쨌든 모종의 유의미한 설명을 제공한다고 가정한다면, 이는 지시하다’, ‘뜻하다’, ‘의미하다등의 단어들이 으레 지니고 있을 것이라 여겨지는 일반성(一般性)에 대한 갈망carving for generality에 굴복하는 셈이다. 이러한 단어들은 다양한 언어활동들을 간략하게 제시하는 데에는 유용하겠지만, 이를 통해 얻어진 통찰력이란 그저 매우 일반적이기 때문에 극히 피상적인 종류에 지나지 않는다. 이러한 단어들은 다양한 언어게임들 간의 차이점을 조명함으로써 언어를 이해하는 데에 진정한 통찰력을 가져다준다기보다는, 그 차이점들을 단지 은폐할 뿐이다(WittgensteinLear 의 한 대사를 인용하여 내가 너희에게 차이들을 가르쳐 주겠다라고 말한다).

지시하다’, ‘뜻하다’, ‘라는 것을 의미한다등의 표현들이 수행하는 진정한 역할은, 하나의 배경background이 일단 이해되고 나면, 즉 한 언어게임이 이뤄지는 지점이 개념적으로 정립되고 나면, 그 언어게임에서 특정 목적을 위해 어떤 단어를 사용(使用)use해야 할지를 나타내는 일일 따름이다. 다소 인위적이지만 분명한 예를 들자면, 달리기 경주에서 선수들이 출발하기 전에 심판이 준비하고 말하는 경우를 들 수 있다. 그 말은 달리기 경주와 연관된 다양한 규칙, 예측, 전통적인 관습 등이 복잡하게 얽혀있는 하나의 배경이 일단 갖추어져 있다는 특정 맥락에서 발화됨으로써만 비로소 유의미해지는 것이다. 이와 마찬가지로, 앞 단락에서 기술 것처럼 빨강이 빨강을 지시한다는 식의 언어게임에 관해 우리가 말할 수 있는 것은, 상당히 많은 사안들이 선제(상정)presuppose되어 있을 뿐만 아니라 그런 식의 언어게임과 연관된 배경을 우리가 상당 부분 이미 이해하고 있기 때문이다. 일상에서 우리는 매우 다양한 삶의 형식들을 무의식중에 공유하고 있으며, 굳이 명시적으로 말하지 않더라도 그것들에 의존하고 있다. 스마트폰으로 통화하면서 복닥거리는 거리를 걸어가는 자신의 모습을 생각해보라. 이는 실제로는 매우 복잡한 활동이지만 우리는 그에 완전히 숙달되어 있기 때문에 이를 매우 쉽게 해낸다. 재삼 강조하지만 의미론적 어휘semantical vocabulary들이 제 역할을 수행할 수 있는 이유는, 우리의 다양한 습관과 성향을 구성하는 삶의 형식들이 배경에 갖춰진 것으로서 이해되고 있기 때문이다.

유념해야 할 또 다른 중요한 점은 앞서 기술된 대로의 언어게임이 완전하다complete는 것이다. 언어게임이 완전하다는 말의 요지는 앞서 묘사된 언어게임에 그 이상으로 추가될 세부 사항이 없다는 것이 아니다. 언어게임을 온전히 수행하기 위해 유일하게 숙달되어야 할 것으로서, 혹은 우리가 숙달master해야 할 소위 언어적 기술로서, 그러한 기술들 집합 이외의 무언가가 더 요구되지는 않는다는 것이다. 언어게임이 불완전하다거나 부분적partial이라고 생각할 이유란 없다. 앞의 언어게임에서 가게 점원은 자신이 사용하는 단어들을 충분히 이해하고 있다. 그 사람은 자신이 사용하는 단어들을 통해 특정 활동을 충분히 능숙하게 해내고 있으며, 이것이 하나의 언어게임에 대해 우리가 말할 수 있는 전부이다.

 

 

가족 유사성, 도구, 도시

 

한 아이가 앞서 기술된 언어게임에서 점원의 역할을 숙달하게 되었다 해보자. 이제 그 아이는 다른 사람들로부터 몇몇 요소들을 차용하기도 하고 아예 새로운 형식의 용어들을 습득하기도 하면서 점점 더 많은 언어게임들을 차츰 숙달해갈 것이다. 그 경우 우리는 그 아이가 숙달한 전반적인 기술들 집합의 그림을, 각각이 상호 얽혀 있으면서도 고유의 영역 및 고유한 목적들을 지니고 있는 다양한 셀cell들로 이루어진 것으로 생각해볼 수 있다. 그러한 요소들로는 이야기하는 것storytelling, 믿는 체 하는 것(가장(假裝)하는 것)making-believe, 학교에서 학생으로서 처신하는 것, 농담하는 것, 원하는 바를 드러내는 것, 무언가를 부탁하는 것 등등 매우 다양한 활동들이 있을 것이다. 중요한 점은 그 아이가 언어를 온전히 숙달한 것으로 여겨질 수 있는 명확한 시점은 없다는 것이다. 이는 정도의 문제matter of degree이다. 앞서 살펴보았듯이 언어를 습득한다는 것은 매우 다양한 능력 및 다양한 종류의 숙달과 연관되어 있다. 그렇다면 언어란 무엇인가? Wittgenstein의 대답은 이 물음이 단 하나로 답해질 수 없다는 것이다. Wittgenstein은 언어에 대한 -본질주의자anti-essentialist이다. 언어를 깔끔하고 명확하게 정의하는 것은 불가능하다. 그보다 언어라는 개념은 가족 유사성family resemblance 개념이라고 해야 한다.

가족 유사성 개념의 또 다른 예가 있다. 게임의 개념을 생각해보자(이는 Wittgenstein이 직접 들었던 유명한 예시이기도 하다. 그러니 그가 언어게임이라는 착상을 제시한 것은 상당히 미묘한 데가 있다). 게임이란 정확히 무엇인가? 어떤 것이 게임이 되기 위한 필요충분조건은 무엇인가? Wittgenstein에 따르면 이 질문에는 답이 없다. 하지만 그렇다고 해도 게임이라는 개념 내지 단어에는 문제될 것이 없다. 어떤 게임에서는 게임판과 말들이 쓰이지만(주사위놀이) 그렇지 않은 게임도 있다. 어떤 게임에서는 참여자들이 경쟁하지만 그렇지 않은 게임도 있다(혼자 하는 카드게임, 몸짓 알아맞히기 놀이). 어떤 게임에는 운이 작용하지만 그렇지 않은 게임도 있다(체스, 오목, 틱택토 게임). 어떤 게임은 팀을 구성하여 진행되지만 그렇지 않은 게임도 있다(전문 스포츠 경기). 이렇듯 게임 개념의 사례가 되는 데 대한 충분조건이라 할 수 있는 특성들은 매우 다양한바, 속성 AB, 또는 BC, 또는 CD를 갖는 것은 게임이라는 식으로 말할 수밖에 없다. 하지만 모든 게임에 고유하거나 공통되는 단 하나의 특성은 없다. 뿐만 아니라 게임 개념은 선언적 속성을 통해 온전히 정의될 수 있는 것도 아니다. 즉 과거에 존재했고 현재 존재하고 있는 모든 게임들이 갖는 속성들을 한데 모아 유한한 수의 선언지들로 취하여, 속성 AB, 또는 BC, 또는 CD 등을 갖는 경우 그리고 오직 그 경우에 게임이라는 식으로 정의될 수는 없다. 이러한 선언적 정의는 Wittgenstein이 말하였듯이 단순한 말장난에 지나지 않을 뿐만 아니라(이런 식의 선언문을 실제로 진지하게 제시하는 사람은 아무도 없을 것이다), Wittgenstein이 드러내고자 했던 가족 유사성 개념의 기본적인 특징, 즉 게임 개념이 열린-구조open-textured를 지니고 있다는 기본적인 아이디어를 놓치고 있다. 모든 게임들이 갖는 속성들을 남김없이 포괄하는 선언적 정의가 실제로 규정된다고 해도, 이러한 정의는 과거에 실현되지 않았던 가능성들과 미래에 실현될 가능성들을 고려하지 못할 수밖에 없다. 한 언어 공동체linguistic community가 어떤 새로운 사례를 마주할 경우, 즉 과거에 적용되었던 게임 개념에 포함될 자격을 갖추지 못한 전적으로 새로운 속성들 집합에 직면할 경우, 그것이 아무리 생소하더라도 기존의 게임 개념에 부합하는 것으로서 공동체에 받아들여진다면, 기존의 게임 개념은 그 새로운 사례 역시 포함되게끔 확장될 것이다. 이렇듯 게임과 같은 가족 유사성 개념은 시간이 흐름에 따라 올바르게 적용되는 영역을 넓혀나가는 열린-구조를 지니고 있다. 이런 식의 열린-구조 모형은 시간에 따른 개념적 변화 및 언어표현의 역동성을 수용한다.

가족 유사성 개념을 일단 파악하고 나면 우리는 이를 다양한 분야에서 발견해낼 수 있다. 특히 좋음[]goodness, 지식, 예술 등의 개념과 같이 철학적으로 중요하고 민감한 영역들에 이 개념을 적용해볼 수 있다.

언어 개념으로 다시 돌아와 보자. 전술했듯이 언어는 그 자체로 하나의 가족 유사성 개념을 표현한다. 우리가 언어라는 단어로써 특징짓는 모든 잡다한 활동들에 공통되는 단 하나의 것이란 없다. 언어의 본질을 원리적으로 명료하게 특성화할 수 있다는 생각 대신, Wittgenstein은 도시와 언어 간의 유사점을 생각해볼 것을 제안한다. 그가 말하는 도시는 밀턴 킨즈1)나 브라질리아 같은 계획도시가 아니라, 런던이나 뭄바이 같이 오랜 역사에 걸쳐 유기적으로 발전해온 도시이다. 전자에 속하는 도시는 실제 건설되고 사용되기에 앞서 단일한 사람에 의해 계획되고는 한다. 언어에서 이에 상응하는 것으로는 화학적 표기법이나 논리계산을 위해 고안된 표기법 등을 들 수 있다. 반면 후자에 속하는 도시는 우리 일상에서 쓰이는 자연언어natural language가 그러했듯이 극히 오랜 시간에 걸쳐 매우 서서히 발전해왔다(또한 단일한 사람이 그에 관한 모든 것을 반드시 알아야한 하는 것도 아니다). 이러한 역사적인 도시들이 발전해온 과정을 모종의 단일하고 전반적인 논리를 통해 추적할 수는 없다. 가령 런던은 2천년이라는 매우 긴 기간에 걸쳐 급격히 확장되기도 하고 발전상의 정체기를 겪기도 하며 때로는 위축되기도 하면서 뒤죽박죽으로 발전해왔다. 한국어나 영어 같은 자연언어 역시 이와 마찬가지이다. 역사적 도시와 자연언어 양자는 유기적이고 자연적인 진화의 산물로서, 그로부터 어떤 전반적이고 포괄적인 틀을 추출해낼 수는 없다. 하지만 그럼에도 불구하고 우리는 그러한 역사적 도시들에서 잘 생활할 수 있으며, 도시 내의 한 지점에서 다른 지점으로 무리 없이 이동할 수 있다. 가령 역사적 도시라고 해서 모든 도로가 죄다 막다른 길은 아니지 않는가. 자연언어 역시 이와 마찬가지로서, 주지하다시피 영어의 문법이나 어휘 규칙이 아무리 뒤죽박죽이고 엉망이더라도, 영어가 전적으로 불규칙적이고 무질서하게만 사용되는 것은 아니다.2) 뿐만 아니라 Haussmann의 디자인에 따라 파리 중심지가 재개발되었던 사례에서 볼 수 있듯이, 중심가든 교외지역이든 역사적인 도시의 일부 지역이 의도적인 설계 하에 개발되는 경우도 있다. 마찬가지로 자연언어의 일부도 계획적이고 엄밀한 방식으로 개조될 수 있는바, 가령 일상에서 느슨하게 쓰이던 특정 용어들이 과학적학문적인 전문용어로 엄밀하게 정의되어 사용되는 경우가 뚜렷한 사례일 것이다.


1) 1967년 버킹엄셔 북부에 건설된 영국 최대의 도시.

2) 그리고 뒤에서 살펴보겠지만 전적으로 규칙적이게 사용되는 언어란 언어조차도 아니며, 언어의 불규칙적인 사용이란 사용조차도 아니다.


따라서 우리는 다양한 언어게임들이 다양한 방식으로 상호 얽혀 있는 것을 볼 수 있다. 한 아이가 언어를 습득하는 과정은 그렇게 복잡다단하게 얽힌 각각의 언어게임들을 차츰 숙달해 가는 과정으로서, 어떤 언어게임이 다른 것보다 먼저 익혀지기도 하고, 어떤 언어게임의 숙달은 다른 것의 숙달을 상정하기도 하며, 일부 언어게임의 경우 여타의 것들과 동시에 익혀지기도 한다. 이 모든 것은 단편적으로 차츰차츰 진행된다. 눈치 빠른 독자는 예상했겠지만, 이러한 고찰에 따라 Wittgenstein은 사실을 기술하는 언어, 무언가를 주장하는 언어, 진리-조건과 논리적 함축을 갖는 언어 등과 같은 소위 인지적 언어cognitive language7에서 살펴본 개념인 화행speech-act보다 선행한다고 생각하지 않는다. 언어에는 주장이나 진술 외에도 인사하기, 모욕하기, 명령하기, 내기를 걸기, 흥정하기, 도덕적 칭찬과 비난, 위협하기, 감탄사, 욕설, 압박두려움혐오고통기쁨 등을 느낄 때 내뱉는 말 등 매우 많은 것들이 존재한다. 각각의 언어표현들은 이러한 다양한 목적들을 위해 존재하며, 그 중 어떤 것들은 지시하기referring의 기능을 일절 수행하지 않는다. ‘안녕!’, ‘Come on!’, ‘지금 비 와등의 말을 생각해보라. 다시 한 번 강조하지만 모든 언어표현 내지 언어행위들이 무언가 공통점을 갖고 있다고 말하기는 매우 어렵다. 모든 언어게임들이 이론적으로 중요한 공통점을 갖는다고 선험적으로 가정할 수는 없다.

Wittgenstein이 언어게임의 다양성을 강조한 것을 감안하건대 그가 다음과 같이 말한 것은 그다지 놀라운 일이 아니다: “전부는 아니더라도 우리가 의미라는 단어를 사용하는 대부분의 경우, 그 단어는 이렇게 정의될 수 있다: 단어의 의미는 언어에서의 그 사용use in language이다.” (1953, 43) 모든 단어가 의미를 표현한다는 식으로 말함으로써 언어에 그럴듯한 획일성을 부여하기보다는, 단어들이 다양하게 사용use되는 방식에 관해 묻는 것이 더 유익하고 오해의 소지가 덜하다는 것이다.

Wittgenstein이 들었던 또 다른 유명한 유비는 언어를 도구상자에, 단어를 도구에 비유하는 것이다. 모든 도구의 공통점이 무엇인지 말하려 해본다면, 우리는 모든 도구는 무언가를 고치는 데에 사용되는 물건이다’(또는 심지어 도구는 사용되는 물건이다’)와 같은 식으로 전혀 정보적이지 않거나 대체로 거짓인 답만을 제시할 수밖에 없다. 망치, , 줄자, 바이스, 드라이버, 접착제, 페인트 붓, 펜치, 수평자, 소켓 렌치, 납땜용 인두, 사포 등을 생각해보라. 대체로 철학자들은 사실을 진술하는 것 내지 사실적 정보를 전달하는 것(그리고 이와 연관된 언어행위로서 정보를 요구하는 것으로 받아들여지는 질문을 하는 것)만이 언어의 유일한 목적인 것처럼 말하고는 한다. 이는 마치 도구상자를 뒤적거리면서 망치와 못만 찾느라 혈안이 된 사람의 행태와 다를 바가 없다. 물론 이러한 유비에는 분명 제한사항이 있으며, 어찌 보면 선결되어야 할 문제 자체에 호소하는 것처럼 여겨질 수도 있다. 언어와 도구 사이의 가장 뚜렷한 차이점은, 언어가 기여하는 모든 목적이 언어와 독립적으로 설명될 수 있다고 가정해서는 안 된다는 것이다. 도구의 경우엔 한 도구를 사용하여 의도했던 결과가 달성되었을 때, (도구를 사용하여 다른 도구를 고치는 경우가 아닌 바에야) 굳이 그 도구를 지시하지 않고도 결과가 달성된 과정을 설명해낼 수 있다. 반면 언어의 경우 어떤 식으로든 무언가를 사고하는 우리의 능력은 언어를 숙달함으로써만 가능한 것처럼 여겨진다. 따라서 우리가 언어를 사용하여 특정 사고내용을 기록하거나 전달할 때 의도되는 결과(대체로 특정 사람으로 하여금 특정 믿음을 갖도록 하는 것)는 언어 숙달과 무관하게 설명될 수 없다. 대부분의 언어적 활동들은 Wittgenstein이 말한 삶의 형식들과, 즉 우리 삶의 특정 부분들을 구성하는 규약적인 관행들이자 특정 목적이나 가치가 존재할 수 있는 기반이 되는 다양한 삶의 형식들과 불가분의 관계에 있다.

 

 

규칙 따르기

 

이제 우리는 Wittgenstein이 제기했던 또 다른 유명한 주제를 살펴보고자 한다. 특히 1982KripkeWittgenstein의 규칙과 사적 언어Wittgenstein on Rules and Private Language가 출간된 이래 이 주제는 수많은 해설과 토론의 대상이 되었다. 이와 연관된 논의는 상당히 복잡하게 진행되었지만, Wittgenstein이 제기했던 핵심 논점 자체는 꽤 단순하다.

여기서는 아주 기초적인 예시가 취해지겠지만, 이것만으로도 일반적인 요점을 간추리는 데에는 충분할 것이다. 당신이 공항 복도를 따라 걷다가 ‘Y’자 형태의 갈림길에서 다음 기호가 적힌 표지판을 마주쳤다고 해보자:

 

(null)

 

당연히 당신은 아무런 주저함도 없이 자연스럽게 오른쪽으로 발걸음을 향할 것이다. 이 기호는 오른쪽으로 가야 한다는 것을 의미한다. 이 기호가 말해주는 바가 바로 그것이기 때문이다. 하지만 당신은 이를 어떻게 알게 되었는가? 이와 관련하여 당신이 알고 있는 사실이란 무엇인가? 대체 어떤 요인으로 인해 당신은 이 기호의 그러한 의미를 이해하게 된 것인가?

이 물음에 대한 답변으로 모종의 정신적인 사건을 제시해볼 수 있다. 즉 우리가 위 기호를 이해할 때 어떤 정신적인 (이미지)mental image이 나타난다는 것이다. 하지만 그러한 정신적인 대상 내지 상태라는 것은 이 기호에 적절히 대응하는 데에 필요하지도 않고 충분하지도 않다. 우선 심적 이미지는 기호를 이해하기 위한 필요조건이 아니다. 특정한 정신적 이미지가 수반되지 않더라도 이 기호에 적절히 대응하는 경우를 충분히 상상해볼 수 있기 때문이다. 당신은 그저 화살표 기호를 보고는 오른쪽으로 갔을 뿐, 이러한 사건을 설명해줄 또 다른 정신적 처리 과정이 추가로 상정되어야 할 필요는 없다.

다음 더욱 중요한 사안으로서, 심적 이미지는 기호를 이해하는 데 대한 충분조건도 아니다. 즉 정신적 사건을 들먹인다고 해도 무언가를 이해하는 절차가 충분히 설명되지는 않는다. 당신이 화살표 기호를 이해하는 시점에 당신의 내적인 정신적 스크린에 무언가가 번뜩 떠올랐다고 해보자. 이 경우 당신의 정신에 떠오른 그 이미지는 표지판에 그려진 화살표 기호와 정확히 동일한 상황에 처해 있다. 다시 말해, 표지판의 기호가 읽히고 적절히 해석되어야 하는 것과 마찬가지로, 당신의 정신에 떠오른 이미지 역시 읽히고 적절히 해석되어야 하는 것이다. 결국 그 심적 이미지의 의미는 또 어떻게 이해될 수 있느냐 하는 앞서와 동일한 물음이 다시 제기될 뿐이다. 단순하게 생각해보라. 외적인 물리적 기호와 그에 대한 내적인 정신적 복제물이 있어서, 전자를 이해하는 데 대해 후자만으로도 충분하다는 점을 일단 받아들여 보자. 그 경우, 그렇다면 내적인 기호 자체에 대한 해석은 또 어떻게 가능한가 하는 물음이 다시 제기되는 것이다. 화살표가 그것이 그려진 방향과 반대 방향을 가리키는 것으로 받아들여지는 사회에서라면 위 기호는 왼쪽으로 가라는 것을 의미할 것이며, 이는 충분히 상상가능하다. 심지어 어떤 사회에서는 위 기호가 진입금지라든가 휴대전화 사용금지를 의미하는 것으로 받아들여질 수도 있다. 이 모든 것들은 [한 공동체 내에서 통용되는] 규약convention이다. [그리고 심적 이미지는 이렇듯 기호가 규약에 따라 의미하는 바를 가려내는 데에 충분하지 않다.] 따라서 정신적 스크린에 떠오르는 또 다른 심적 기호는 외부 기호를 이해하는 데에 아무런 효력도 발휘하지 않는다. 이에서 더 나아가 우리 마음속에서 일어나는 그 무엇이 되었든, 설사 그것이 여기서 살펴본 형태의 내적 복제물보다 훨씬 더 복잡하고 상세하더라도, 사정은 마찬가지이다. 그 어떤 내적 현상도 이해를 구성(構成)constitute하지 못한다. Wittgenstein의 말을 빌자면 그 어떤 심적과정mentalprocess도 의미를 가져다줄 수는 없다.”(1953, :218)

[그렇다면 대체 무엇이 우리의 이해를 구성하는가?] 이에 대해 Wittgenstein이 전적으로 회의적인 입장을 취하는 것은 아니다. 그는 우리가 아무것도 이해할 수 없다거나, 혹은 의미와 이해에 관한 생각들이 모조리 헛된 것이라 주장하지는 않는다. Wittgenstein이 말하고자 하는 바는 기호에 대한 파악이 기호를 해석함으로써 이루어진다는 식으로 접근해서는 안 된다는 것이다. ‘해석이라는 것이 한 기호를 통해 다른 기호를 이해하는 것으로 간주되는 한, 바로 앞 단락에서 살펴본 문제가 곧바로 대두된다. 그렇다기보다 기호를 따르거나 준수하는 것은 특정 성향(性向)disposition을 습득하는 훈련(교육)training의 문제이다. 공항 사례의 경우 우리가 그 기호를 이해하고 적절히 대응할 수 있는 이유는, 그러한 환경에서 그러한 기호를 마주쳤을 때는 오른쪽으로 간다는 성향을 훈련을 통해 습득했기 때문이다. 탐구의 초입에서 Wittgenstein은 이렇게 강조한다: “설명은 어딘가에서 멈춰야 한다.” 계속 파헤쳐 내려가다 암반(巖盤)에 도달하면 삽의 방향을 돌려야 한다.” 우리가 평소에 하던 대로 반응할 수 있는 이유는 단지 우리가 그렇게 하는 경향이 있기inclined to do때문이다. 그게 바로 우리가 행하는 방식일 뿐이다. 그리고 이런 식으로 무언가를 배우는 능력은 우리의 자연사(自然史)natural history의 특징이다. 우리와 매우 다른 성향을 지닌 다른 종의 생물체들은 우리가 훈련하는 방식으로 반응할 수조차 없을 것이다. Wittgenstein이 말하듯이 의사소통의 가능성은 판단에서의 일치agreement in judgement에 달려 있다(1953, 242). 거꾸로 말해 만약 우리 각자가 어떤 사건들에 대해 유사한 방식으로 반응하는 성향을 지니고 있지 않다면, 의사소통은 애초에 시작될 수조차 없을 것이다. 종종 이러한 논점은 Platon의 대화편 Menon에 나오는 노예 소년 이야기와 비교되고는 한다. 그 대화편에서 Socrates는 한 노예 소년에게 일련의 질문들만 던지면서 그 소년으로 하여금 한 기하학 정리를 증명하도록 유도함으로써, 기하학적 지식이 선천적innate임을 실증해보이고자 한다. Wittgenstein은 우리가 기하학에 관한 지식을 선천적으로 타고난다는 데에는 동의하지 않겠지만, (기하학을 배우기 위한) 특정 성향을 이미 지니고 있어야 한다는 것을 Socrates가 보여주었다는 점에는 동의할 것이다.

 

 

규칙 따르기

 

그렇다면 기호에 반응하는 성향을 갖는 것은 기호에 대한 이해를 구성하는가? 그렇지 않다. 왜냐하면 최소한 기호에 올바르게in the right way 반응하는 성향을 가져야 하기 때문이다. 즉 언어적 공동체의 다른 구성원들이 하는 방식대로 기호에 반응하는 성향을 지녀야만 한다. 이는 앞서 언급한 Kripke의 책 Wittgenstein의 규칙과 사적 언어의 핵심이 되는 주제로서, Wittgenstein을 공부하는 사람이라면 이미 이를 눈치챘을 것이다. 이 주제는 두 부분으로 나뉠 수 있다.

첫 번째로, 주어진 규칙이 사용되는 범위가 실제적으로도 잠재적으로도 무한하다는 점을 감안했을 때, 한 사람은 분명 상당히 많은 경우에 걸쳐 그 규칙에 노출되었을 것이다. 그 사람은 자신이 올바른 성향을 갖고 있는지 여부에 대해 결코 확신할 수 없다. 그 사람이 지닌 성향과 그 규칙이 실제로 지시하는 바가, 그 사람이 이제껏 마주쳐온 경우들에서는 부합했을지라도 그 사람이 아직 마주쳐본 적 없는 경우들에서는 부합하지 않을 수 있기 때문이다. 따라서 성향을 갖는 것과 규칙을 준수하는 것 간에는 다음과 같은 회의적인 문제 혹은 인식론적인 틈epistemic gap이 있는 셈이다: 우리는 자신이 규칙을 올바르게 준수하고 있다는 것을 도대체 어떻게 알 수 있는가?

두 번째로, 다른 사람들이 하는 방식대로 기호에 반응하는 성향을 갖는 것과 기호를 이해하는 것 간에는 규범적인 normative gap이 존재한다. 실제로 올바른 방식으로 하는 성향을 내가 갖고 있다 하더라도 [즉 내가 나의 성향에 따라 행했던 바가 실제로도 올바른 것이었더라도], 나의 그 행동이 그 자체로 올바른 것이 되지는 않는다. 왜냐하면 성향 그 자체는 아무런 규범적 효력도 지니고 있지 않기 때문이다. 한 생물체가 특정 성향을 갖고 행동한다는 사실이, [그 성향이 산출하는 행동이 실제로 올바른 것이라 해도,] 그 생물체가 올바른 방식으로 행동하고 있음을 의미하지는 않는다. 이뿐만 아니라 성향을 갖는 것은 내가 무엇을 해야ought 하는지를 말해줄 수도 없다. 이는 등을 긁어줄 때 개가 다리를 위아래로 움직인다고 해서, 반드시 해야만 하는 올바른 행위를 그 개가 하고 있는 것은 아닌 것과 마찬가지이다. 하지만 이것이 규칙과 의미에 관해 말할 수 있는 요점의 전부이다. 우선 한편으로, 나는 체계적으로 실수할systematically make mistake 수 있다. 이는 나의 수행이 규칙에 비추었을 때 체계적으로 올바르지 않은 경우로서, 성향을 갖는 것이 규칙 준수와 연관된 전부라면 이러한 일은 원천적으로 불가능하다(당연하고 사소한 말이겠지만, 사람은 자신이 행동하는 대로 행동하는 성향이 있기 때문이다). 규칙을 파악하고 있는 상태에서 실수를 하는 것과, 그와 정확히 동일한 행동인데도 아무런 규칙에 따르지 않은 채로 행동하는 것 간에는 분명 차이가 있다. 또 다른 한편으로 내 행동이 규칙과 정확히 일치하더라도, 그 행위는 규칙에 의해 인도된guided 게 아니라 단지 규칙에 부합하게끔match the rule 행해진 것일 수 있다(와플 굽는 틀이 아무리 완벽한 모양으로 와플을 구워낸다고 해도 그것은 규칙을 따르고 있는 게 아니다).

이러한 문제에 대해 Wittgenstein이 어떤 입장을 취하는지 혹은 취했을지와 관련하여 매우 많은 논란이 있어왔다. KripkeWittgenstein을 대신하여 제시한 답이 상당히 많은 지지자를 얻긴 했지만, 그와 다른 답변들도 많이 제안되었다. 여기서 나는 두 번째 난점에 대해 개인적으로 생각하기에 Wittgenstein이 다소 동의했을 법한 해결책을 간략히 제시해보고자 한다. 이 문제를 해결하기 위한 핵심은 언어적 행동이 발생하는 사회적 맥락social context이다. 우리가 행하는 다양한 언어게임들 중에는 타인의 잘못된 언어적 성향을 올바르게 교정하려는 습관이 있다. 바로 이것이 규범성normativity을 설명해주는 우리 관행의 기본적인 요소로서, 그 이상을 기대할 수는 없다. 우리는 다른 사람의 잘못되거나 탈선적인 언어행위를 꾸짖거나 지적하고는 한다(심지어 처벌하는 경우도 있다). 그런 식의 제재를 가하는 공동체가 없는 한 어떤 발화를 옳거나 그르다고 규정하는 것은, 언어-사용자가 보기에는 외견상 그런 것처럼 보인다 하더라도 실상은 아무런 의미가 없을 것이다. 이는 주어진 기호의 해석과 관련하여 현실적으로나 잠재적으로 제기될 수 있는 모든 문제를 한 개인이 해결할 수 없는 것과 마찬가지로, 하나의 언어 공동체 전체 역시 그렇게 할 수 없다는 것을 의미하지는 않는다. 언어 공동체는 그러한 문제를 상당량 처리할 수 있지만, 그 전부를 남김없이 깔끔하게 해결할 수는 없다. 그런데 애초의 문제는 공동체의 관행 내에 규범성을 허용하는 것뿐이었다. 내가 지금 제시한 해결책이 하고 있는 바가 바로 이것이다. [즉 규칙 준수의 올바름을 규정하는 규범성의 원천은 특정 관행이 통용되는 언어 공동체 자체일 뿐이다?]

내가 생각하기에 Wittgenstein은 첫 번째 문제에 대해서는 그다지 개의치 않았을 듯하다. 이에 대해서는 소위 귀납(歸納)induction의 문제, 즉 관찰된 사례에 근거하여 아직 관찰되지 않은 사례들을 추론하는 것이 어떻게 정당화되느냐 하는 문제에 대해 그가 취했던 입장이 그대로 적용될 수 있을 것이다. Wittgenstein그 문제에 대해 해결책을 제시하는 것이 아니라 그저 침묵할 것을 권하는바, 그 문제와 관련하여서는 단순히 삶의 형식을 묵묵히 따를 수밖에 없다는 것이다. 다만 이를 상세히 논의하자면 이야기가 너무 길어질 것이다.

 

 

사적 언어

 

탐구243절에서 Wittgenstein은 단어들이 적 경험inner experience에 적용되는 언어, 오로지 말하는(혹은 글로 쓰는) 사람 자신에게 알려질 수 있는 그런 언어에 대해 고찰한다. 필연적으로 화자 자신에게만 이해되는 이러한 사적(私的) 언어private language의 이해 가능성에 대해 Wittgenstein은 의구심을 던진다. 다만 이는, 단어들이 원리적으로 타인에게 이해될 수 있는 한, 우리가 자신의 내적 경험에 관해 말하기 위해 단어들을 사용한다는 명백한 사실을 부인하는 것은 아니다(이에 관해서는 잠시 뒤에 살펴볼 것이다). 그는 필연적으로 사적일 수밖에 없는 언어라는 생각을 반대하고자 하는 것이다.

당신이 그러한 사적 언어를 고안하려 한다고 가정해보자. 당신이 느낀 한 감각(感覺)sensation을 당신은 ‘S’라고 부르기로 한다. 당신은 ‘S’를 노트에 적어 놓고는 그것이 S의 이름이 된다고 생각한다. 시간이 흘러 당신은 또 어떤 감각을 느끼게 된다. 그것은 S인가? 단순하게 생각해보자면 마음속으로inwardly 그 감각을 이전의 경우와 비교해볼 수 있다고 답할 것이다. 하지만 문제는 그러한 내적 비교inner comparison라는 게 전연 무의미하다는 것이다. 이 질문에 답할 수 있는 기준(基準)criterion 자체가 없다. 왜냐하면 어떤 내적 감각이 존재하는 것existence과 그것이 단지 존재하는 것처럼 여겨지는 것seeming to exist간에는 아무런 차이가 없기 때문이다. 우리는 다음과 같이 말할 수 없다: 그 감각이 S인 것처럼 여겨진다면 그것은 S이고, 그렇지 않다면 S가 아니다. 한 단어가 유의미해지기 위해서는 그 단어가 실수로 오용(誤用)되는mistakenly misapply 경우가 상상가능해야 한다. 즉 실제로는 S가 아닌데도 S인 것처럼 여겨지는 경우가 상상될 수 있어야 한다. 그렇지 않은 경우 ‘S’라는 단어로 무언가를 의미하고자 하는 시도는 실패한다.

Wittgenstein은 이렇게 말한다:

 

첫 번째 경우[(최초에 한 감각을 ‘S’라고 칭했던 경우)] 나에게는 올바름correctness에 대한 기준이 없다. 혹자는 이렇게 말할지 모르겠다: 내가 생각하기에 올바르다고 여겨지는 것이라면 무엇이든 그것은 올바르다. 이는 우리가 여기에서 올바름에 관해 더 이상 논할 수 없다는 것을 의미할 뿐이다.

(1953, 258)

 

그리고 이렇게 말하기도 한다:

 

규칙을 따르는 것은 관습이다. 그리고 규칙을 따른다고 생각하는 것과 규칙을 따르는 것은 다르다. 따라서 규칙을 사적으로따른다는 것은 가능하지 않다. 그렇지 않다면 규칙을 따른다고 생각하는 것은 규칙을 따르는 것과 같아질 것이다.

(1953, 202)

 

우리는 다음과 같이 물을 수 있다: 감각의 동일성 기준은 무엇인가? 감각 S-확인하는re-identifying 기준은 무엇인가? 또는 다음과 같이 물을 수 있다: 어떤 감각이 S가 되는 기준은 무엇인가? 이러한 질문이 단 하나라도 답해질 수 없는 한 ‘Sbeing S에 대해서는 아무런 표준도 없으며, 따라서 [그에 대한 명칭으로 의도된 ‘S’ 역시] 아무런 의미가 없는 셈이다. 사적 언어 개념에 대한 Wittgenstein의 반대를 간결한 논증 형태로 구성해보자면 다음과 같다:

 

1 한 단어로 무언가를 의미하기 위해서는 그 단어가 올바르게 적용되는 경우와 올바르게 적용되는 것처럼 여겨지는 경우가 구분될 수 있어야 한다.

2 사적 단어는 필연적으로 그러한 구분을 결여할 수밖에 없다.

 

따라서 사적 단어란 존재하지 않는다.

 

탐구가 첫 출간되었을 당시, 많은 사람들에게는 사적 언어의 불가능성 논제가 감각이 필연적으로 공적(公的)public이라는 논제를 함축하는 것으로 받아들여졌다. 혹은 Wittgenstein이 모종의 행동주의(行動主義)behaviourism를 주장하는 것처럼 받아들여지기도 했다. 하지만 이는 상당히 복잡미묘한 문제이다. 예를 들어 고통-단어pain-word를 배우는 것은 실제로 약간의 행동을 배우는 것이긴 하지만, 다음과 같이 고통 및 그에 수반되는 것과 관련되어 습득된다:

 

단어들은 감각을 원초적(原初的)primitive이고 자연스러운 방식으로 표현하는 것과 연관된다. 아이는 아픔을 느끼면 일단 운다. 어른들은 아플 때 내뱉는 단어들과 문장들을 그 아이에게 말해주며 가르친다. 이제 아이는 이전과는 다른 새로운 고통-행동pain-behaviour을 배우게 되는 것이다.

(1953, 244)

 

다시 한번 강조하지만 단어들은 특정한 맥락 내에서 사용되는 경우에만 의미를 갖는다. 단어들은 풍부한 삶의 틀 내에서 상호 얽혀있으며 그렇게 얽힌 전체 체계 내부에서만 각자의 역할을 수행한다. 앞서 말한바 우리는 언어를 가능하게 만들어주는 성향을 [언어를 실제로 습득하기에] 앞서 이미 지니고 있다. 위 인용문에서 Wittgenstein이 기술하듯이 그러한 성향 중에는 자연스런 표현방식이라는 것이 존재한다. 고통과 같은 내적 감각을 나타내기 위한 단어들은 그러한 자연스런 표현 방식 위에 올라타 있는ride atop 셈이다. Wittgenstein은 내적 상태inner state의 존재 자체를 부정한다는 의미에서의 행동주의자는 아니다. 단지 그는 내적 상태를 나타내기 위해 사용되는 단어들에 대한 기준이 인간 존재가 갖는 자연스런 행동방식과 결부되어야한 한다는 것을 주장할 뿐이다.

이러한 사적 언어 반대 논변을 우리가 앞서 살펴보았던 신조와 연관지어볼 수 있다. 2에서 살펴본 형태의 Russell의 언어이론은 감각-자료sense-data 개념 및 그와 밀접히 연관된 직접대면의 원리를 기초로 구성되었다. 그 원리에 따르면 우리가 이해하는 모든 명제는 우리가 직접대면하는 요소들로만 구성된다.” 여기서 우리가 궁극적으로 직접대면하는 것이란 바로 감각자료로서, 이를 수단삼아 우리는 물리적 대상을 지시할 수 있는 것이다. 이런 점에서 감각-자료는 가장 탁월한 사적 실체private entities par excellence. 하지만 분명 Wittgenstein은 이러한 신조뿐만 아니라, 감각-자료의 좀 더 현대적 버전으로서 심리철학에서 논의되는 감각질(感覺質)qualia3) 개념 역시 극도로 의심스럽게 생각할 것이다.


3) 우선 첫 번째로 심적 현상들mental phenomena 중에서 고통스러움, 가려움, 간지러움, 잔상을 느낌, 둥근 녹색 조각을 봄, 암모니아 냄새를 느낌, 메스꺼움 등과 같이, 감각 내지 감각적 특질sensory quality과 연관된 것들을 구분해볼 수 있다. 이러한 심적 상태들은 현상적내지 질적(質的)qualitative특성을 갖는다고, 즉 그것들이 느껴지거나feel 보이거나look 나타나는appear 고유한 방식을 갖는다고 말해진다. 종종 선호되는 용어를 사용해 말해보자면, 그러한 현상을 경험하거나 그러한 상태에 처할 경우 여차하게 느껴지는 어떤 느낌something it is like이 존재한다. 그래서 고통은 고통에만 고유한 질적 느낌 즉 아프다는 느낌을 갖는다. 마찬가지로 가려움은 가렵게 느껴지고 간지러움은 간지럽게 느껴진다. 당신이 녹색 조각을 볼 때 그 조각은 독특한 방식으로 보인다. 그것은 녹색으로 보이며 당신의 시각 경험에는 그러한 녹색의 느낌이 동반된다. 이러한 갖가지 감각들은 나름대로의 독특한 느낌을 지니고 있으며, 최소한 각각이 속하는 일반적인 유형들(가령 고통, 가려움, 녹색을 봄 등)마다 우리가 직접적으로 식별할 수 있을 것이라 여겨지는 질적 특질에 따라 특성화된다. 이러한 항목들은 현상적 상태질적 상태내지는 간혹 날 것 그대로의 느낌raw feel등으로 칭해진다. 작금에는 이러한 감각적질적 상태 내지 그러한 상태에서 경험되는 감각적 특질을 가리키기 위해 감각질이라는 용어가 표준적으로 널리 쓰인다. 이러한 부류의 심적 현상들은 총괄적으로 현상적 의식phenomenal consciousness을 구성한다고 말해진다.” (김재권, 심리철학Philosophy of Mind, 3, Routledge, 2011, 15.)

     

 

역사적 사항

 

(前期 Wittgenstein履歷에 관해서는 4, ‘역사적 사항참조) 1929년 철학계에 돌아온 뒤 얼마 지나지 않아 캠브릿지로 복귀한 Wittgenstein은 곧 급진적이고 새로운 사고방식을 발전시키기 시작하였다. 이는 1930년대에 비공식적인 필사본 형태로 유포되었던 청색책The Blue Book갈색책The Brown Book에 개진되었으며, 최종적으로는 그의 死後 2년 뒤인 1953년에 출간된 철학적 탐구로 집결된다. Wittgenstein의 철학을 다루는 책들은 엄청나게 많다. 그 중 가장 예리한 것들은 Wittgenstein의 저술들을 거의 신성시하는 그의 추종자들의 관점에서 쓰인 경우가 많다. 이는 그의 추종자들을 폄하하고자 함이 아니라 그의 後記 저술들이 상당히 독특하고 매력적이라는 점을 언급하기 위함이다. 심지어 Wittgenstein의 사고노선을 명시적으로 따르지는 않았더라도, Crispin Wright부터 John McDowellStanley Cavell에 이르기까지 상당히 넓은 범위에 걸친 철학자들의 저술이 Wittgenstein으로부터 막대한 영향을 받았다고 할 수 있다.

 

 

이번 의 요약

 

後期 Wittgenstein은 언어에 대해 FregeRussell의 견해 및 Wittgenstein 자신이 이전에 견지하였던 바와도 상당히 다른 사고방식을 제안한다. 이에 따르면 언어란 다양한 언어게임들로 이뤄진 조직fabric과도 같은 것으로 생각될 수 있으며, 그 조직 전체를 통괄하는 단일한 실이 존재한다고 상정할 수는 없다. 언어란 가령 총각과도 같이 필요충분조건 형식으로 정의될 수 있는 명료한 개념이 아니라 가족 유사성 개념으로서, 다양한 언어게임들이 상호 교차하고 중첩되면서 유기적이고 변화무쌍한 방식으로 결합되어 있는 그 전체가 바로 언어이다. 언어에 존재하는 이러한 다양한 활동들을 지시라든가 의미와 같은 개념을 통해 추상화하여 이론화하고자 할 경우, 지시와 의미 개념은 언어게임에 대한 우리의 이해를 왜곡할 뿐이다. 예를 들어 Wittgenstein이 묘사한 심부름꾼과 점원 사례를 우리가 그저 살펴본다면”, 그러한 언어게임을 기술하는 데에 의미론적 개념들이 더이상 특권적priviledged이지 않다는 점을 알게 될 것이다. 의미론적 어휘들의 적용가능성에는 상당한 배경이 상정되어 있지만, 하나의 배경이 일단 정해지고 나면 그 어휘들은 언어를 이해하는 데에 당초에 생각했던 것만큼 크게 기여하지는 않게 된다.

언어에 대한 사고방식에 만연해 있는 가장 심각한 환상은, 의미라는 것이 기이하고 불가사의한 유령처럼 단어에 깃들어 있어서, 우리가 단어에 어떻게 반응해야 할지를 알려준다는 생각이다. 단어들이 실제로 사용되는 많은 경우들을 유심히 살펴보면 그런 식으로 생각된 의미란 언어를 이해하는 데에 일절 무력하다는 점이 드러난다. 가령 단어의 의미란 단어가 사용되거나 이해될 때 파악되는 모종의 정신적인 실체라고 해보자. 그 경우, 본래의 단어 자체가 해석되어야 하는 것과 마찬가지로, 단어의 의미라고 여겨진 그 정신적 실체 역시 어떻게든 해석되어야 한다. 따라서 우리는 단어의 의미를 찾는 시도를 포기하고, 언어를 이해하는 일이란 의미에 대한 해석 없이 언어를 파악하는 문제라는 점을 받아들이는 수밖에 없다. 즉 신발끈 묶는 법을 배울 때처럼, 우리는 가장 낮은 단계에서 [즉 그 이상 이론적으로 더 파고들어갈 수 없는 원초적인 수준에서] 다른 사람들이 하는 방식대로 반응하도록 훈련될 뿐이다. 다만 이러한 훈련은 조련된 앵무새가 보여주는 기계적인 반응처럼 단순한 행동으로만 귀결되는 것이 아니라, 최종적으로 언어에 대한 이해를 산출한다. 왜냐하면 훈련 과정에서 우리는 언어 공동체에 통용되는 관습적인 배경에 알맞게 반응하는 법을 배우는바, 그 배경은 규범 즉 올바름에 대한 기준을 구성하며 언어 사용자는 이 규범에 부합하려고 노력하기 때문이다.

Russell을 비롯하여 많은 철학자들은 물질적 대상에 관해 말할 수 있는 우리의 능력이 감각자료에 대한 직접대면에 의존한다고 주장해왔다. Wittgenstein은 감각-자료와 같이 내적이고 사적인 것에 적용되는 언어란 불가능하다고 논증한다. 단어가 유의미하게 사용될 수 있으려면 그 단어가 올바르게 적용되는 경우와 단지 올바르게 적용되는 것처럼 여겨지는 경우가 구분될 수 있어야 하기 때문이다. 감각-자료와 같은 내적 현상에 적용되는 사적 단어는 이러한 구분을 결여한다.

 

 

탐구문제

 

1 이번 장의 초입을 되돌아보건대 우리가 가져봄직한 의문은, 구성성 원리가 제시하는 요구사항에 Wittgenstein이 어떻게 대응할 수 있겠느냐는 것이다. 왜냐하면 그는 의미를 등한시함으로써, 구성성 원리가 부과하는 조건을 충족해야 할 의미론적 개념 자체를 폐기해버린 셈이기 때문이다. Wittgenstein의 입장에서 이에 어떻게 대응할 수 있겠는가?

 

2 공장 작업대에서 사용되는 언어, 연인들이나 절친한 친구 사이에서 사용되는 언어, 길거리, 술공집, 법정 등 다양한 상황에서 사용되는 언어 등을 생각해보라. 분명 그러한 언어들에서도 모종의 규칙들(짐작건대 대체로 Grice적인 준칙들)이 작용하고 있을 것이다. 하지만 그 규칙들이 맥락에 따라 변하는가? 변한다면 어떤 식으로 변하는가? 대화 참여자들은 그 규칙을 어떤 방식으로 이해하고 있는 것인가?

 

3 언어에 관한 Wittgenstein의 그림은 각기 다른 유형의 언어들 간 차이점만을 지나치게 강조할 뿐, 그 유사성을 간과하고 있지는 않은가?

 

4 타인의 마음에 관한 문제, 즉 다른 이에게도 마음이 존재한다는 것을 어떻게 알 수 있는가 하는 문제에 대해 John Stuart Mill은 다음과 같은 해결책을 제시한 바 있다: 내가 고통을 느낄 때 나는 울부짖는다. 다른 인간 존재자들 역시 나와 유사한 방식으로 행동한다. 따라서 다른 사람이 울부짖을 때, 유추에 의해 나는 그 사람이 고통을 느끼고 있다고 추론할 수 있다. Wittgenstein의 사적-언어 반대 논증을 감안하건대, Mill의 해결책을 어떻게 생각하는가?

 

 

주요 읽을거리

 

Wittgenstein, L. (2009), 철학적 탐구Philosophical Investigations, 4.

 

 

추가적인 읽을거리

 

Ahmed, A. (2010), Wittgenstein의 철학적 탐구Wittgenstein’s Philosophical Investigation.

Lugg, A. (2000), Wittgenstein의 탐구 1-133: 안내 및 해설Wittgenstein’s Investigation 1-133: A guide and interpretation


댓글(0) 먼댓글(0) 좋아요(0)
좋아요
북마크하기찜하기
 
 
 

10

Quine의 언어철학

 

Willard Van Orman Quine은 언어철학의 거물급 인사들 중 한 명이다. 그는 2차 세계대전 이후의 언어철학이 전개되었던 흐름 및 향방에 막대한 영향을 끼쳤으며, 대표적으로 8에서 일부 살펴보았듯이 명제적 태도에 관한 논쟁이 형성 및 촉발되는 데에 비중 있는 역할을 담당하기도 했다. 이러한 중요성에도 불구하고 이제껏 우리는 막상 Quine의 고유한 철학에 관해서는 그다지 많은 것들을 말하지 않은 편이었다. 그 이유는 언어에 대해 그가 실제로 제시한 이론이 이전에 주류를 이루던 관점과는 다소 동떨어진 것이었기 때문이다. 우선 한편으로 Quine을 유명하게 만든 착상들 대부분은 많은 사람들이 보기에 다소 불합리할 정도로 너무 비판적이거나 부정적인 것으로 여겨졌기에, 이전의 주류 흐름을 고수하던 철학자들은 자신들이 발전시킨 긍정적이고 건설적인 관점을 본질적으로 수정해야 할 만큼 Quine의 제안을 중요한 것으로 여기지는 않았다. 또 다른 한편으로는 Quine이 제시한 나름의 긍정적인 언어철학적 제안들이 그다지 폭넓게 고려되지는 못한 편이었다. 대부분의 Quine 연구가들이 입을 모아 지적하는 바에 따르면 이러한 실정은 Quine 철학의 다양한 측면들이 어떤 식으로 상호 보완하고 강화하는지를 면밀히 파악하지 못함에 따라 그의 입장이 갖는 진정한 힘을 인정하는 데에 걸림돌이 된다. 이번 에서 우리는 Quine 철학의 파괴적인 측면과 건설적인 측면 양자를 균형 있게 살펴보면서, 특히 이러한 측면들이 어떻게 서로를 지지하는지에 초점을 맞출 것이다.

 

 

Quine의 자연주의

 

FregeRussell 같은 인물들이 제시하는 언어철학은 다소 과장되고 거만한 것처럼 보인다. 전통적인 언어철학자들은 언어 자체에 대해, 그리고 언어와 세계 간의 관계 내지 언어와 마음 간의 관계에 대해 사실상 선험적인 가정들을 거리낌 없이 제시한다. 이전의 언어철학적 이론들은 가령 이나 이해understanding와 같은 언어철학의 핵심 개념들이 충분히 명료하다고 가정한 채, 그러한 개념들이 현실 세계에서 이뤄지는 발화와 같은 실제 언어적 현상에 무리 없이 명확하게 적용될 수 있다는 것을 무비판적으로 가정한다.

Quine이 언어철학에서 설정한 주된 기획은 이런 식의 가정을 몰아내는 것이다. 그 대신 우리는 자연과학natural science의 견지에서 언어 및 의미이론에 접근하고자 할 경우 일이 어떤 식으로 진행될지를 면밀히 조사해보아야 한다. 언어에 대한 엄밀한 과학rigorous science of language이란 과연 어떤 모습이어야 하겠는가? 의미라는 개념은 과연 과학적으로 존중될 만한scientifically respectable 개념이 될 수 있는가? FregeRussell은 의미, 명제를 파악한다는 것grasping, 지시, 진리-조건 등을 거침없이 들먹이면서, 그러한 개념들 및 그 개념들의 설명적 가치explanatory value가 오로지 철학적 반성(反省)philosophical reflection을 통해 선험적으로 상정되거나 수립될 수 있는 것처럼 여긴다. 하지만 그들은 그러한 개념들이 세계에 존재하는 실제 대상과 사건에 적용된다는 것을 우리가 어떻게 알아낼 수 있는지에 관해서는 그다지 많은 것을 말해주지 않는다. 단지 두 인물 모두 자신들의 형식논리 내지 기호논리formal or symbolic logic를 설계하는 과정에서 그러한 개념들이 명확하게 적용될 수 있다고 추정하거나 혹은 그러한 적용 가능성을 약정(約定)stipulate할 뿐이다. 하지만 분명 언어 및 언어의 유의미성이라는 것은 실제 세계에 존재하는 생물체가 구체적으로 소리를 만들어냄으로써 발생시키는 현상이다. 그렇다면 의미이론은 조음(調音)된 특정 소리나 표기된 특정 기호가 어떻게 유의미significance해지는지를 기술해야 하지 않겠는가? 자연언어natural language에 관한 이론은 특정 생물이 행동하는do 것에 관한 이론으로서, 관찰가능한 증거observable evidence를 토대로 입증될confirmed 수 있는 경험적 이론이어야 하지 않겠는가? 인간 두뇌에 있는 신경세포들의 축색돌기 및 시냅스와 같은 물질에서 발생하는 인지(認知)cognition현상을 설명하기 위해, 과연 의미라든가 명제와 같은 물질적인immaterial 것들을 굳이 가정해야할 필요가 있겠는가?

이러한 질문들을 제기한 뒤 Quine 스스로가 내놓는 답을 간단히 요약하자면, 의미에 대한 과학science of meaning으로서 단어에 특정 의미를 할당하는 식으로 작업하는 전통적인 부류의 그런 학문이란 기실 존재할 수 없다는 것이다. 물론 의미 개념은 실용적으로는practically 필수불가결하다. 즉 우리가 실생활을 영위해나가면서 언어를 통해 상호작용하는 데에는 의미 개념이 필요하다. 하지만 엄밀한 자연과학적 견지에서 보자면 그러한 의미라 할 만한 것은 존재하지 않는다. 우리는 그저 불가피하게 실용적(實用的)pragmatic이고 관심에-따른interest-relative 이유에서 직관적으로 언어에 의미를 귀속시키거나 할당하는 것일 뿐, 그 중 어느 것도 우리가 생각할 수 있는conceivable 객관적 증거에 의해 정당화될 수는 없다. 한 진술의 의미를 기술한다는 것은 항시 변하기 마련인 우리의 관심사에 따르는바 상당 부분 편의의 문제matter of convenience로서, 편의상의 목적을 넘어선 그 이상의 것을 의미에 대해 이론적으로 기도(企圖)할 수는 없다.

Quine은 자신이 발전시킨 철학적 관점 일반을 자연주의naturalism라 칭하였다. 그가 말하는 자연주의란 자연과학 전체의 총합sum of natural science으로서, 과학의 상위 혹은 배후(背後)above or behind에서 지식과 실재에 대한 준거를 정립하는 선험적 학문분과로서의 철학은 아니다. 이런 관점에서 보았을 때 철학이란 그것을 정의해주는 단일한 본질a defining essence을 지닌 체계라기보다는 그저 세계에 대한 여러 과학적 지식이 뒤범벅된 총체라고 할 수 있다. 다만 철학이 자연과학과 구분되어 나름의 적법한 자리를 차지할 수 있는 이유는, 대개 철학이 과학의 개별 분야들에 비해 더욱 일반성generality과 추상성abstractness을 띠기 때문이다. 그와 동시에 철학은 인류가 지식을 습득하고자 도모하는 과정에서 제기되는 각종 문제, 역설, 혼동 등에 주로 관심하는데, 이러한 문제영역들은 몇몇 이유에서 아직도 자연과학적 관점에서는 명료하게 해명되지 못하고 있기 때문이기도 하다. 어쨌든 Quine의 유명한 선언에 따르면 1철학first philosophy이란 없다. “철학은 오로지 자연과학 자체 내에within science itself 있을 뿐, 실재(實在)reality를 확인하고 기술하는 선험적인 철학이란 존재하지 않는다.” (1981[사물과 이론 내에서 사물의 지위Things and Their Place in Theories, 이론과 사물Theories and Things, 1-23쪽에 수록]: 21) 종종 Quine은 오스트리아의 과학철학자 Otto Neurath(1882-1945)가 제시한 다음과 같은 이미지를 원용하기도 한다: 과학자든 철학자든 우리 모두는 바다 위에서 항해중인 배에 탑승한 채로 그 배를 수리해야만 하는 처지에 있는데도, 배를 가라앉히지 않으면서 그 배 전체를 해체할 수 있으리라는 생각은 오로지 철학자들의 허황된 환상에 지나지 않는다(4, ‘비엔나 학단과 프로토콜 논쟁참조). 어떤 식으로든 원리상 자연과학의 범위를 벗어나는unscientific 지식이란 존재하지 않는다. 즉 과학적 지식은 통상적인 지식ordinary knowledge과 종류를 달리하지 않는다. [다르게 말하면 어떤 것이 도대체 지식이라 불릴 수 있으려면 그것은 과학적으로 존중될 수 있는 성격의 지식이어야 한다.] 역사 속의 점성술이나 동종요법(同種療法)homeopathy 사례에서 볼 수 있듯이, 만약 어떤 학문분과가 자연과학에서 우세한 규준이나 준거에 부합하지 않으려 한다면, 이는 그 학문이 정말로 지식인지 [즉 참거짓을 가리기에 앞서 애초에 진리치-평가 가능한 지식이기나 한지] 여부를 의심해볼 충분한 이유가 된다.

다만 앞서 지적했듯이 Quine의 관점이 무작정 부정적이기만 한 것은 아니다. Quine은 언어에 대해 긍정적인 구상을 제시하는 데에도 상당히 관심하여, 위에서 언급된 자연주의적 기조에 저촉되지 않으면서도 의사소통을 설명해낼 수 있고, 특히 언어가 우리의 믿음과 지식을 담아내어 보존하는 측면을 설명해낼 수 있는 이론을 모색하였다.

 

 

정글 언어학자

 

특정 사람 Jo가 특정 시점 t에 드러내는 모든 언어적 성향(性向)linguistic disposition이 기술된 기록부를 당신이 갖고 있다 해보자. 그 기록부는 시점 t나 그 직후에 Jo가 어떤 문장을 실제로 말할지would actually say를 예측할 필요까지는 없다. 부분적으로 이는 Jo에게 어떤 일이 발생하느냐에 따라 달라지기 때문이다. 단지 그 기록부가 말해주는 바는 Jo가 사용하는 언어의 각 문장 및 Jo를 둘러싼 환경들 집합에 대해, 특정 환경 하에서 특정 문장에 대해 질문을 받을 경우 Jo가 그 문장을 긍정할 것인지would assent 여부이며, Jo가 부정하는 경우 및 Jo의 반응이 미결정되는 경우에 대해서도 동일한 정보가 제공된다. 이런 의미에서 그 기록부는 Jo가 말하는 문장들의 의미와 연관된 모든 자료들을 포함하고 있다. Quine에 따르면 의미에 관해 이 이상의 것을 요구하는 것은 큰 착각이다. 만약 그 이상의 무언가가 분명 있어야 한다고 생각한다면, 이는 한 사람의 언어적 성향을 모두 알고 있는데도 그 사람의 말을 이해하지 못할 것이라 말하는 셈이다. 물론 당연한 얘기겠지만 Jo가 자신의 말을 통해 의미한 바를 당신이 잘 이해하지 못하는 경우도 분명 있을 수 있다. 하지만 언어가 연관되어있는 한에서 당신이 Jo에 대해 알고 있는 바는, 당신 이외의 대부분의 사람들이 통상적으로 서로에 대해 알고 있는 바와 별반 다르지 않다. 우리는 다른 사람들이 언제 어떤 말을 하느냐를 관찰observe함으로써 언어를 배우기 때문이다. 이것이 바로 Quine행동주의behaviourism로서, 이에 따르면 언어현상은 적어도 잠재적으로 공적(公的)public 것이다.

따라서 이런 식으로 언어적 성향이 기록된 자료를 토대로 한 개인의 언어를 설명해낼 수 있다. 좀 더 생생한 논의를 위해, 우리가 이적지 여타 언어로 번역된 적 없는 언어가 사용되고 있는 가상적인 지역을 탐험하고 있다 상상해보자. 우리는 그 지역의 언어를 정글라 부를 것이다. Quine이 말하듯이 우리가 해야 할 과업은 정글어를 한국어로 번역(飜譯)translation하는 체계적인 번역편람을 만드는 일이다. 우리가 한국어를 제대로 이해하고 있다 가정한다면, 이 작업의 결과 만들어진 매뉴얼은 정글어의 각 문장을 하나의 동등한 환경 하에서 그와 적절하게 대응하는 것으로 간주되는 한국어 문장과 짝지어줄 것이다. 이것이 바로 원초적 번역radical translation의 과업이다. 이는 앞서 9에서 살펴본 Davidson의 원초적 해석radical interpretation과 유사해 보이지만(사실 QuineDavidson보다 먼저 이 사고실험을 제기하였다), 양자 간에는 적어도 한 가지 중요한 차이점이 있다. 이에 대해서는 차후에 설명할 것이다.

우리 논의에서 늘상 그래왔듯이, 원초적 번역작업에서도 우리가 주목해야 할 것은 서술문(敍述文)declarative sentence이다. 번역에 착수하기 위해 우리는 정글어-화자들의 분주한 일상에서 이뤄지는 다양한 발화행위들을 관찰하면서, 특히 그들이 무엇에 대해 말하고 있는지 알아내려 노력할 것이다. 어느 날 갑자기 토끼 한 마리가 나타나자 원주민 한 명이 ‘Ga-va-gai!’ 하고 소리친다. 자연스럽게 우리는 이 말이 (온전한 문장인 저기 토끼가 있다!’에 대략 상응하는) ‘토끼다!’와 동등한 문장일 것이라 추측해볼 수 있다. 그런데 좀 더 생각해보자. 원주민이 말한 바는 그 시점에 나타난 특정 토끼만의 이름일 수도 있고, 혹은 통통히 살이 오른 토끼가 나타나자 이를 본 원주민이 저녁식사가 푸짐할 것이란 기대에서 내뱉은 말일 수도 있다. 심지어 토끼와는 아주 무관한 말이었을 수도 있는바, 원주민이 그 말을 한 시점에 토끼가 출몰했던 게 순전한 우연이었을지도 모른다. 가령 그 원주민은 자신의 할아버지에 관해 말하고 있던 중인데 때마침 토끼가 나타났을 뿐인 것이다. 이러한 여러 가능성들을 번뜩 떠올린 당신은 좀 더 확실하게 확인하기 위해 아직 토끼가 눈앞에 있는 상황에서 그 원주민에게 ‘Gvagai?’ 하고 묻는다. 원주민은 ‘Ja’ 하고 대답한다. 잠시 후 토끼가 사라지자 당신은 똑같이 ‘Gavagai?’ 하고 묻는다. 원주민은 이번엔 ‘Nie’라고 답한다. 이제 당신은 다음과 같은 가설을 세운다: ‘Gavagai’저기 토끼가 있다내지 그와 비슷한 것으로 번역될 수 있으며, ‘Ja’‘Nie’는 각각 아니오에 해당한다. 이에 그 부족의 여타 구성원들을 대상으로 토끼가 있거나 없는 상황에서 앞서와 동일한 실험을 여러 번 반복한 뒤, 이 가설을 입증하게 되었다고 해보자.

Quine의 원초적 번역과 Davidson의 원초적 해석 간에는 두 가지 차이점이 있다. 우선 우리의 논의 목적상 결정적인 차이점은, Quine이 오로지 인과적인 용어causal terminology만을 사용하여 원주민의 언어행동을 기술하는바, 관찰가능한 특정 환경에서 원주민으로 하여금 특정 말소리를 발화하도록 야기하는 사안들에만 집중하는 반면, Davidson은 애초부터 원주민의 언어행위를 주장assertion으로서, 즉 진리치를 지닌 무언가를 말하는 것 내지 대상을 언어적으로 지시하는 행위로서 기술한다는 점이다. 다음으로 보다 덜 중요한 차이점은, Quine의 경우 원주민 언어의 한 문장을 가령 바로 그거야That’s it와 같이 지표사가 포함된 문장으로 번역하는 것만으로도 충분하지만, 발화에 진리 개념을 적용하고자 하는 Davidson의 입장에서는 해석자가 지표사의 지시체를 찾아낼 수 있게 해주는 방법을 말해줄 도식 역시 필요하다는 점이다.

‘Gavagai’Quine이 칭한바 관찰문장observation sentence이다. 관찰문장을 긍정하는 성향은 즉각적으로 관찰가능한 환경에서 분명하게 식별되는 사안이 무엇인지에 따라 현시된다. 직접적으로 관찰가능한 환경을 Quine은 관찰문장에 대한 상호주관적 자극-의미intersubjective stimulus-meaning라 칭하였다. 관찰문장은 가령 나 배 아파와 같이 공적(公的)으로 관찰될 수 없는 사안들에 적용되는 주관적subjective 문장과는 다르다. 또한 관찰문장은 Quine이 칭한바 고정문장standing sentence도 아니다. 고정문장이란 대체로 상호-주관적이거나 공적인 문장이되 그 참이나 거짓이 일단 결정되면 그 진리치가 변하지 않는 문장이다. ‘Neil Armstrong1969년 달에 착륙했다헬륨원자의 핵에는 두 개의 중성자가 있다와 같은 문장들은 고정문장이다. 원초적 번역을 시작할 때 당신이 우선적으로 찾아내야 하는 것은 분명 주관적 문장이나 고정문장이 아니라 ‘Gavagai’와 같은 관찰문장이다. Quine의 비유를 들자면 관찰문장은 번역을 고정하는 쐐기“entering wedge” of translation와 같다. 원주민 화자가 ‘Gavagai’를 긍정하는 성향은 토끼가 나타났다가 사라짐에 따라 그와 마찬가지로 현시되었다가 사라질 것이다. 반면 원주민 화자의 긍정-성향과 관찰가능한 환경변화 간의 상관관계를 아무리 살펴보아도 헬륨원자에는 두 개의 중성자가 있다로 번역될 수 있는 원주민 언어의 문장을 찾아내기란 불가능하다. 헬륨원자에는 언제나 두 개의 중성자가 존재하므로, 원주민 화자는 그 문장에 대해 언제나 긍정하는 반응을 보일 것이기 때문이다. 이런 식의 [고정적인] 상관관계는 그 문장의 의미와 전연 무관하며 이는 모든 고정문장에 대해서 마찬가지이다. 요컨대 고정문장의 진리치는 우리 주변 환경에서 무슨 일이 일어나든 동일하게 유지되기 때문에, 모든 고정문장들은 언제나 동등한 자극-의미를 갖는 셈이다. 동일한 자극-의미를 아무리 모은다 한들 그것이 의미가 되지는 않는다.

사실상 우리는 주변에서 일어나는 일들에 관해 상당히 간략하게as shorthand 말하곤 한다. 엄밀히 말해 관찰문장의 자극-의미는 특유의 방식으로 자극되는 신경(神經)nerve being stimulated과만 연관되어 있다. 우리말의 아 뜨거!’와 동일한 역할을 수행하는 원주민 문장의 경우 그 자극-의미는 상대적으로 매우 단순할 것이다. 하지만 ‘Gavagai’의 경우 그에 개입되는 자극-의미는 상당히 복잡할 것이다. 가령 토끼를 명확하게 목격하는 것이나 언뜻 보는 것과 같이 토끼임rabbithood과 연관될 수 있는 모든 다양한 감각적 징후sensory manifestation[즉 어떤 것을 토끼로서 지각할 때 현시되는 신경상태들]뿐만 아니라, 토끼에 의해 통상 야기되는 감각적 상태는 아니지만 실제로는 토끼가 없는데도 착각으로 인해 토끼를 긍정할 때 주체의 감각기관 표층에서 나타나는 상태events at the sensory surfaces , 실로 다양한 사안들이 ‘Gavagai’에 대한 자극-의미와 연관되어 있는 것이다. 다만 이에 대한 상세한 논의는 우리의 당면 목적상 불필요하게 복잡하므로, 이 정도만 언급하는 것으로 충분하다.

관찰문장을 넘어서 정글어의 모든 표현을 번역하기 위해서는, 정글어 문장을 분해하여 문장 이하 단위의 단어들 내지 표현들에 번역을 할당해야 한다. 어떻든 간에 당신은 각 문장들을 하나하나 번역하는 것만으로는 정글어 전체를 번역하는 데에 충분하지 않다는 것을 잘 알고 있다. 중요한 것은 원리적으로 무한히 많은 모든 정글어 문장을 번역하게끔 해줄 수단을 찾는 일이다. 이 지점이 바로 구성성compositionality 원리 내지는 Quine이 말한바 분석적 가설들의 체계system of “analytic hypotheses”가 도입되는 지점이다. 단순한 예로, 우리가 ‘Gavagai blei’저기 하얀 토끼가 있다로 번역하고 ‘Bollogai pirg’저기 검은 뱀이 있다로 번역했다면, ‘blei’하얗다에 해당하고 ‘pirg’검다에 해당한다고 추측해볼 수 있다. 검은 토끼가 나타날 경우 당신은 원주민에게 ‘Gavagai pirg’를 시험해볼 수 있을 것이다. 이러한 절차를 Quine유추적 대체analogical substitution라 칭한다.

방금 예시된 문장들은 관찰문장이었지만, 개별 용어들에 대한 번역은 고정문장을 번역하기 위한 핵심 단초가 된다. 개별 용어들을 번역하는 데에 필수적인 사안은 정글어에서 논리적 관계들을 나타내는 용어terms for logical relations들을 번역하는 것, 가령 정글어에서 우리말의 라면 이다모든등에 해당하는 논리적 용어들을 번역하는 것이다. 다음 절차를 생각해보면 이 작업이 어떻게 진행될 수 있는지 대략 파악할 수 있을 것이다: ‘og’가 정글어에서 하나의 문장-연결사라고 추측하게 되었다 해보자. 임의의 번역된 관찰문장 %에 대해, 전형적인 원주민이 % 각각을 모두 긍정할 경우 og %’를 긍정하고 그 이외에는 og % ’를 부정한다면, 우리는 ‘og’그리고로 번역된다고 확신할 수 있다. 여타 논리적 연결사들에 대해서도 동일한 전략이 활용된다. 이러한 절차에 따라 우리는 정글어의 토대에 있는 논리적 뼈대를 추출해냄으로써, 가령 한국어로 눈이 오면 춥다로 번역될 수 있는 문장들 같은 정글어의 고정문장 및 이론적 문장을 다룰 수 있게 될 것이다.

그런데 우리는 이를 넘어서 번역절차를 더 진행하진 않을 것이다. 이제 Quine이 주장하는 바의 핵심을 살펴볼 지점에 이르렀기 때문이다.

 

 

미결정성

 

이런 방식을 통해 ‘Gavagai’토끼다!’로 번역될 수 있다 해도, 직관적으로 생각하기에 의미론상 그와 동등하지 않은 다른 문장으로의 번역, 토끼다!’와 동일한 것을 의미하지 않는 문장으로의 번역이 존재하지 않는다는 결론이 따라나오지는 않는다. 예를 들어 토끼의 일시적인 시간적 단면을 가리키는 토끼-단면(斷面)rabbit-stage이라는 개념을 생각해보자. ‘토끼다!’토끼-단면이다!’ 내지는 저기 토끼가 있다!’저기 하나의 토끼-단면이 있다!’라는 관찰문장 쌍과 연관된 관찰가능 환경은 동일하다. 다시 말해 둘 중 하나가 주장될 수 있는assertable 경우 다른 문장 역시 주장가능하다. 이뿐만이 아니라 저기 떼어지지 않은 토끼-부분이 있다!’라든가 저기 토끼성()rabbithood이 현시되었다!’와 같이, ‘저기 토끼가 있다!’자극-동의적stimulus-synonymous[동일한 자극-의미를 갖는] 문장들은 얼마든지 인위적으로 만들어질 수 있다. 따라서 토끼다!’(혹은 저기 토끼가 있다!’)‘Gavagai’가 자극-동의적이라는 점에서 동등한 문장이라는 사실이, ‘토끼‘gavagai’가 동의 내지는 -외연적 용어라는 것을 함축하지는 않는다. 그렇다면 우리는 ‘gavagai’토끼토끼-단면중 정확히 어느 것과 동일한 지시를 갖는지 알아내야 한다. 그리고 이를 알아내기 위해서는 와 동일하다is the same as에 해당하는 정글어 표현을 먼저 번역해야 한다.

이제 원주민이 ‘ipso’와 동일하다로 사용하고 ‘yo’를 한국어의 ()’과 같은 지시대명사로 사용한다는 것을 확인했다 해보자. 우리는 원주민이 다음 문장을 긍정하는 모습을 보게 된다:

 

(1) Yo gavagai ipso yo gavagai.

 

원주민은 우리가 동일한 토끼를 두 번 가리킬 때 그리고 오직 그럴 때에만 이 문장을 긍정한다. 이러한 사실은 ‘gavagai’토끼-단면보다는 토끼를 의미한다는 가설을 입증해주는 것처럼 여겨진다. 하지만 기실 그렇지가 않다. (1)은 다음 두 문장으로 번역될 수 있기 때문이다:

 

(2) 저 토끼는 저 토끼와 같다.

That rabbit is the same as that rabbit.

(3) 저 토끼-단면은 저 토끼-단면과 동일한 동물-이력(履歷)의 일부이다.

That rabbit-stage is part of the same animal-history as that rabbit stage.

 

이 두 문장과 연관된 관찰가능 환경은 정확히 동일하다. 복수의 토끼-단면들이 동일한 동물-이력의 일부인 경우 그리고 오직 그 경우에만 우리는 동일한 하나의 토끼에 대해 말할 수 있기 때문이다. 따라서 원주민의 발화성향은 용어 ‘gavagai’의 지시를 고정fix the reference해주지 않는다.

물론 (1)에 나타나는 ‘gavagai’‘ipso’ 등의 용어들은 정글어의 다른 문장들에서도 사용되며, 따라서 이 용어들에 대한 특정 번역은 여타 번역에 영향을 미칠 것이다. 하지만 (1)을 구성하는 용어들에 지시를 할당할 때 우리에게 가용한 관찰자료에 근거하여도 복수의 선택지들이 가능했던 것과 마찬가지로, 번역편람의 다른 부분과 관련하여 또 다른 선택지들이 나타날 경우 앞서 우리가 결정했던 선택지는 [새로운 선택지와 부합하게끔] 적절하게 보완compensate될 수 있다. 이는 우리와 원주민들이 토끼와 토끼-단면의 차이점을 구분하지 못한다는 말이 결코 아니다. 우리가 한국어나 정글어와 같은 하나의 언어를 사용하는 한 토끼 = 토끼-단면은 분명 거짓이다. 그럼에도 불구하고, 번역과정 어느 지점에서든 토끼토끼-단면에 각각 상응하게끔 번역편람을 전체적으로 조정adjustment하기만 한다면, 우리는 정글어 단어 ‘gavagai’를 둘 중 어느 쪽으로든 번역할 수 있다.

이러한 상황은 특정 주소지가 여기에서 얼마큼 떨어져 있느냐는 질문에 대해 ‘7만큼이라고 대답하는 것과 비슷하다. 그 대답이 의미하는 바는 일곱 블록이라는 것인가? 아니면 7마일? 7킬로미터? 7리그? 이런 비근한 사례를 제외한다면, 적어도 Quine이 생각하기에 번역의 불확정성(미결정성)indeterminacy 문제를 원리적으로 해결할 수 있는 방안은 없다. 이는 번역이 일절 불가능하다는 것을 의미하지 않는다. 문제는 오히려 번역이 너무나 쉽다는 것, 특정 단어를 어떻게 번역하느냐에 관해 옳은 답들이 너무 많이 있다는 것이다. 언어적 행동linguistic behaviour에 의해 그어지는 일정 한도를 일단 넘어서면, 번역에 있어서 옳은 것과 그른 것 간의 구분은 사라진다. 사실상 우리는 그저 가장 친숙한 선택지를 취할 뿐이다. 정글어 단어 ‘gavagai’가 우리말 토끼에 상응한다고 약정(約定)stipulate할 순 있겠지만 이는 단지 편의에 따른 선택일 뿐인바, 원주민이 드러내는 언어적 행동이라는 사실만을 고려하는 한 다른 대안들이 취해졌을 수도 있다는 점을 명심해야 한다. 어쩌면 원주민의 사고방식에서는 우리가 취하지 않았던 다른 선택지가 더욱 자연스럽게 여겨질 수도 있다.

이상이 Quine이 말한바 지시 불가해성(不可解性)inscrutability of reference에 대한 간략화된 논증으로서, 이 논제는 소위 번역 불확정성(미결정성)indeterminacy of translation 논제의 일부를 이룬다. 이에 대한 몇몇 논증들 및 그것이 함축하는 바가 더 있긴 하지만, 여기서 살펴본 사항들만으로도 다음과 같은 Quine의 핵심 주장을 파악하는 데에는 충분하다: 명제 개념 및 의미의 동일성 개념은 언어가 사용된다는 평범한 사실을 명확하게 설명해내지 못할 뿐만 아니라, 그러한 사실을 설명하는 데에 필요하지도 않다. 이론적으로가 아니라 통상적으로 생각할 때, 우리는 어떤 언어표현이 다른 표현과 동일한 의미를 갖는 경우에만, 그리고 한 문장이 다른 문장과 동일한 명제를 표현하는 경우에만, 양자가 서로에 대한 정확한 번역이 된다고 생각한다. 어떤 두 번역편람이 하나의 언어를 성공적으로 번역해낸다면, 우리는 그 두 편람이 하나로 수렴converge될 수 있을 것이라 생각한다. 양자 모두 그 언어를 동일하게 번역할 것이라 기대하기 때문이다(물론 번역상의 강조점, 문체, 문법 및 어휘상의 미묘한 차이 등은 있겠지만, 이는 피상적인 차이일 뿐 최종적으로는 두 번역 모두 동일한 의미를 전달하리라 기대하는 것이다). Quine은 이러한 생각을 거부한다. 설사 당신과 내가 의심할 여지없이 올바르고 정확하게 정글어를 각기 번역해냈다고 해도, 우리 둘의 결과물이 상술한 바와 같은 식으로 일치할 것이라 가정할 합리적인 이유는 없다. 두 편람은 정글어의 한 문장을 각기 다른 한국어 문장들로 번역할 것이며, 그 두 한국어 문장은 Quine이 말했듯이 아무리 줄잡아보더라도 그럴싸한 등가관계relation of equivalence에 있지 않다.”(1960[말과 대상Word and Object]: 27) 의미는 객관적인 그 무엇이 아니다. 한 언어표현에 특정 의미를 할당하는 것은 그 이상으로 더 설명될 수 없는 직관적intuitive이고 관심-상대적interest-relative인 사안일 뿐, 특정 개인과 무관한impersonal[객관적중립적인] 과학적 절차에 의해 승인validate될 수는 없다. 주어진 어떤 사람의 한 언어행위에 대해 두 개의 동등하지 않은 판정verdict이 이루어졌는데, 둘 모두 각기 그 사람의 언어에 대한 하나의 완전한 분석(즉 그 사람의 언어적 성향 전부가 빠짐없이 속속들이 기재된 목록)의 일부라는 사실이 밝혀졌다 해보자. 이 경우, 의미가 어떤 식으로든 객관적이라고 생각한다면, 두 판정은 그 사람의 언어행위에 각기 다른 의미가 귀속되고 있다는 데 대한 증거로 간주될 수 있을 것이다. 그런데 이렇게 주장하기 위해서는 의미란 것이 실재하긴 하되 다만 그 사람의 언어적 성향에서 드러나지는 않는다고 가정해야 한다. 하지만 이번 의 초입에 살펴보았듯이 이런 생각을 전면적으로 거부하는 것이 Quine의 자연주의의 기본 착상이다. Quine이 보기에 이러한 사고방식은 [언어현상을 설명함에 있어] 달성될 수 없는 높은 기준치를 요구하고 있을 뿐만 아니라, 그런 요구를 만족할 만한 것이 대체 무엇인지조차 극히 불분명하다.

 

 

의미와 분석적 참

 

이러한 고찰에 따라 Quine은 의미에 대한 과학science of meaning을 정당화해줄 만한 것, 즉 문장에 명제를 할당하고 단칭용어에 대상을 할당하는 식의 의미이론을 정당화해줄 수 있는 사실이란 없다고 결론내린다. 하지만 다른 한편으로 Quine의미라는 단어가 일상언어ordinary language에서 확고하게 자리 잡힌 단어라는 점 역시 잘 알고 있다. 분명 일상적으로 우리는 단어의 의미에 관해 말하고, 단어사전이나 관용구사전 등에서는 언어의 의미를 다루며, 사전을 편찬하는 사람들이나 번역가들은 의미 개념을 활용하여 작업한다. Quine이 의미개념을 부정한다고 해서 이 모든 명백한 사실들마저 부인하는 것은 아니다. 그는 단지 우리가 단어의 의미에 관해 말하는 것들 대부분이 언어적 성향 내지 언어적 행동의 측면에서 설명될 수 있다고 주장하는 것이다. 가령 한 언어-사용자가 말하는 단어 내지 phrase, 주어진 그 상황에서 그 사람이 사용할 법한 여타 단어 내지 구로 대체될 수 있다. [의미 내지 그 비슷한 것에 관해 말할 필요가 있을 경우] ‘그것은 를 의미한다라고 말하기보다는 그 대신, 나는 라고 말하는 경향이 있다be disposed to say 라고 말하는 편이 더 낫다는 것이다. 의미에 대한 이러한 일반적인 관점의 일부로서, Quine의미라는 단어가 실제로 일상어에 도입되고 사용되는 방식 및 사전 편찬가들과 같은 전문가들에 의해 더욱 체계적이고 전문적으로 사용되는 방식에 대해서도 설명을 시도한다. 이에 따르면 의미가 사용되는 매우 다양한 방식이 존재하며 이것들은 그 어떤 식으로도 하나의 체계적인 이론으로 통합되지 않는다. 우리는 언어표현에 의미를 주기give a meaning위해 종종 그와 동등한 여타 단어나 구를 끌어오는 경우가 있다. 특정 목적에 비추었을 때 그에 상응하는 표현을 제시하며, 다른 목적을 위해서는 역시 그에 상응하는 다른 표현이 인용된다. 어떤 경우에는 한 표현의 의미를 설명하기 위해, 가령 ‘‘호랑이는 아시아에서 서식하며 덩치가 크고 주황색 바탕에 검은 줄무늬를 지닌 고양잇과 동물을 가리키기 위해 사용되는 단어이다와 같은 식으로 다른 형식의 단어들을 제시하는 게 아니라, 아예 사물 자체를 가리키기도 한다. 또 어떤 경우에는 특정 표현과 연관된 유추 내지 암시라든가 그 표현에 대한 올바른 용례(用例)들을 통해 표현의 의미를 파악해내기도 한다. 다만 이러한 모든 것들은 일상적으로는 매우 유용하긴 하되 궁극적으로는 부정확한 활동으로서, 엄밀한 과학이라기보다는 모종의 기교art와 같은 것이다.

적어도 Hume 이래로 철학자들은 두 가지 중요한 구분으로서 사실의 문제matters of fact를 표현하는 진술과 관념들 간의 관계relations of ideas를 표현하는 진술 간의 구분에 대해 말해왔다. 이에 상응하여 Kant종합적진술과 분석적진술이라는 구분을 도입하였으며 이 용어는 오늘날까지도 이어져 내려오고 있다. 현대적인 관점에 따르면 분석성과 종합성을 지닐 수 있는 것은 명제, 판단judgement, 마음--사고(思考)thought-in-mind 등이 아니라 진술, 보다 엄밀히 말해 발화-맥락--진술statement-at-context-of-utterance로서, 분석적 진술은 다음과 같이 정의된다: 한 진술은 논리적으로 참logical truth이거나 혹은 동의어를 대체함으로써 논리적 참으로 변환될 수 있는 경우 그리고 오직 그 경우에 분석적이다(緖論 참조). ‘모든 총각은 미혼이다를 생각해보자. 이 진술 자체만으로는 논리적 참이 아니다. 이 진술의 논리적 형식은 그 어떤 A[모든 A] B가 아니다[(x)(Ax→∼Bx)]’인데, 이 형식의 대입례들 중에는 논리적으로 참이 아닌 것들이 무수히 많기 때문이다. 가령 그 어떤 사람도 금성이 아니다는 참이긴 하되 논리적으로 참은 아니며, ‘그 어떤 사람도 캘리포니아에 살지 않는다는 거짓이기 때문에 당연히 논리적 참도 아니다. 그런데 총각미혼 남성와 동의어이다. 따라서 모든 총각은 미혼이다에서 총각미혼 남성으로 대체함으로써 모든 미혼 남성은 미혼이다를 얻을 수 있다. 이 진술의 논리적 형식은 ‘B가 아니면서 C인 모든 것은 B가 아니다[(x)((Bx&Cx)→∼Bx)]’로서 이는 논리적으로 참이다.

동의성 개념은 두 표현이 동일한 의미를 갖는 것으로 정의될 수 있다. [그리고 앞 단락에서 보았듯이 분석성의 정의항에는 동의성 개념이 나타난다.] 이렇듯 분석성 개념은 용어의 의미라는 개념을 가정하고 있다. 그런데 지금까지 살펴보았듯이 Quine은 의미 개념이 일상적인 목적에 유용하다는 데에는 동의하지만 그 이상으로 명확하게 사용될 수 있다는 것은 부정하는바, 정밀한 과학적 목적 내지 철학적 목적에 비추었을 때 의미 개념은 언어적 성향이라는 개념에 자리를 내주어야 한다고 생각한다. 따라서 Quine[의미 개념과 밀접하게 얽혀 있는] 분석성 개념이 철학에서 중요하고 필수불가결한 역할을 담당한다는 생각에 강하게 반대한다.

 

 

경험주의의 두 독단의 논증

 

사실 Quine은 의미 개념을 직접 공박하기에 앞서 분석성 개념을 먼저 공격하였다. 분석성 개념에 대한 사전 공격은 의미에 대한 간접적 공격이었던 셈이다(이와 관련된 저술들로서, Quine의 유명한 혹은 악명 높은 논문 경험주의의 두 독단Two Dogmas of Empiricism1951년에 나왔으며이 글은 약간 수정되어 Quine 1961[논리적 관점에서From a Logical Point of View]에 수록되었다, 말과 대상1960년에 출간되었다). Quine이 우선 명시적으로 공격 대상으로 삼았던 타겟은 Carnap의 견해였지만, Quine이 그 기반을 약화시키고자 했던 분석성/종합성 구분에 의존하고 있는 Ayer, Strawson, Grice 등과 같은 여타 철학자들의 견해 역시 이 논박의 대상이 된다(1961StrawsonGriceQuine의 공격에 대한 답변으로 독단을 옹호하여In Defense of a Dogma共著했다).

이 논문에서 Quine은 의미 개념 및 그와 동일하지는 않지만 매우 밀접하게 연관되어 있는 것들로서 개념적 내용conceptual content, 검증조건verification condition, 명제적 내용propositional content, Frege 등의 개념들이 적법성을 갖는다고 상정되지 않는 한, 분석성 개념 역시 결코 만족스럽게 정의되거나 특성화될 수 없다고 주장한다. 다소 조악하긴 하더라도 이러한 개념들이 어쨌든 받아들일 만하다고 생각하는 사람이라면 Quine의 주장이 매우 의아하게 여겨질 것이다. 여기서 핵심은 [분석성 개념에 대한 논의의 맥락에서] Quine에게 대단히 중요한 사안이 지식(知識)knowledge의 본성, 구조, 역학(力學)관계dynamics 등의 문제라는 점이다. Quine의 공격을 구체적으로 살펴보기에 앞서, 우선 그가 공격하는바 분석성 개념이 요구되는 인식론적 관점의 기본 착상을 오해하지 않도록 주의를 기울이면서 간략히 살펴보자.

진술들 중에는 우리의 감각(感覺)sensation이나 경험에 직접적으로 주어지는immediately given 것에 관한 진술들이 있다. 다음으로 그 자체로는 직접적인 경험적 진술이 아니지만 개념적으로 경험적 진술과 연관되는 진술들이 있다. [이를 -분석적 진술이라 해보자.] -분석적 진술의 사례로서 비근한 것들로는 냉장고 안에 맥주가 있다’, ‘그녀는 싱글이다등이 있고, 이론적인 진술로는 다람쥐의 평균 수명은 25년 이하이다’, ‘에탄올은 -114에서 냉각된다등을 들 수 있다. -분석적 진술과 직접적인 경험 진술 간에 맺어지는 개념적 연관관계는 분석적이다. -분석적 진술과 경험 진술들이 다양하고 복합적인 분석적 관계로 맺어져 있기 때문에, -분석적 진술 S가 특정 의미를 지니게 되고 특정 경험적 진술 E1, E2 등을 논리적으로 함축할 수 있는 것이다. 이러한 경험적 진술 E1, E2 등은 총괄적으로 진술 S의 참을 경험적으로 시험test하는 데에 이용된다. E1, E2 등이 전부 참이라면 S 역시 참인 것으로서 받아들여지며, E1, E2 중 적어도 하나가 거짓이라면 S 역시 거짓인 것으로 기각된다. 그렇다면 몇몇 진술들은 경험과 무관하게 입증되는 것일 수밖에 없다. 가령 냉장고에 맥주가 있거나 없다가 그러한 부류에 속한다. 이러한 진술들은 모든 총각은 미혼이다모든 미혼 남성은 미혼이다와 같이 분석적으로 참인 진술이다. 지식의 전체 체계를 이런 식으로 보는 견해를 인식론적 환원(還元)주의epistemological reductionism라 한다. 이에 따르면 진술을 정당화justification하는 절차란 경험에 직접 주어진 토대(土臺)foundation에서 시작하여 사실상 경험과 직접 비교될 수는 없는 이론적 진술theoretical statement에 이르는 과정이다. 이렇게 수립된 이론적 진술들 각각은 그 자체로 경험에 의해 직접 검증될 수 있는 성질의 것은 아니지만, 경험적 토대에 의해 직접 검증가능한 진술들로 이뤄진 복합체와 인식론적으로 동등한 지위를 갖는다. 요컨대 인지적으로 유의미한cognitively meaningful 모든 진술은 분석적이거나 아니면 검증가능한 진술들 조합에 상당하는 것이거나 둘 중 하나에 속한다.

이 지점에서 Quine이 제시하는 기본 착상은 지식체계의 정당화에 대한 이러한 선형적(線形的)linear인 그림을 인식론적 전체론epistemic holism으로 대체하는 것이다. 우리가 어떤 이론적 진술 S를 시험하려 한다. 그리고 그에 상응하여 기대되었던바 우리의 이론 에서 S에 의해 함축되는 경험적 진술 E5가 실제 시험 결과 반증(反證)되었다falsified 해보자. 전형적으로 S는 자신 혼자만으로 E5의 참을 함축하는 게 아니라 여타 진술들과 함께 E5를 함축한다. 일테면 시험장비의 특성 및 신뢰도에 관한 진술이라든가, 관찰되는 물질의 화학적 구성방식 및 작용방식에 관한 진술, 이론을 검증하는 데에 활용되는 논리적수학적 진술들 등이, S와 더불어 전체적으로 E5를 함축하는 것이다. 그렇다면 원리적으로 시험대에 오르는 대상은 독자적인 S 하나만이 아니라 그것이 포함된 진술들의 집합 전체whole raft of statements인 셈이다. 물론 그 집합의 여타 구성원들은 대체로 S 자체에 비해 좀 더 확고하게 수립된 것들일 수 있다. 우리가 굳이 S를 시험하는 이유는 사실상 검증되는 이론 내에서 그것이 여타 진술들보다 더욱 결정적이기 때문[즉 우리의 이론적인 관심상 더 문제시되기 때문]이다. 하지만 이러한 취사선택은 정도의 문제일 뿐이지 종류의 문제는 아니다. E5가 거짓인 것으로 입증된 경우, 원리적으로는 S와 함께 E5를 함축하는 진술들의 총체에서 S가 아닌 여타 진술들을 철회할 수도 있는 것이다. 그리고 그렇게 철회되는 진술들은 이론 내에서 차례로 또 다른 진술들과 연관되어 있을 것이며 이러한 연결고리는 이론 전체에 걸쳐있다. 이는 매우 원리적인in-principle수준에서의 사안이긴 하지만, 어쨌든 우리가 각각의 특수한 시험, 실험, 관찰 등을 통해 테스트하는 것은 원리상 과학 전체whole of science이며, 이에는 소위 분석적 진술및 수학적 진술들 역시 포함된다. 그렇다. 산술(算術)이나 수학의 진술들마저도 일반적으로 수정(修整)될 수 있는revisable 것이다. 극단적으로 드문 일이긴 하겠지만, 이론을 테스트해본 결과 필요하다면 우리는 수학적 진술들마저 관찰에 부합하게끔 조정할 수 있다.

자주 인용되는 유명한 구절에서 Quine은 다음과 같이 말한다:

 

상당히 평범한 분야인 지리학이나 역사학에서 다뤄지는 사안들부터, 매우 심오한 분야인 원자물리학의 법칙이라든가 심지어 순수수학 및 논리학의 법칙에 이르기까지, 이른바 우리의 지식혹은 믿음의 총체는 인류에 의해 만들어진 거대한 조직체fabric로서 이는 그 가장자리를 따라서만 경험에 영향을 미친다. 또는 비유적으로 말하자면 전체 과학은 경험을 경계조건boundary condition으로 갖는 힘의 field of force과 같다. 그 장의 주변부에서 경험과의 충돌이 발생할 경우 장 전체의 내부에 조정readjustment이 가해진다. 우리가 갖고 있는 진술들의 진리치가 재분배되어야 하는 것이다. 장 전체는 그 경계조건인 경험에 의해 매우 과소(寡少)하게-결정되기under-determined 때문에, 기대에 하는 경험contrary experience을 단 하나라도 마주할 경우 장 내부에서 어느 진술을 재평가해야 하는가에 관해 매우 폭넓은 선택지가 주어진다. 전체로서의 장에 도모되는 평형을 고려했을 때 간접적으로만 경험과 연결된다는 의미에서가 아닌 한, 장 내부에 있는 그 어떤 진술이든 특정 경험과 연결되어 있지 않다.

 

(Quine 1961[논리적 관점에서]: 42-3)

 

인식론적 전체론에 따르면 인간 지식의 총체는 비유적으로 말해 느슨하게 조직된 믿음의 망()web of belief과도 같다. 지식의 체계는 경험적 진술이라는 토대 위에 분석적 진술이라는 시멘트를 발라 수직적으로 세워진 고정적이고 완고한 구조물이 아니다. 따라서 지식은 분석성 개념 내지 의미 개념을 이용하지 않고도 특성화될 수 있다.

본디 Quine이 해결하고자 한 문제는 언어학에 관한 것이 아니라 인식론적인 것이었지만, 인식론적 문제들에 대한 그의 해결책은 언어학에도 직접적인 영향을 끼쳤다. 그리고 실제로 Quine두 독단을 저술하던 시기는 경력 초기에 몰두했던 논리학 및 수학철학에서 차츰 벗어나 언어철학으로 진출하던 시기이기도 하다.

 

 

Quine의 제안: 분석이 아닌 대체

 

8에서 살펴보았듯이 Quine은 명제적 태도 문제에 관한 논의를 본격적으로 촉발한 인물이기도 하다. 믿음과 관련하여 최종적으로 드러났던 문제는, 대언적(對言的) 믿음de dicto belief(명제에 관한 믿음)이 대물적(對物的) 믿음de re belief(통상적인 대상에 대한 믿음)을 함축하는지 여부를 결정해줄 명확하고 정보적인 기준을 정식화하기가 극히 어렵다는 것이었다. 이에 우리는 Kaplan이 발전시켰던 Frege적인 해결책과 Salmon이 제시한 Mill주의적 혹은 Kripke적인 해결책을 간략히 살펴본 바 있다. 그럼 Quine의 해결책은 무엇이었던가? 최종적으로 Quine은 아무런 해결책도 내놓지 않는다! 그는 차라리 이 문제를 그저 제쳐두는 편이 좋다고 생각했기 때문이다. 대물적/대언적 믿음 구분은 믿음-주체가 대상을 무엇이라고 알고 있는가knowing what에 궁극적으로 의존하는데, Quine이 보기에 이는 철저하게 맥락에 따라 좌우되는context-bound 사안일 뿐이다. 우리는 어떤 사람의 얼굴을 알면서도 그 사람의 이름은 모르거나, 반대로 이름만을 알 뿐 그 사람의 외모는 전연 모르는 경우가 있다. ‘내포성intensionality에 대한 일반적인 개념 및 그에 대한 진지하고 일반적인 이론이란 있을 수가 없다. 내포성은 특정 상황에서 유용하게 쓰이는 장치일 수는 있겠으나, 오로지 그뿐이다. 어떤 사람의 믿음이 무엇에 관한 믿음인지 명시하고자 할 때, 믿음-주체가 대상을 특징짓는 고유의 방식에 따라 그 사람의 믿음 내용을 대언적으로 보고하기de dicto belief-report보다는, 그저 대물적으로 보고하는 편이 더욱 편리한 경우가 종종 있다. 가령 ‘AbF하다고 믿는대[AbF라는 것을 믿는대]’와 같이 대언적으로 기술하기보다는, ‘b 알지? A는 그게it F하다고 믿는대와 같이 대물적인 방식으로 기술하는 것이 더 간편할 수 있다.

사실 더욱 중요한 사안은 이 쟁점과 얽힌 논쟁들 자체를 상세히 이해하는 일이 아니라, 애초에 Quine이 이 문제를 제기했던 동기를 파악하는 것이다. Quine은 개별 문장들이 의미를 지닌다는 생각을 거부하며, 이에 따라 자연히 소위 개념적 분석conceptual analysis이라는 것은 존재하지 않는다고 생각한다. 일상언어의 용어들에 의해 표현될 것이라 여겨지는 개념 내지 의미를 발견하고 분리해내어 탐구하는 것은 Quine이 기도하는 목표가 아니다. 엄밀히 말해 Quine은 그러한 것들이 존재한다는 생각 자체를 거부한다. 그런 식의 개념 내지 의미란 걷잡을 수 없이 모호할 뿐이며, 과학적인 관점에서 보자면 아예 존재하지도 않는다. 이 문제와 관련하여 Quine이 개념적 분석 대신 겨냥하는 목표는, 부정확하고 문제투성이인 일상언어로 된 어구locution들을 그러한 결함이 없는 이론적 언어theoretical language대체(代替)replacement하는 것이다. 이는 단순히 언어를 개선하는 것 자체만을 위한 게 아니라, 일상적인 표현방식ordinary mode of expression이 지닌 위험성을 피하면서, 특정 주제에 관한 공식적인 이론적 진술들을 엄밀하게 표현할 방식rigorous mode of expression을 제공하기 위함이다. 대물성/대언성 구분은 일상적으로 유용하긴 하지만, 이에 너무 집착할 경우 사태를 호도할 수 있으며 실제로는 그러한 구분이 없는데도 마치 이를 구분해야 한다고 여기게끔 오도될 수 있다. 이 구분은 극히 과학적인 근거에서 자의적이거나 오해의 소지가 있는 방식으로만 유지될 수 있을 뿐이다. 문장의 의미 개념 역시 이와 마찬가지 운명에 처해있는바 이론적인 목적상 언어적 성향 개념으로 대체되어야 한다.

일상용어를 이론적 용어로 대체하는 전략은 심지어 고유명의 경우에도 적용될 수 있다. 고유명은 일상언어에서 매우 비근하게 사용되지만, 우리가 이제껏 살펴보았듯이 고유명을 만족스럽게 정의하거나 설명하는 일은 매우 어렵기로 악명이 높다. Quine의 해결책에 따르면 고유명은 더욱 단순한 개념이나 의미들로 분석되는 것이 아니라, 고유명이 포함된 언어가 그것이 포함되지 않은 언어로 대체됨으로써 언어에서 아예 제거될 수 있다. 그러면서도 일상언어에서 사용될 때 고유명이 갖는 중요한 측면들은 고유명이 없는 언어에서도 여전히 보존될 수 있다. 예를 들어 우리가 통상적인 고유명 ‘Socrates’가 수행하는 역할을 해낼 법한 하나의 이름을 갖길 원한다 해보자. 그 경우 [특정 고유명이나 이름을 사용하기보다는] 그저 αSocrates한다α Socratises와 같은 술어를 고안하여 그 술어가 그 특정 남자에 대해서만 참이라고 약정할 수 있다 그렇게 하면 우리가 이름 ‘Socrates’를 사용하고자 할 경우 그 이름 자체를 사용하는 대신 앞서와 같이 고안된 술어를 활용하여 ‘Socrates하는 바로 그것the Socratiser라는 기술구를 만든 뒤, 이를 Russell[기술구 이론에 따른] 방식으로 정의하면 되는 것이다.1) 분명 ‘Socrates는 그리스인이다라는 문장은 [이것이 Russell 식으로 분석된 존재 양화문인] ‘그러한 x가 존재한다 (x는 유일하게 Socrates화한다uniquely Socratises x는 그리스인이다)[(x)(USxGx)]’2)가 참인 경우 그리고 오직 그 경우에 참일 것이다. 더욱 결정적인 사항으로서, 그 어떤 이유에서든 그 기술구가 실제로는 아무런 대상도 지칭해내지 못한다면, 후자와 같은 진술은 진리치를 결여하는 게 아니라 단순지 거짓값을 할당받을 것이다.

 

1) “‘Pegasus’와 같이 단순한 낱말로 된 이름 내지 추정상의 이름을 Russell의 기술구 이론 하에 포섭하기 위해 우리는 처음에 이 낱말을 하나의 기술구로 번역할 수 있어야 한다. 이는 사실상 제약을 가하는 요소가 아니다. 만약 Pegasus의 개념이 너무 모호하거나 원초적이어서 그것을 하나의 적당한 기술구로 번역할 방도가 없다면, 다음과 같이 다소 인위적이게 여겨지는 기법을 이용할 수 있다: 즉 더이상 분석될 수도 환원될 수도 없으며 가설적인 개념으로서 Pegasus-being Pegasus이라는 속성에 기대는 것이다. 이를 표기하기 위해 동사로서 ‘Pegasus-이다is-Pegasus내지는 ‘Pegasus하다pegasize라는 표현을 도입한다. 그러면 ‘Pegasus’라는 명사 자체는 파생어로 처리될 수 있고, 최종적으로 ‘Pegasus-인 사물’, ‘Pegasus한 사물등과 같은 하나의 기술구와 동일하게 간주될 수 있다.” (W. V. O. Quine, 존재하는 것에 관하여On What There Is, 논리적 관점에서From a Logical Point of View, 1953/1961: Milton Karl Munitz, 현대분석철학Contemporary Analytic Philosophy(1981), 박영태 , 서광사, 1997(수정판), 636-7쪽에서 인용.)


2) 이를 3에서 살펴본 방식대로 좀 더 상세하게 표기하면 다음과 같다:

그러한 x가 존재한다 (xSocrates한다 모든 y에 대해 (ySocrates한다 y=x) x는 그리스인이다).

(x)(Sx(y)(Syy=x)Gx).

 

즉 이 양화문의 내부 부속문에서 유일성을 표현하는 절인 모든 y에 대해 (ySocrates한다 y=x)[(y)(Syy=x)]’, 본문에서는 α는 유일하게 Scrateses화한다[USα]’와 같이 유일하게라는 부사가 첨가된 술어를 활용하여 간단하게 표기하였다.

또는 술어 및 변항과 결합됨으로써 (즉 개방문과 결합됨으로써) 한정 기술구를 형성하는 요타 연산자 ι를 활용하여 다음과 같이 표기할 수도 있다: 우선 개방문 ‘Sx’에 요타 연산자를 결합하여 한정 기술구 ‘(ιx)Sx’를 형성한다. 이는 자연언어로 ‘S한 바로 그 하나the single thing S에 해당한다. 이 표현 전체는 하나의 개체상항이므로 술어 ‘Gα의 공란에 채워질 수 있으며, 이에 최종적으로 (양화문이 아닌 단순 단칭문장) ‘G(ιx)Sx’가 얻어진다.

* “본디 기초적인 술어논리 언어에는 한정 기술구를 나타내는 단순한 표기법이 없지만, 요타(ι)-연산자iota-opertator라는 논리상항을 도입합으로써 표기법이 확장될 수 있다. 이 연산자를 이용해서 속성 F를 지닌 그 x’‘(ιx)Fx’로 표기할 수 있다. 이 표현은 ‘a, b, c’ 등과 같은 개체상항으로 쓰일 수 있으며, 따라서 가령 ‘G(ιx)Fx’처럼 술어의 논항이 됨으로써 속성 F를 지닌 그 x는 속성 G를 지닌다내지는 더욱 간단하게 나타내면 FG하다로 해석되는 문장을 형성할 수 있다. 요타-연산자는 양화사와 약간 비슷하지만 그 차이점을 아는 것이 중요하다. 양화사는 개방문 앞에 놓임으로써 그 개방문을 폐쇄문으로 만들지만, 요타-연산자는 개방문을 단칭용어(개체상항)로 전환시킨다.” (Jens Allwood , 언어학에서의 논리학Logic in Linguistics, 전병쾌 外 譯, 한신문화사, 1987, 169.)

 


자연주의의 지위

 

전술했듯이 미결정성 논제의 결론은 번역이 불가능하다는 것이 아니다. 일테면 외계인의 마음을 우리가 알 수 없다는 것을 의미하지는 않는다. 오히려 번역이란 게 너무 쉽게 이뤄지기 때문에, 외계인의 마음을 묘사하는 단 하나의 정확한 방식이 있다는 자연스런 생각을 유지하기 어렵다는 것이다. 이는 우리와 동일한 언어를 사용하는 사람들의 마음 및 심지어는 우리 자신의 마음에 대해서도 마찬가지이다.3) Quine의 요지는 특정한 정신주의적(유심론(唯心論))mentalistic 개념들, 가령 정신이 명제를 파악한다는 것이라든가 정신이 외부 대상과 -인과적으로 접촉한다는 등의 개념들이, Quine이 염두에 두는 식의 과학주의적scientistic 관점에서 보자면 진지한 설명의 대상이 될 자격을 갖추지 못한 개념들이라는 것이다. 나는 과학주의적관점이라고 말했지만, Quine에게는 이것이 모든 지식 영역을 포괄하는바 그 어떤 분야의 지식이 되었든 자연주의적 제약에 따라야만 한다. 물론 일상적인 지식ordinary knowledge이 인식적 가치epistemic value가 아예 없다거나 그 어떤 의미에서도 지식이 아닌 것은 아니다. 다만 그것은 엄밀성과 최대한의 객관성을 지니지 못하기에 엄정한 과학적이론적 요구조건에는 부합하지 못할 뿐이다.


3) “Quine은 원초적 번역에서 직면하기 마련인 극단적인 정도의 지시 불가해성이 모국어에서도 일어날 수 있다고 단언한다. Quine은 지적하길, 우리는 누구나 일상적으로 이웃 사람의 말을 (다소 무의식적으로) 자신의 말로 번역하며, 이러한 번역은 실제적으로는 훌륭하게 정당화된다. 하지만 그것이 이론적으로는 그처럼 명쾌하게 정당화되지 않는다. 그 이유는 이웃 사람이 사용하는 개체 구별장치를 번역할 때 번역상의 전체적 균형을 위해 우리의 해석 방식으로 조정하고자 하기만 한다면, 이웃 사람의 담화에서 지시된 내용이라고 간주되는 것을 우리 마음대로 변경시킬 수 있기 때문이다. (中略) 결국 우리는 나 자신이 사용하는 말의 지시작용에 대해서마저, 그 말이 정말로 어떤 대상을 지시하기는 하는지 문제 삼을 수 있다. 한 예로 내가 토끼를 사용할 때 그것이 과연 토끼 내지 그 밖의 무언가 존재하는 것이라 할 만한 대상을 정말로 지시하는지 여부까지도 문제 삼을 수 있는 것이다.” (George D. Romanos, Quine과 분석철학: 언어에 대한 언어Quine and Analytic Philosophy: The Language of Language(1983), 곽강제 , 한국문화사, 2002, 86-7.)

* “우리는 토끼에 대한 이웃 사람의 분명한 언급을 참으로 토끼-단면에 대한 언급으로, 산술학 공식에 대한 분명한 언급을 참으로 Gödel에 대한 언급으로 체계적으로 바꿔 해석할 수 있으며, 이런 일은 그 반대 방향으로도 가능하다. 우리는 다방면에서 결합되어 있는 그 사람의 술어들을, 우리의 번역에 따라 존재론에서 변경된 사항들이 보충되도록 적절하게 재정리함으로써, 이 모든 것을 이웃 사람의 언어습관과 일치시킬 수 있다.” (W. V. O. Quine, 존재론적 상대성 및 다른 小論Ontological Relativity and Other Essays, 1969: 같은 책, 같은 곳에서 인용.)

** “지시 불가해성은 개개인 자신에게도 적용될 수 있는 문제이다. 만일 누군가가 토끼-단면이나 Gödel를 지시하는 게 아니라 한 마리의 토끼나 특정 형식문을 지시하고 있다고 자신에게 말하는 게 의미가 있다면, 다른 누구에게 그렇게 말하는 것 역시 유의미해야 하기 때문이다.” (W. V. O. Quine, 같은 책: 이명숙, 존재론의 상대성, 서광사, 1994, 127쪽에서 인용.)


자연주의에는 동의하되 의미에 대한 Quine의 부정적 결론이 꺼림칙하게 여겨진다면, 다음과 같은 선택지를 취해볼 수 있다: Quine에 반대하여 많은 이들이 주장해왔던 것처럼, 번역이라는 것이 Quine이 주장하는 식으로 미결정적이지는 않다고 가정해보자. Quine적인 원초적 번역에 가해지는 자연주의적 제약을 벗어나지 않되, 번역을 결정해주는 특정 사실들을 Quine이 놓친 것이라 가정해보자는 것이다. 이 경우 문장의미라는 개념은 다음과 같이 이용될 수 있다: 주어진 특정 문장이 올바르게 번역된 모든 문장들, 즉 그 번역 에서 동의적인 모든 번역문들을 취한다. 이 문장들은 직관적으로 동등하게 번역된 문장들 집합(xy를 올바르게 번역한다x correctly translates y라는 관계 하의 동치류(同値類)equivalence class4))을 형성한다. 이 집합은 이전에 문장에 의해 표현되는 의미, 명제, 뜻이라 칭해진 것의 기능을 수행할 수 있다. 즉 이 집합은 진리치의 담지체이자, 원래의 번역 문장과 동의적인 모든 문장들이 (정의에 의해) 공통으로 지니는 것이자, 명제적 태도의 대상이 될 수 있는 그러한 것이다. 이 집합을 의미라 칭하자. 이러한 의미개념에 의존하면 언어에 대한 Quine의 전체적인 그림은 기존의 의미 개념에 대한 우리의 상식뿐만 아니라 통상적으로 생각되어온 의미론과도 자연스럽게 조화를 이루게 될 것이다. 그럼에도 이 집합이 그러한 의미론적 기능을 수행할 수 있는지를 설명하는 일은, 의미개념을 적용하는 것과는 무관하며 그에 선행한다. [자연과학의 관점에서 보자면] 의미개념은 [전통적인 의미개념과 마찬가지로] 여전히 인과적 견고성causal robustness을 결여하는바 진정한 [과학적행동주의적] 설명력을 지니지 않는다. 즉 기본적으로 한 문장이 언어적 행동패턴 내에서 여차여차한 역할을 수행할 수 있는 원인, 그 문장이 특정한 의미를 지니고 있기 때문이라는 식으로 설명할 수는 없다. 왜냐하면 한 문장이 의미를 지닌다고 규정하여도, 그 문장이 특정한 행동적 역할을 수행한다는 설명에는 아무것도 보태지는 것이 없기 때문이다. 기실 자연주의적 테두리 내에서는 의미개념이 인과적으로 견고한 모종의 역할을 수행할 수 있는지 여부 자체가 매우 불분명하다. 의미개념이 그러한 역할을 담당할 수 있다고 생각하는 것은 사적(私的) 언어private language가 가능하다는 극히 잘못된 생각에 개입하는 데에 지나지 않는다.5) 사적 언어 개념에 대한 상세한 논의는 다음 장에서 後期 Wittgenstein을 살펴볼 때 등장하게 될 것이다.


4) 집합 X 위에서의 동치관계equivalence relation란 반사적이고 대칭적이며 이행적인 관계를 뜻한다. 관계 RX 위에서의 관계일 때, 원소 xR-R-classx와 관계가 있는 모든 원소들의 모임, {yX(x, y)X}를 의미한다. R이 특히 동치관계일 때 xR-류를 R에 의한 x의 동치류라 부른다.” (Peter Jephson Cameron, 집합, 논리, 카테고리Sets, Logic and Categories(1998): 이예찬 外 譯, 수리논리학입문, ()신한출판미디어, 2019, 14.) 본문에서 말해지는 관계는 동의성 관계로서 이는 반사성, 대칭성, 이행성을 모두 만족하는 동치관계이며, 따라서 이 집합은 동치류이다. 다만 여기서 말해지는 동의성 개념은 (Quine 식의 자연주의적 제약 하에서) xy를 올바르게 번역한다는 관계를 전제로 하고 있음을 염두에 두어야 한다.


5) “Quine은 개별 낱말의 지시체가 무엇인지 확인함으로써 낱말의 의미를 이해할 수 있다는 식으로 언어를 이해하는 통상적인 견해를 칭하기 위해 박물관 신화(神話)myth of the museum라는 은유를 사용하고, 언어에 대한 경험적 탐구, 특히 초기의 언어습득 과정에 대한 경험적 탐구를 통해 이 신화를 제거하고자 한다. / 박물관 신화에 따르면 낱말들은 제각기 의미를 가지며, 이 의미는 화자의 생각 속 즉 정신이라는 박물관에 고정되어있는 정신적인 것들이다. (中略) Dewey와 같이 자연과학의 주제들을 탐구하는 경험주의적 정신에 입각하여 언어를 탐구하는 자연주의 철학자들에 의하면, “언어란 우리 모두가 공적으로 인지할 수 있는 상황에 처한 다른 사람들의 공공연한 행위를 증거로 삼아서만 습득되는 사회적 기술이다.”(Quine, 1969) 일단 언어라는 제도를 이렇게 파악한다면 사적 언어란 있을 수 없다. “혼잣말도 다른 사람들과의 대화의 산물이요 그 반영이다.”(John Dewey, 경험과 자연Experience and Nature, 1958) Dewey는 언어의 사회적인 면을 강조하여 다음과 같이 말한다: “언어는 적어도 두 사람 즉 화자 및 청자가 상호작용하는 한 가지 양식이다. 언어는 상호작용하는 사람들이 속해 있으면서 언어습관을 그로부터 익히게 되는 조직화된 집단을 전제한다. 그러므로 언어란 일종의 관계이다.” (J. Dewey, 같은 책) 이처럼 언어를 사회적 제도로 간주한다면, 언어를 배우고 그 의미를 이해하는 데 중요한 것은 사람들이 공공연하게 현시하는 언어적 행동성향 뿐이다.” (이명숙, 같은 책, 112-3.)

* “무비판적인 의미론은 의미가 그 전시물이고 낱말들을 그에 관한 꼬리표로 간주하는 박물관 신화이다. 언어를 바꾸는 것은 단지 그 꼬리표를 바꿔 다는 일일 뿐이라는 것이다. (中略) 사람들이 공공연하게 내비치는 행동성향에서 암묵적으로라도 드러날 수 있는 것을 넘어서서, 한 사람의 정신 내에 어느 정도 확정되어 있으리라 간주되는 것을 그 사람의 언어에 대한 의미론으로 간주하는 한, 의미론은 해로운 정신주의mentalism에 오염된다. 하지만 행동을 통해 설명되어야 하는 것은 의미에 관한 사실들이지 의미된 모종의 실재가 아니다. (中略) 언어에 대한 자연주의적이고 행동주의적인 견해로 전향할 때 우리가 포기하는 것은 대화의 모양으로 된 박물관뿐만이 아니다. 우리는 확정성에 대한 보장까지도 포기하는 것이다. 박물관 신화에 따르면 한 언어의 낱맡들과 문장들은 각기 확정적인 의미를 지니고 있다. 원주민이 사용하는 낱말들의 의미를 발견하기 위해서는 그의 행동을 관찰해야 할 것이다. 그런데 이 때 기준이 되는 행동이 낱말의 의미를 발견하는 데에 아무 효력이 없을 경우에도, 그 낱말의 의미는 여전히 원주민의 정신 즉 그의 정신적 박물관에 확정되어 있으리라는 생각이 전제되어 있다. 하지만 Dewey를 따라 의미란 본디 행동의 한 속성이다라는 사실을 깨닫는다면, 겉으로 드러나는 행동성향에 암묵적으로 포함된 것을 넘어서서는 의미와 그 특성 및 그와 유사한 것조차도 존재하지 않는다는 것 역시 깨닫게 된다. 자연주의의 관점에서 보자면 어떤 두 표현의 의미가 동일한지 여부에 관한 물음은 인식되었든 그렇지 않든 간에 확정적인 답을 갖고 있지 않다. 다만 그 답은 인식되거나 그렇지 않거나 간에 사람의 대화 성향에 의해 원리적으로 결정될 뿐이다.” (W. V. O. Quine, 같은 책: M. K. Munitz, 같은 책, 677-8쪽에서 인용.) 

 


역사적 사항


W. V. O. Quine은 젊은 시절 수학적 논리학자로서 학계활동을 시작하였다. 1950년대까지 출간된 논문들 및 단행본들은 대부분 논리학 및 집합론에 관한 것들이었다. 다만 1933, 당시 프라하에서 인식론, 언어철학, 논리학 등의 분야를 연구하던 Carnap과의 만남은 이후 Quine에게 지속적인 영향을 미치는 계기가 되었다. CarnapQuine에게 깊은 인상을 남겼지만 많은 의구심을 들게 하기도 했는데, 대표적으로 Quine이 고작 스물여섯 나이였던 1935년에 저술한 규약에 의한 진리Truth by Convention에서 그러한 의문점이 다소 드러나고 있다.6) 어쨌든 1937년에 나온 수리논리학의 새로운 토대New Foundations for Mathematical Logic는 집합론과 논리학 분야에서 Quine의 명성을 확고하게 만들어주었다. 전쟁 기간 중에는 그간 갈고 닦아온 논리적 기술을 활용하여, 더욱 효율적인 전기적 변환회로를 설계하는 데에 쓰일 알고리즘을 구성하는 작업에 종사하기도 하였다.


6) “Quine은 논리적 진리에 대한 논리실증주의자들의 대답 역시 받아들이지 않는다. 논리실증주의는 논리적 진리란 정의에 의한 진리이자 규약에 의한 진리라고 본다. 이런 입장을 규약론conventionalism이라고 한다. 논리적 진리의 체계란 하나의 형식체계formal system, 즉 정의 없이 주어지는 원초용어primitive terms로부터 정의를 통해 다른 용어들을 얻어내고, 증명 없이 받아들여지는 공리axiom들 및 추론규칙들로부터 다른 진리들을 연역해 내는 체계로 정식화 될 수 있다. 규약주의자들은 여기서 정의 및 공리를 일종의 언어적 규약linguistic convention으로 이해할 수 있다고 생각한다. (中略) Quine은 먼저 논리학이 규약에 의한 진리체계라고 한다면 상당히 많은 경험과학의 이론적 명제들도 마찬가지로 규약에 의한 진리라고 말해야 할 것이라 주장한다. 논리학의 논리체계에 대해서만 규약적 진리라는 이름을 붙일 이유가 어디 있는가? 이론물리학의 공리계도 원초용어와 공리에서 시작하는바 그렇다면 잘 공리화된 이론물리학 역시 규약적 진리라 해야 할 것이다. 이런 식으로 생각하면 논리실증주의자들이 결코 규약적 진리라고 말하고 싶어 하지 않을 많은 진리들이 규약적 진리의 범위에 들어오게 된다. / 더 나아가 Quine은 규약에 의한 진리라는 개념이 논리적 진리의 성격을 해명해주기보다는, 역으로 논리적 진리들을 전제로 하고서만 규약의 체계라는 것이 이해될 수 있다고 논증한다. ‘총각 = 미혼의 성인 남자라는 언어적 규약이 있다고 하자. 이 규약과 모든 미혼자는 재산권을 행사할 수 없다로부터 모든 총각은 재산권을 행사할 수 없다를 도출할 수 있다. 이렇듯 두 규약으로부터 제3의 문장을 도출할 수 있게 해주는 요인은 무엇인가? 그것은 대강 말해 ‘AB이고 BC이면 AC이다라는 논리적 원칙에 의거한 것이 아닌가? 공리와 정의가 규약이라 해도 그로부터 정리들이 연역되려면 다시 논리적 진리에 호소할 수밖에 없다. 이런 점에서 논리란 규약보다 깊은 차원의 것이다Logic does deeper than any convention.”✻✻ 이렇게 해서 Quine은 논리적 진리가 경험적 진리와는 달리 규약에 의해 참이 되는 진리라고 보는 견해를 근거 없는 것이라고 물리친다.” (민찬홍, 윌라드 반 콰인, 현대철학의 흐름, 박정호 外 編, 동녘, 1996, 451-3.)

* 기하학을 다루는 이 마지막 방법[기하학의 공리계를 규약적으로 참이 되게 하는 방법]은 논리로의 정의적 환원에 저항할 수 있는 수학의 여타 분야에도 마찬가지로 적용된다. 각각의 경우 우리는 단지 그 분야에서 참이라 인가된 공준들의 연언문을 구성하되, 거기 나타나는 원초용어들의 의미를 규약적으로 한정하도록 하며, 그럼으로써 그 분야의 모든 정리들은 규약에 의해 참이 된다. (中略) 그러나 이 방법은 수학을 넘어 이른바 경험과학들에게조차 수행될 수 있다. 이 분야에서 최대한으로 정의들의 틀을 짠 뒤. 논리학과 수학에서 채택된 규약들 집합에 새로운 규약을 첨가함으로써, 우리가 원하는 만큼 많은 수의 경험적 원초용어들을 한정할 수 있다. 그러면 경험과학은 기하학의 경우에서 앞서 살펴보았던 방식과 마찬가지로 규약적으로 참이 된다. (W. V. O. Quine, 규약에 의한 진리Truth by Convention, 1936: Hilary Putnam 外 編, 수학철학 選集Philosophy of Mathematics: Selected Readings(1983, 2nd ed), 박세희 , 아카넷, 2002, 533.)

** “논리 자체를 [규약적으로 참이 되게끔] 세우는 데에 쓰여야 할 그 규약들을 채택할 때 직면하게 되는 난점이 있다. 이들 각 규약은 일반적이며 어떤 기술과 일치하는 무한대의 진술들 중 각각이 참임을 말하고 있다. 따라서 일반적 규약으로부터 임의의 특정 진술의 참을 이끌어내는 절차에도 논리적 추론이 요구되며, 이는 무한퇴행infinite regress을 포함한다. (中略) 한 마디로 말해 난점은 논리가 규약들로부터 매개적으로mediately 진행되어야 한다면, 여기에도 규약들로부터 추론하는 논리가 역시 필요해진다는 것이다. (中略) 난점은 [논리적 어법들의 의미를 한정하기 위한] 그 규약들 자체를 전달하는 일이, 우리가 본디 제한하려 시도하고 있는 그 논리적 어법들을 자유롭게 사용하는 데에 의존하고 있고, 또 우리가 그 어법들에 이미 숙달되어 있을 때에만 성공할 수 있다는 데에 있다.” (W. V. O. Quine, 같은 글: 같은 책, 537-9.) 


전후 1951년에 출간된 경험주의의 두 독단Two Dogmas of Empiricism에서 Quine은 철학자들이 비판적으로 사용하는 분석성 개념을 비판하는 동시에, 분석성 개념이 사용되지 않은 Quine 고유의 지식론적 구도를 간략히 제시하면서 철학계에 큰 파문을 일으켰다. 이후 1961년에 출간된 말과 대상Word and Object에서는 특히 번역 미결정성 논제를 논증해내고 의미 개념에 의존하지 않으면서 인간의 지식과 언어를 상세하게 설명하는 데에 주력하였다. 1990년대에는 그간의 견해들을 지속적으로 발전시키고 정교하게 다듬었으며, 이러한 노력은 특히 지시의 뿌리Roots of Reference(1974)진리의 추구Pursuit of Truth(1992, 개정판)에서 드러난다.

좀 더 일반적인 철학사적 견지에서 보자면 Quine은 분석철학이 전개되는 데에 다방면으로 깊은 영향을 미쳤던바, 특히 분석철학에서 선도적인 문제들을 설정하고 그에 대한 나름의 해결책을 분명하게 제시하면서 많은 철학자들로부터 찬동과 비판을 동시에 받았다. 흥미롭게도 Quine지시의 뿌리에서 분석성에 대해 자연주의적인 견지에서도 받아들여질 만한 정의를 제안하면서, 그 개념이 Carnap과 같은 철학자들이 생각했던 종류의 인식론적 역할을 수행하지는 못하겠지만 그래도 그 단어를 사용할 수 있다는 입장을 내비쳤다.

Quine 철학 전반을 철저하게 추종하는 사람은 그리 많지 않다. 하지만 Gilbert Harman, Daniel Dennett, Peter Hylton 등과 같은 비교적 최근의 인물들은 대체로 Quine의 철학을 지지하는 입장을 취해왔다. David Lewis(1941-2001)는 엄밀히 말해 Quine의 추종자는 아니었지만 박사과정 기간 동안 Quine의 지도를 받았으며, 그의 견해는 형이상학 분야에서 여전히 지배적인 위치를 점하고 있다. Donald Davidson은 부분적으로 Quine 철학으로부터 고무되었음에도 엄밀한 의미이론이 가능한지 여부라는 핵심 문제에 대해서는 최종적으로 Quine과 의견을 달리하지만, 그가 발전시킨 견해 역시 다방면에서 많은 추종자들을 거느리고 있다.

 

 

이번 의 요약

 

당시 진행되고 있던 언어철학에 가해진 Quine의 비판은 언어적 현상에 대한 진지한 탐구가 의미 및 그와 연관된 개념들을 비판적으로 수용하지 말아야 한다는 그의 과학적 자연주의로부터 추동되었다. 이와 연관된 논점은 언어적 유의미성linguistic significance이라는 것이 제3자 관점에서 접근가능해야 하며, 특히 언어적 행동의 측면에서 재구성될 수 있어야 한다는 주장이다.

원초적 번역에 대한 사고실험이란 (번역자에게) 일절 알려진 바가 없는 언어를 이미 알려진 다른 언어로 번역하는 절차에 관한 것이다. 이러한 사고실험은 언어에 관한 탐구와 관련하여 자연주의적 입장이 제시하는 요구사항 및 금지사항을 명료하게 드러내주며, 인지적 의미 내지 명제적 내용이라는 개념의 객관적 지위를 확인할 수 있는 방도를 제공한다. 번역의 시초에 활용될 수 있으면서 의미로부터 자유로운meaning-free 개념은 관찰문장이라는 개념으로서, 이는 즉각 관찰가능한 환경에 따라 참이거나 거짓이 되는 문장이다. 이와 대조적으로 고정문장은 일단 참이나 거짓인 것으로 결정되고 나면 그 진리치가 변경되지 않는 문장이다. 이러한 사고실험을 통해 Quine이 내리는 결론은 원초적 번역이 미결정적이라는 것이다. 예를 들면 원주민 언어의 특정 관찰문장을 저기 토끼가 있다로 번역하는 것과 저기 토끼-단계가 있다로 번역하는 것 간에는 객관적인 차이가 없다. 원주민 언어 문장에 나타나는 개별 표현들에 대한 번역을 관찰자료와 부합하도록 조정함으로써, 전체 번역결과가 일관되게끔 유지할 수 있기 때문이다. 앞서 강조했듯이 번역 미결정성 논제가 의미하는 바는 번역이 원체 불가능하다는 것, 그래서 우리와 다른 언어를 사용하는 생물체의 마음을 알 수 없다는 것이 아니다. 오히려 번역이 너무 쉽게 이뤄지기 때문에 그 생물체를 묘사하는 단 하나의 정확한 방법이 있으리라는 생각을 유지할 수 없다는 것이 그 요지이다.

분석적으로 참인 문장이란 오로지 그것을 구성하는 단어들의 의미에 의해서만 참이 되는 문장이다. 독단에서 Quine은 역시 자연주의적 관점에서 의미 개념을 몰아냈으며 그에 따라 분석성 개념도 몰아내 버린다. 분석성 개념은 HumeKant에서부터 AyerCarnap에 이르기까지 지식론 분야에서 중요한 역할을 담당해왔다. Quine은 지식체계에 관한 대안적인 관점으로서 전체론을 제시하면서 분석성 개념을 이 그림으로부터 폐기해버린다. 그가 제시하는 전체론에 따르면 이론이 예측했던 바가 틀릴 경우, 분석적 규칙에 따라 이론 내의 특정 부분에만 조정이 가해지는 게 아니라, 여러 지점에 다양한 조정 가능성이 주어짐에 따라 보다 유연하게 대처할 수 있다. 인간의 지식체계는 경험진술이라는 토대 위에 수직으로 세워진 건축물이라기보다는 느슨하게 짜인 믿음의 그물과도 같다.

일견 Quine의 저술들은 일상적인 언어표현들을 분석하고자 시도하고 있는 것처럼 여겨지기 쉽다. 하지만 기실 Quine이 제안하는 전략은 일상언어 및 심지어 과학적 언어에서 문제의 소지가 있는 표현들을 그렇지 않은 표현들로 대체하는 것이다. 일상언어에서 사용되는 대다수의 표현들은 종종 모호하고 부정확하며 그것들이 함축하는 바가 상호 모순되기도 한다. Quine의 해결책은 그러한 표현들의 의미를 명료하게 분석해낼 방법을 찾을 게 아니라, 이론적인 목적상 필요한 기능을 정상적으로 수행하면서도 일상언어에서 드러나는 문제점을 지니지 않는 언어로 대체하는 것이다.

 

 

탐구문제

 

실용적인 관점에서 의미 개념이 필수불가결하다면, 자연과학은 의미가 실재하지 않는다는 견해를 어떻게 받아들여야 하는가?

 

Mary는 한국어만 할 줄 알고 Baob은 한국어가 변형된 언어인 Quin국어만 할 줄 안다고 해보자. 다소 이상하게 들리겠지만, Bob의 언어와 Mary의 언어는 간접적으로나마 상호 접촉한 바가 일절 없다고 해보자. 그렇지만 두 사람이 만나게 되면 두 사람 모두 어떻게든 원초적 번역작업에 착수하게 될 것이다. Bob저기 토끼가 있다라는 한국어 문장 S저기 토끼-단면이 있다에 해당하는 Quin국어 문장 S로 번역하였다. 꽤나 센스 있는 MaryQuin국어 문장 S저기 토끼-단면이 있다라는 한국어 문장 S✻✻로 번역하였다. S✻✻S의 번역문이고 S✻✻S의 번역문이므로, S✻✻S의 번역문이다. 하지만 이 경우 한국어 문장 저기 토끼가 있다는 한국어 문장 저기 토끼-단면이 있다와 동등하게 되어 버리는 셈이다. Mary는 이 상황에 어떻게 대처할 수 있겠는가?

 

Quine의 이론은 우리가 우리 자신의 마음과 생각 역시 알 수 없다는 것을 함축하는가?

 

분석성을 정의하는 다른 대안이 있다: 한 진술은 그것을 받아들이는 것accepting이 그것을 이해하는 것understanding에 대한 기준인 경우 (즉 한 진술과 연관된 개념을 이해하기 위해서는 그 진술을 받아들여야 하는 경우) 그리고 오직 그 경우에 분석적이다. 이 정의는 기존의 분석성 개념에 대한 Quine의 비판을 견뎌낼 수 있겠는가?

 

BC가 정글어를 모국어로 사용하는 화자라 해보자. 그런데 BC는 매우 다른 감각기관을 갖고 있어서, BC에 대해 ‘Gavagai!’가 갖는 자극-의미 역시 각기 다르다. 이 경우 도대체 무슨 근거로 두 사람이 동일한 언어를 사용한다고 할 수 있겠는가? 두 사람은 어떻게 ‘Gavagai!’라는 관찰문장을 사용하여 의사소통할 수 있는가? 의사소통이 어떻게 발생하는지에 관한 Russell의 구상을 이와 비교해보라.

 

 

주요 읽을거리

 

Quine1차문헌들은 매우 어렵다. 우선적으로 추천하고픈 것은 존재론적 상대성 및 다른 小論Ontological Relativity and Other Essays(1969)에 있는 대상에 관해 말하기Speaking of Object, 選集 논리적 관점에서From a Logical Point of View(1961, 2)에 있는 경험주의의 두 독단Two Dogmas of Empiricism이다. 그 다음으로는 選集 이론과 사물Theories and Things(1981)에 있는 小論 사물과 이론에서 사물의 지위Things and Their Place in Theories를 추천한다. 좀 더 숙달된 독자라면 말과 대상Word and Object(1960)을 읽어보라.

 

 

추가적인 읽을거리

 

Hylton, P. Willard van Orman Quine, 스탠포드 철학 백과사전.


댓글(0) 먼댓글(0) 좋아요(2)
좋아요
북마크하기찜하기
 
 
 

9

Davidson의 언어철학

 

지금까지 살펴본 이론들은 의미와 지시라는 개념을 그 어떤 의미로든 정의(定義)define하고자 시도하지 않고 그저 사용할 뿐이었다. 물론 통속적 이론folk theory에 비해서, 즉 우리가 일상적으로 의미에 관해 말하는 상식적인 방식에 비해서는 그 개념들이 훨씬 더 학술적이고 체계적인 방식으로 사용되었다고 할 수는 있다. 앞선 이론들은 통속적인 이론에서 의미 및 지시에 대응하는 개념들에 암묵적으로 포함되어 있는 요소들을 더욱 세밀하고 명료하게 다듬어 사용한 셈이다. 이와 유사한 사례로서 Newton의 중력이론은 중력 개념을 명확히 정의하지는 않고, 다만 그 개념을 더욱 세밀하게 다듬은 뒤 질량이라든가 거리와 같은 여타 물리학적 개념들과 연관 지어서 사용한다.

의미와 지시라는 개념 내지 현상은 직관적으로 상당히 친숙하지만, 그런 만큼 외려 어떤 점에서는 약간 기이하고 불가사의하게 여겨지기도 한다. 그것들은 대체 어떤 종류의 현상들인가? 소화(消化)digestion현상이나 전하(電荷)현상과 같은 종류의 것인가? 아니면 수학적인 속성 내지는 여타 모종의 -물리적 속성을 지닌 것인가? 내가 킬리만자로산 꼭대기에 있는 돌멩이에 관해 말하기 위해 이 문장에 쓰인 이러한 단어들을 사용하여 그 돌멩이를 언어적으로 지시할 때, 모종의 의미-()meaning-ray 내지 지시-referential-ray이라는 것이 발생해서 나의 마음과 그 대상을 연관 지어주는 현상이 일어나게 되는 것인가?

Donald Davidson은 의미와 지시개념을 사용하지 않으면서 그 개념들의 토대를 이루고 있는 사실들을 기술하고자 시도한다. 대체로 Davidson의 문제의식은 10장에서 살펴볼 인물인 Quine의 철학에서 고무되었다고 할 수 있다. Quine은 자연주의naturalism의 한 형태를 천명한 인물로서, FregeRussell을 비롯하여 여타 많은 철학자들에 의해 사용된 의미 개념에 회의적이었던 인물로 유명하다. 그는 우리가 언어를 갖는다는 현상이 무엇인지를 의미 개념을 무비판적으로 가정하지 않은 채로 설명해줄 수 있는 대안적인 구상을 제시하였다. 이에 DavidsonFregeRussell이 무비판적으로 수용했던 가정에 의존하지 않고 언어적 현상을 탐구해야 한다는 Quine의 주장에 동의한다. 또한 만일 의미에 대한 생산적인 이론화 작업이 가능하다면, 의미 개념이란 경험적으로 답해질 수 있는 것이자 제3자의 관점에서from a third-point of view 언어-사용자language-user에게 적용될 수 있는 것으로서 설명되어야 한다는 Quine의 자연주의적 기조에도 동의한다.

하지만 앞서 말했듯 Quine이 이러한 가능성에 대해 극히 회의적인 입장을 취하는 반면, DavidsonQuine에 비해 훨씬 낙관적으로 전망한다. 적어도 Davidson은 의미이론theory of meaning이 자연과학 내에서 적절한 자리를 차지하지 못하는 한, 만족스런 의미이론을 수립할 가망성 자체가 전무하다고 생각하지는 않는다. 거칠게 말하자면 Davidson은 우리가 의미에 대해 무언가를 알고 있다know는 사실만큼은 부인할 수 없다고 생각하며, 따라서 의미와 관련하여 우리가 알고 있는 바를 체계적이고 이론적으로 기술할 수 있을 것이라 생각한다. 또한 그는 의미이론이 경험적으로 설명될 수 있고 상호-주관적으로 접근 가능한 이론으로 수립될 수 있는 방법을 상당히 성공적으로 제시함으로써, 만족스런 의미이론이 수립될 수 있다는 생각을 실질적으로 고취하였다.

전술했듯 DavidsonQuine으로부터 많은 영향을 받았으며 저술시기 역시 Quine보다 이후이다. 하지만 의미이론에 대한 Davidson의 긍정적인 관점은 앞 들에서 살펴본 주류 이론들에 대해 Quine의 관점보다 덜 멀리 떨어져 있기 때문에, Quine의 철학을 살펴보기에 앞서 Davidson의 이론을 우선 살펴보고자 한다.

 

 

방법론

 

Davidson의 핵심 개념은 L에 대한 의미이론theory of meaning for L이라는 착상이다. 이는 포르투갈와 같은 특정 자연언어 L a given natural language L에 대한 모든 의미론적 사실semantical fact들을 진술하는 하나의 이론이다. L에 대한 이상적인 의미이론은, 문장과 같은 L의 모든 유의미한 표현들에 대해 그 의미를 진술해주는 하나의 정리를 함축한다entail a theorem는 점에서, 그 표현들의 의미를 진술한다. [즉 의미이론은 L에서 형성될 수 있는 모든 개별 문장들에 대해 그 의미를 낱낱이 명시하는 게 아니라, 그 문장들의 의미를 명시해주는 정리들이 도출될 수 있는 하나의 체계인 것이다. 이를 간단히 말하자면 L에 대한 의미이론은 유한하게 공리화finite axiomatize 된 이론이어야 한다.] 여기서 의미이론에 의해 기술되는 언어는 대상언어object language라 불리고, 의미이론을 기술하기 위해 사용되는 언어는 메타언어meta language라 불린다. 이러한 의미이론은 현실에 존재하는 포르투갈와 같은 우연적인 언어현상을 기술하고자 하는바 분명 경험적인 이론이다. 만약 (i) 대상언어로 취해질 수 있는 그 어떤 언어 L이 되었든, L에 대한 의미이론의 모습shape 내지 형식form이 어떠한지 말할 수 있고 [즉 임의의 언어에 대한 의미이론 일반이 갖춰야 할 형식을 기술할 수 있고], (ii) 일반적으로 그런 식의 이론이 실제 현장에서in the field어떻게 입증confirm될 수 있는지 말할 수 있다면, 의미 일반에 대해 철학적으로 말할 수 있는 사안들은 다 밝혀내게 되는 셈이다. 요컨대 의미에 대한 철학적 지식은 (i) 의미이론의 일반형식general form(ii) 의미이론의 입증방식 두 가지를 기술하는 것으로 요약될 수 있다.

 

 

의미이론의 일반형식

 

구성성

 

이러한 의미이론은 과연 어떠한 모습을 갖추고 있어야 할까? 우선 의미이론은 緖論에서 살펴보았던바 다음과 같은 의미에서 구성적(構成的)(합성적(合成的), 조합적(調合的))compositional이어야 한다: 의미이론은 한 문장의 의미가 그 문장을 구성하고 있는 부분들의 의미에 의해 결정되는 방식을 보여주어야 한다. [이러한 요건이 왜 충족되어야 하는지 알아보기 위해] 우선, 한 언어를 안다know는 것이 정확히 어떤 의미인가 하는 문제는 잠시 접어두고, 한 언어 L을 알고 있는 화자(話者)speaker를 생각해보자. 너무 길거나 복잡한 문장을 일단 차치해둔다면, 화자는 L의 한 문장이 제시될 경우 그 의미를 알 것이라는 점에서 L의 모든 문장들을 이해한다understand. 그런데 문장의 길이가 어느 정도를 넘어서 너무 길어질 경우 화자는 그것을 이해하기 어려워할 것이다. 그러니 화자가 현실적으로actually 이해할 수 있는 문장들의 집합은 유한한 셈이다. 하지만 이 집합이 유한한 이유는, 비유적으로 말하자면 화자의 프로그램이 유한하기 때문이 아니라 하드웨어야 유한하기 때문이라고 봐야 한다. 즉 어떤 문장을 화자가 이해하기 어려워하는 이유는, 그 문장의 의미를 결정하는 원리를 화자가 (암묵적으로라도) 알고 있지 못하기 때문이 아니라, 단지 화자가 그 원리를 올바르게 적용하지 못할 정도로 길거나 복잡하다는 데에 기인하는 것이다.

다소 단순한 사례를 취하여, 우리가 이해하는 임의의 단칭용어가 X에 삽입될 경우 ‘X의 아버지의 의미를 결정하는 원리를 우리 모두 파악하고 있다 해보자. 그런데 X에 삽입되는 표현 자체가 의 아버지의 형식을 지니고 있을 수도 있다. 따라서 우리는 ‘X의 아버지의 아버지’, ‘X의 아버지의 아버지의 아버지등과 같이 이어지는 단칭용어들을 얻게 된다. 이렇듯 어떤 장치가 자신의 산출값output을 다시 자신에 대한 입력값input으로 취할 수 있는 재귀적 작동방식reflexive behaviour회귀적(回歸的) 절차recursion라고 알려져 있다. 이런 식으로 회귀성을 갖는 언어표현을 이용하여 가령 ‘X의 아버지는 뚱뚱하다와 같이 임의의 길이를 갖는 문장을 얻을 수 있다. 물론 전술했듯 특정 지점을 넘어서 문장이 너무 길어지면, 우리는 너무 혼란스럽고 지겨워져서 그것을 해석하지 못할 것이다. 하지만 우리의 능력이 우연히 현재와 같은 방식으로 제한되어있지 않았더라면 그렇게 긴 문장 역시 이해하게 해 줄 그러한 원리를 우리는 파악하고 있는 셈이다. [인간 언어능력의 이러한 창조성(생산성)productivity이 반영되도록 언어의 의미를 설명하기 위해서는 의미이론 일반의 형식이 구성적이어야 한다.]

의미이론에 구성성이 요구되는 이유는 다른 방식으로도 고찰될 수 있다. 화자가 자신이 구사하는 언어로 이뤄진 문장의 의미를 안다는 그런 의미에서 한 문장의 의미를 안다는 것은, 어떤 sp에 대해 단지 sp라는 것을 의미함that s means that p을 안다는 데에 그치는 일만은 아니다. 예를 들어 ‘La neige est blanche’라는 프랑스어 문장이 눈이 하얗다는 것을 의미한다는 사실을 안다고 해도, 그 프랑스어 문장을 충분히 이해하고 있는 것은 아니다. 라오스문자들이 나열된 어떤 문장의 의미가 눈이 하얗다는 것임을 당신이 누군가로부터 듣게 되었다 해보자. 그렇다고 해서 당신이 그 라오스문장을 이해하게 되었다 할 수 있는가? 전혀 그렇지 않다. 특정 언어의 문장들을 이해하기 위해서는 문장 전체의 의미가 무엇인지를 알아야 할 뿐만 아니라, 문장을 구성하는 단어들의 의미가 무엇인지, 그리고 문장 전체의 의미가 문장을 구성하는 단어들의 의미에 의해 결정되고 구성되는 방식 역시 알아야 한다. 일련의 언어표현을 이해한다는 것, 즉 방금 살펴본 방식대로 문장의 의미를 안다는 것은 본질적으로 구성적이다.

언어학자들이 종종 사용하는 용어법에 따라 이러한 논점을 말해보자면, 의미에 대한 설명은 언어적 숙달 능력capacity for linguistic competence이 어떻게 생성적generative 혹은 생산적productive인 성격을 갖는지를 적어도 추상적인 수준에서나마 보여주어야 한다. 이와 유사하게 Frege는 언어적 능력이 창조적creative 능력임을 강조한 바 있다.

따라서 의미이론은 다음과 같은 형식을 지녀야 한다: 첫 번째로, 대상언어에 존재하는 (유한한 수효의) 각각의 단순표현들(-복합적인 표현들)에 의미를 할당한다assign. 두 번째로, 유의미한 표현들을 문법적으로 올바르게 결합하는 방식에 따라 복합표현의 의미를 결정하는 원리를 진술한다.

 

잘못된 길

 

이러한 지침을 따르는 도중에, 우리는 대상언어에 있는 각각의 단순표현 내지 복합표현 e에 대해 의미이론이 다음과 같이 말하는 정리를 함축해야 한다고 가정하고픈 생각이 들 것이다:

 

e의 의미는 이다.

The meaning of e is .

 

여기서 의 자리는 의미이론이 목표하는 대상object을 명명하는 단칭용어 즉 표현 e의 의미를 일컫는 단칭용어로 채워질 것이다. 이러한 생각은 일견 매우 자연스러워 보인다. 이런 식의 정리를 산출하는 의미이론은 표현들에 의미를 회귀적으로 할당하는 형식을 지닐 것이다. [즉 이렇게 구상된 의미이론은 우선 구성성 요건을 충족한다.]

하지만 다음과 같은 물음을 제기해보면 이러한 도식이 우리가 원하던 방식대로 작동하지 않는다는 것을 즉시 깨닫게 된다: 각 언어표현에 할당되는 그 의미라는 것은 대체 무엇인가? 단순하거나 복합적인 언어적 항목item들에 할당되는 그 의미란 대체 어떤 종류의 실체entity인가? Frege에 따르면 우리는 뜻과 지시를 구분할 수 있다. 언어표현을 통해 이해되는 것은 바로 Frege가 말하는 뜻이므로, Davidson 식의 의미이론은 단순하거나 복합적인 각 표현들에 을 할당해야 하는 것처럼 여겨진다. 그러나 이제 이러한 작업이 구체적으로 어떻게 진행될 수 있는지 물어본다면 우리는 즉시 문제에 봉착할 것이다. Frege의 도식에서 뜻은 특별한 종류의 실체이다. 가령 ‘Theaetatus’의 뜻은 그러한 하나의 실체이다. 하지만 그 실체란 과연 무엇인가? 이에 대해 이름 ‘Theaetatus’의 뜻을 표현한다고 간주되는 적당한 기술구로서 한 그 남자라는 기술구를 활용하여, ‘‘Theaetatus’의 뜻은 한 그 남자이다라고 답할 수는 없다. 왜냐하면 이 문장은 어떤 이름의 뜻이 특정 남자라고 말하고 있는데, 이는 문자 그대로는 불합리하기 때문이다. [추상적 실체인 뜻이 구체적물리적 실체인 한 남자와 동일할 수는 없다.] ‘Theaetatus’의 뜻을 명시하기 위해 우리가 취할 수 있는 방식은, 그 이름의 뜻이 그저 ‘Theaetatus’의 뜻이라 말하는 것뿐이다. 따라서 우리는 ‘‘Theaetatus’의 뜻은 ‘Theaetatus’의 뜻이다라는 정리를 얻는다. 이는 그저 x = x 형식을 지닌 동일성 법칙의 한 사례일 뿐, 언어표현의 뜻에 관해서는 아무런 정보도 제공해주지 않는다.

뿐만 아니라, Frege적인 접근법에 따르면 ‘Theaetatus가 날아다닌다의 뜻은 ‘Theaetatus’의 뜻과 날아다닌다의 뜻에 의해 구성된다. 여기서 구성된다는 말은 정확히 무엇을 의미하는가? 분명 이러한 접근법은 그 문장의 뜻을 말해주지 않는다. 왜냐하면 우리로 하여금 그 문장을 이해하게 해주지 않기 때문에, 즉 그 문장을 어떻게 해석(解釋)interpret해야 하는지 알려주지는 않기 때문이다. 한 문장의 의미를 구성하는 실체들이 무엇인지를 안다는 것만으로는 그 문장의 의미를 아는 데에 충분하지 않다. 요컨대 의미이론은 실체들이 단순히 나열된 목록list 이상의 것을 요구하는바, 우리는 그러한 실체들이 한데 결합되는 방식 역시 알 필요가 있다. 이는 ‘SocratesPlaton의 스승이다와 같은 관계문장의 경우를 생각해보면 더욱 분명해진다. 이 문장의 의미가 문장을 구성하고 있는 세 가지 표현들의 의미들로 구성된다는 게 의미이론이 말해주는 전부라면, ‘SocratesPlaton의 스승이다의 의미와 ‘PlatonSocrates의 스승이다의 의미는 구분될 수 없다.

의미를 모종의 실체로 간주하는 이러한 접근법이 무망하다는 점은 다른 고찰을 통해서도 깨달을 수 있다. 예를 들어 단일언어 화자monolingual speaker로서 프랑스어만을 사용할 줄 아는 사람이 자신의 언어에 대한 명시적인 지식을 얻고자 한다면, 그 사람은 프랑스어 자체에 대해 프랑스어로 표현된 의미이론을 알아야만 한다. 그런데 그 이론이 한국어로 번역(飜譯)translate된다면 그 의미이론은 한국어 화자에게도 프랑스어에 관한 의미론을 말해줄 수 있어야 한다. 이러한 사정은 프랑스어로 서술된 일반상대성이론이 한국어로 번역되더라도 일반상대성이론 자체에 관한 지식이 한국어 화자에게 온전히 전달되고 이해될 수 있어야 한다는 것과 같다. 그런데 지금 고찰되고 있는 접근법에 따르면 이러한 일은 불가능하다. 프랑스어에 관해 프랑스어로 표현된 이론이 한국어로 번역될 경우, 그 이론에서는 가령 ‘‘boire’의 뜻은 ‘boire’의 뜻이다와 같은 문장이 정리로 도출될 텐데, 이 정리는 한국어만을 사용할 줄 아는 화자에게는 프랑스어로 된 그 단어의 의미를 전혀 말해주지 않기 때문이다.

 

Davidson의 방법: T-문장

 

이러한 난점들을 피하기 위해 Davidson이 취하고자 하는 방식은 문장의 의미가 그 자체로 모종의 실체여야만 한다는 생각을 거부하는 것이다. 다시 말해 우리는 다음 형식의 문장

 

e의 의미는 이다.

 

의 공란을 채워줄 단칭용어를 찾으려 해서는 안 된다는 것이다. 그 대신 Davidson이 실제로 제안하고자 하는 방식의 첫 단계는 일단 다음과 같은 문장을 고려해보는 것이다:

 

sp라는 것을 의미한다.

s means that p.

 

들에 걸쳐 우리는 명사절 ‘p라는 것이 문장 ‘p’에 의해 표현되는 명제를 지시하는 단칭용어라고 가정한 바 있다. 하지만 반드시 그런 식으로 가정해야만 하는 것은 아니다. 우리는 라는 것을 의미한다means that, 한 문장을 다른 문장과 결합시킴으로써 새로운 문장을 산출하는 표현으로 [즉 일종의 문장 연결사로] 간주해볼 수 있다. 따라서 s가 논의 중인 대상언어에 속하는 문장이라면, ‘p’의 자리는 그 언어를 기술하기 위해 사용되는 메타언어 문장으로 채워질 것이다. 이러한 문장의 한 사례를 들자면 다음과 같은 식이 될 것이다:

 

‘La neige est blanche’는 눈이 하얗다는 것을 의미한다.

‘La neige est blanche’ means that snow is white.

 

지금 우리의 과업은 대상언어로 형성된 임의의 문장에 대해 그 문장이 무엇을 의미하는지를 메타언어로 말해줄 이론을 고안하는 것이라는 점을 기억하자. 위 형식의 정리를 함축하는 이론은 그러한 과업을 달성해낸다고 할 수 있는가? 결과물을 보자면 그런 것처럼 여겨지긴 하지만, 이러한 이론을 고안하려는 시도는 매우 난감한 문제에 봉착한다. 문제는 이 이론이 라는 것을 의미한다라는 표현을 사용하고 있다는 점이다. ‘sp라는 것을 의미한다가 참인 정리로 도출되었다 해보자. 여기서 p를 그와 동일한 진리치를 지닌 다른 문장 q로 대체한 ‘sq라는 것을 의미한다의 진리치는 ‘sp라는 것을 의미한다의 진리치와 반드시 동일하지는 않다. 간단히 말해 이 문장에서 ‘s’가 차지하고 있는 자리는 -외연적이다. 앞 장에서 살펴본 용어를 활용하여 말해보자면 사실 이 맥락은 단순-내포적인 게 아니라 -내포적이다. 이 맥락은 가령 ‘p&q’의 경우와 다르다. 여기서 q를 그와 동일한 진리치를 갖는 임의의 문장 r로 대체하더라도, 바뀐 문장 전체의 진리치는 원래의 ‘p&q’와 동일하다. 반면 sp라는 것을 의미한다와 같은 -내포적 맥락의 경우에는, pq가 동일한 의미를 갖는 경우에만 전자를 후자로 대체하여 sq라는 것을 의미한다를 얻을 수 있다. 따라서 의미이론이 라는 것을 의미한다와 같은 형식의 단어를 사용하고 있는 바에야 의미에 관한 우리의 지식을 어떻게 설명해줄 수 있을지는 불분명하다. 이러한 이론은 의미 개념을 선제presuppose함으로써만 의미 개념을 설명할 수 있는 것처럼 여겨진다.

Davidson에 따르면 우리가 진정 원하는 것은 라는 것을 의미한다와 같은 -외연적 개념을 사용하지 않는 의미이론이다. 물론 의미이론이 어쨌든 다음과 유사한 무언가를 말해줄 수는 있어야 한다:

 

‘La neige est blanche’는 눈이 하얗다는 것을 의미한다.

 

다시 말해 의미이론이 산출하는 진술은, 위 문장에서 눈은 하얗다가 사용된 것처럼, 사용된used 메타언어 문장을 통해서 명명된named 대상언어 문장을 해석할 수 있도록 해주어야 한다. , 이것이 순전히 외연적인 방식으로 진행됨으로써 우리 작업의 논리적 성격이 명확히 이해될 수 있어야만 한다.

Davidson의 제안에 따르면 이러한 외연성 요건은 은 참이다is true라는 술어를 사용함으로써 충족될 수 있다. 다음 문장을 보자:

 

‘La neige est blanche’는 눈이 하얀 경우 그리고 오직 그 경우에 참이다.

‘La neige est blanche’ is true iff snow is white.

 

이러한 문장은 Alfred Tarski의 용어법에 따라 T-문장T-sentence이라 불린다. 지금 Davidson이 제안하는 논점을 일반화해서 말하자면, 의미이론은 대상언어에 있는 각각의 문장 s에 대해, 다음과 같은 형식의 T-문장

 

sp인 경우 그리고 오직 그 경우에 참이다.

s is true iff p.

 

를 정리로서 함축해야 한다. 여기서 문장 연결사 인 경우 그리고 오직 그 경우에 이다는 확실히 외연적이며, 그러므로 이 연결사에 의해 형성된 문장의 논리 및 의미는 명료하다(이 연결사는 온전한 진리표를 지니고 있다). 이에 더해 술어 α는 참이다역시 외연적인바, ‘s는 참이다‘s = s로부터 ‘s는 참이다를 올바르게 추론할 수 있다.

확실히 T-문장은 한 문장의 진리-조건을 진술하는 것으로 간주될 수 있다. 하지만 T-문장이 한 문장의 뜻 내지 의미를 진술하는 것으로도 간주될be regarded수 있겠는가? 어떤 문장 s에 대한 T-문장은, ‘sp라는 것을 의미한다와 같이 문자 그대로 s의 의미를 말해주는가? 분명 그렇지 않다. T-문장은 그 자체로는 대상언어 문장의 의미를 기술해주지 않는다. 그런데 사실 DavidsonT-문장이 대상언어에 있는 각 문장의 의미를 직접적으로 진술해야 한다고 주장하지 않는다. Davidson의 요점은 한 언어에 대한 전체로서의as a whole 진리이론theory of truth이 그 언어에 대한 전체로서의 의미이론으로 간주될 수 있다는 것이다. [즉 의미이론이 한 언어의 의미를 기술하는 작업은 의미를 언어표현 낱낱에 직접적으로 할당하는 식이 아니라 진리이론을 경유하여 총체적으로 할당하는 식으로 이루어진다.] 이것이 바로 Davidson의 유명한 전체론(全體論)holism 신조이다. Davidson이 의미에 대한 전체론적 관점을 견지하는 근거를 이해하기 위해서는 다음과 같은 두 가지 사항을 상세히 살펴볼 필요가 있다: (1) 진리이론의 구조, (2) 한 언어에 대한 진리이론이 올바른지correct 여부가 알려질 수 있는can be known 방식.

 

 

의미이론의 정확한 형식

 

한 언어에 존재하는 적형문(適形文)well-formed sentence의 수는 무한하기 때문에 그 언어를 위한 T-문장 역시 무한하게 많을 것이다. 따라서 진리이론이 의미이론의 역할을 수행할 수 있으려면, 그리고 의미이론이 어떤 식으로든 알려질 수 있는 것something knowable이어야 한다면, 한 언어에 대한 진리이론은 유한한 수의 진술들 집합으로 정식화될 수 있어야만 한다. 그러한 진리이론의 한 사례로서, 오로지 원자문장들 및 그것들이 진리-함수적으로 조합된 문장들만으로 이루어져 있는 매우 단순화된 모형언어‘toy’ language L에 대한 진리이론을 살펴보자. 물론 우리에겐 한국어나 프랑스어 같은 자연언어가 훨씬 친숙하게 여겨지겠지만 그러한 자연언어들은 작금의 당면 목적상 엄청나게 복잡하기 때문에, 이러한 가상의 언어를 예시로 삼는 편이 Davidson의 요지를 파악하기에는 훨씬 수월할 것이다. (이하에서 용어 ‘S1‘S2는 메타언어에서 [즉 진리이론을 기술하는 언어에서] 메타언어적 변항metalinguistic variable으로 사용된다. 그리고 한 술어가 한 대상에 대해 참true-of인 경우에만 그 대상은 그 술어를 만족한다satisfy고 말해진다.)

 

() 구문론

 

언어 L은 다음 항목들로 구성된다:

 

(a) 이름: a, b.

(b) 술어: Fα, Rαβ.

(c) 문장 연결사: , .

(d) 문장: 하나의 L-술어에 있는 모든 그리스 문자를 이름으로 대체한 결과는 L의 원자문장이다. L의 모든 원자문장은 L의 문장이다. L의 문장 앞에 가 결합된 것 역시 L의 문장이다. L의 문장 두 개를 로 결합한 것 역시 L의 문장이다. 그 이외의 어떤 표현도 L의 문장이 아니다.

 

() 의미론

 

(a) ‘a’의 지시체 = Socrates.

‘b’의 지시체 = Platon.

(b) 임의의 x에 대해, x‘Fα를 만족한다 iff x는 철학자이다.

임의의 xy에 대해, xy‘Rαβ를 만족한다 iff xy의 스승이다.

(c) L에 있는 임의의 원자문장은 참이다 iff 이름()의 지시체()가 술어를 만족한다.

(d) S1S2L에 있는 임의의 원자문장이라면:

S1은 참이다 iff S1은 참이 아니다.

‘S1S2는 참이다 iff S1은 참이거나 S2는 참이다.

 

이상이 모형언어 L에 관한 사항들 전부이다.1) 


1) (原註) 다만 언급할 몇 가지 사항이 있다. 여기에는 개별 표현들이 적절한 범위 내에 묶여 있음을 나타내기 위해 사용되는 괄호에 대한 사안이 누락되어 있다. 또한 2항술어를 만족하는 순서쌍(順序雙)ordered pairs 개념 역시 명확히 언급되지 않았다. 이 언어에는 더 많은 수의 이름들과 술어들이 추가될 수도 있다. 반면 모든 진리-함수적 연결사들은 여기 주어진 두 개의 연결사에 의해 정의될 수 있기에, L에 여타 연결사들을 추가한다고 해서 본질적으로 달라지는 것은 없다. 양화사가 추가된다면 이 언어가 새로운 방식으로 복잡해지긴 하겠지만, 그렇다고 해도 이 모형언어 사례에 대한 우리 논의의 단순성 내지 간결성에는 큰 차이가 없을 것이다.


이제 L에 대한 진리이론이 어떻게 T-문장을 함축하는지 구체적인 예시문을 들어 살펴보자. 하나의 L-문장 Fb’를 생각해보자. 우선 조항 (-d)로부터 다음을 얻는다:

 

(1) ‘Fb’는 참이다 iff ‘Fb’가 참이 아니다.

조항 (-c)에 의해 다음을 얻는다:

(2) ‘Fb’는 참이다 iff ‘b’의 지시체가 ‘Fα를 만족한다.

조항 (-a)에 의하면 ‘b’의 지시체 = Platon이므로, (2)로부터 다음을 얻는다:

(3) ‘Fb’는 참이다 iff Platon‘Fα를 만족한다.

따라서 조항 (-b)에 의해

(4) ‘Fb’는 참이다 iff Platon은 철학자이다.

이는 다음과 논리적으로 동치이다:

(5) ‘Fb’는 참이 아니다 iff Platon은 철학자가 아니다.

따라서 (1)(5)로부터 최종적으로 다음을 도출한다:

(6) ‘Fb’는 참이다 iff Platon은 철학자가 아니다.

 

(6)L-문장 Fb’에 대한 T-문장이다. 이에서 알 수 있듯 분명 L에서 부정기호에 해당한다(그리고 는 포괄적 의미의 또는인 선언기호에 해당한다). 현실세계에 비추어보자면 Fb’는 거짓 문장이지만 그렇다고 해서 (6)이 참이 될 수 없는 것은 아니다. (1)-(6)은 언어 L에 대한for the language L T-문장을 도출(導出)derivation하는 절차를 구성하는데, 이 도출 자체는 메타언어로 수행되고carried out 있다. 즉 우리는 L의 의미론을 진술하고 그 정리를 추론하기 위해 한국어를 사용하였다. 그리고 L 자체의 문장들은 (1)-(6)의 과정에서 사용use되고 있는 게 아니라 언급mention되고 있다. L에 대한 T-문장인 (6)은 다음과 같은 점에서 특별히 주목을 끈다: (6)은 하나의 L-문장의 진리-조건을 진술한다. (6), 이제껏 L에 주어진 의미이론 전체와 더불어, L의 화자가 (암묵적으로) know 것이라 기대되는 사안을 진술하고 있다. 물론 L-화자는 한국어를 모르겠지만 (6)에서 진술되는 내용 그 자체는 (암묵적으로) 알고 있다. 이는 L-화자가 한국어를 모르더라도 달은 지구를 공전한다에서 진술되는 사실 자체는 알고 있는 것과 마찬가지이다.

무척 단순한 언어이긴 하지만 어쨌든 L 역시 무한한 수의 문장을 지니고 있다. 그렇지만 이제껏 살펴본 다소 간략한 의미이론은 앞서와 똑같은 방식을 통해 L의 모든 문장에 대해 T-문장을 함축한다. L보다 훨씬 복잡한 언어의 경우 그에 대한 진리이론은 여기서 우리가 살펴본 바보다 훨씬 복잡해지겠지만, 앞서 든 예시만으로도 다음과 같은 기본 착상을 이해하는 데에는 충분하다: 진리이론은 [우선 개별 언어표현에] 의미론적 속성semantic property들을 할당하며(가령 에 의해 만족된다와 같은 [메타언어의 의미론적] 술어는 언어표현의 지시에 대한 술어라 할 수 있다[?] [즉 진리이론이 함축하는 정리는 개별 언어표현이 특정 의미론적 속성을 지니고 있음을 기술한다]), “회귀적인recursive장치(가령 조항 (-c) (-d)와 같이 반복적으로 적용가능한repeatedly applicable 수단)를 사용하여 앞서 할당된 의미론적 속성들을 토대로 각 문장에 진리-조건을 할당한다.

 

 

의미이론의 경험적 입증: 원초적 해석

 

앞서 설정되었던 대단히 중요한 과업 하나는 의미이론이 제3자의 관점에서 어떻게 경험적으로 적용가능empirically applicable한지를 보이는 일이다. 지금까지의 논의를 통해 우리는 그러한 이론이 한 언어의 진리-조건에 대한 외연적 이론이어야 한다는 점을 살펴보았으며, 단순화된 예시를 통해 이것이 본질적으로 어떻게 작동하는지 살펴보았다. 하지만 그러한 이론이 현실에서 어떻게 고안되고 실질적으로 입증될 수 있는지는 아직 살펴보지 않았다. 단지 약정에 의해 참이 될 뿐인 가상적인 경우를 살펴보았을 뿐이다.

이제 당신이 아직 학계에 알려지지 않은 언어를 찾아내어 조사하기 위해 인도네시아의 정글 지역을 탐험하는 언어학자라고 상상해보자. 탐험 도중 드디어 당신은 이전에 학계에 확인된 바도 없고 외부로 전파된 적도 없는 언어를 사용하는 한 부족을 만나게 된다. 이러한 원초적 해석자radical interpreter의 처지에 있는 당신이 수행해야 할 과업은 이 언어에 대한 올바른 Davidson적 의미이론(즉 그 언어에 대한 진리이론)을 고안하는 것이다. 당신은 이 작업에 어떻게 착수할 수 있겠는가? 그리고 당신이 고안한 이론이 올바른지 여부를 어떻게 판별할 수 있겠는가(즉 그 이론을 경험적으로 어떻게 입증할 수 있겠는가)?

우선 당신은 원주민들과 많은 시간을 보내면서 그들의 생활을 관찰할 것이다. 그 와중에도 특히나 어떤 사항들을 유심히 관찰해야 하겠는가? 원주민들이 이따금 ‘Gav-a-gai’라 말하는 것을 당신이 듣게 되었다고 해보자(이는 본디 Quine이 제시했던 가상적 사례이다). 원주민들이 어떤 상황에서 무엇에 관해 그 말을 하는지 주의 깊게 관찰한 결과 토끼가 나타났을 때에만 그 말을 한다는 사실을 당신은 알아내게 된다. 다음번에 토끼가 총총거리며 나타나자, 당신은 원주민 한 명에게 질문하는 억양으로 ‘Gavagai?’하고 묻는다. 물론 당신은 이러한 일련의 말소리가 원주민 언어에서 어떤 개별 단어들에 대응하는지 일절 모르는 상태이다. 당신의 말을 들은 원주민이 환하게 웃으며 ‘Jai!’라 말한다. 이번에는 토끼가 없는 상황에서 먼젓번과 동일한 시도를 해본다. 원주민은 ‘Nie!’라고 말한다.

당신은 ‘Jai’가 우리말의 , 맞아요와 같이 긍정을 나타내는 기호sign of assent이고 ‘Nie’는 부정dissent을 나타내는 기호라고 추측한다. 이 원주민 언어에 ‘Nove라고 이름을 붙인 뒤, 당신은 다음 문장이 Nove에 대한 하나의 T-문장이라는 잠정적인 가설을 세워본다:

 

‘Gavagai’는 토끼가 나타난 경우 그리고 오직 그 경우에 Nove-에서-참이다.

‘Gavagai’ is true-in-Nove iff a rabbit is present.

 

그리고 이러한 절차를 원주민들이 말하는 여타 문장들에 대해서도 동일하게 적용해나간다. 지금까지 당신은 문장들에 진리-조건을 직접적으로 할당해왔다. 하지만 개별 문장별로 작업해나가는 이러한 단편적인 방식으로는 Nove전체the whole language에 대한 의미론을 고안하는 데에 턱없이 부족하다. 당신에게 필요한 것은 문장-이하 수준의 개별 표현들sub-sentential expressions에 의미를 할당함으로써, 그 언어 전체에 대한 구성적인 의미론compositional semantics 즉 구성적인 진리-이론compositional truth-theory을 정식화하는 것이다.

그렇다면 문장-이하 표현들에 대한 가설은 어떻게 정식화할 수 있을까? Nove의 또 다른 문장을 들은 당신은 그에 대한 잠정적인 T-문장을 다음과 같이 정식화해본다:

 

‘Bavagai’는 원숭이가 나타난 경우 그리고 오직 그 경우에 Nove-에서-참이다.

 

이제 다음과 같은 단서가 얻어진 셈이다: 확실히 ‘agai’가 나타났다내지는 가 있다정도를 의미하는 표현이고, ‘Gav’토끼, 'Bav'원숭이를 각각 의미한다. 다른 사례를 통해 ‘Bello’으로 해석하게 되었다 해보자. 당신은 뱀이 나타난 상황에서 ‘Bello-agai’ 하고 말해본다. 이에 대해 원주민이 긍정의 반응을 보인다면 이는 당신이 세운 가설을 더욱 입증하는 것으로 여겨질 것이다.

이러한 방식과 유사한 절차를 통해 당신은 Nove에 대한 의미론을 점차 완성해 나간다. 어느 날 유인원에 속하는 오랑우탄 한 마리가 나타난다. 이를 본 원주민 한 명이 ‘Bavagai!’ 하고 외친다. 이 상황에서 당신에게는 다음과 같은 두 개의 선택지가 주어진다: 우선 ‘Bav’원숭이만 의미하는 게 아니라 원숭이-또는-유인원을 의미한다고 가정해볼 수 있다. 겉으로만 보기에 원숭이와 유인원은 매우 유사하게 생겼으므로 이는 나름 그럴듯한 가정이다. 다음으로, ‘Bav’는 오직 원숭이만을 의미하지만 Nove원주민들이 오랑우탄을 원숭이라고 잘못 믿고 있다falsely believe고 가정해볼 수도 있다.

둘 중 어느 선택지를 취해야 하겠는가? 두 선택지 모두 동등한 정도로 설득력을 갖는다equally plausible고 가정하자. 당신은 어떤 근거에서든 둘 중 하나를 선택할 것이다. 이를 기반으로 작업은 어느 정도 순조롭게 진행될 것이다. 하지만 이와 유사한 다른 사례들이 발생할 때마다, 당신은 이전의 해석을 수정revise하거나 아니면 원주민들에게 잘못된 믿음false belief을 귀속시키거나 둘 중의 하나를 선택해야만 한다. 즉 원초적 해석자로서의 당신은 늘상 다음과 같은 처지에 있는 셈이다: 앞서 고안된 이론과 상충하는 사례들을 마주칠 때마다 관찰자료observation를 이론과 조화시키는 방식은 언제나 하나보다 많을 것이며, 그 중 하나의 특정 방식을 취할 경우 이전에 결정했던 이론적 선택은 수정되어야 한다. Davidson에 따르면 이러한 처지에서 각 사례들을 개별적으로individually 결정할 수 있게 해 줄 하나의 일반적인 규칙이란 없다. 오히려 당신이 해야 할 일은 관찰자료들이 이론 내에 조화롭게 수용될 법한 전반적인 패턴overall pattern에 주의를 기울이면서, 어떤 선택지가 그 전체 패턴과 가장 잘 부합하는지를 결정하는 것이다. [즉 관찰된 변칙적 상황을 의미론적 사안 각각에 개별적으로 조화시킬 것이 아니라, 해석된 의미 전체의 패턴과 조화를 이루는 방향으로 취사선택이 이뤄져야 한다.]

이것이 바로 Davidson의 그 유명한 전체론 신조로서, 의미 전체론meaning holism 혹은 의미론적 전체론semantical holism이라 칭해지기도 한다. 이에 따르면 앞서 살펴본 바와 같은 원초적 해석자가 해야 할 과업은, 한 언어의 각 단어들 내지 각 문장들을 개별적으로 해석하여 이를 경험적으로 입증하는 것이 아니라, 이론과 증거가 최대한 전반적으로 합치best overall fit하는지 여부를 확인하는 것이다. ‘이 단어를 이렇게 해석하는 것은 올바른가?’ 하는 물음은 엄밀히 말해 무의미한 질문이다. 단지 우리가 의미 있게 물을 수 있는 것은 한 의미이론 전체가 올바르거나 경험적으로 입증될 수 있는지 여부일 뿐이다. 물론 일상에서든 학문적 논의에서든 우리는 특정 단어들에 대해 앞서와 같은 물음을 종종 던지기는 한다. 다만 그 물음에 대한 특정 대답이 정당한 것으로 받아들여질 수 있는 이유는, 전체로서의 언어에 대한 해석이 그러한 문답이 의미 있게 성립할 수 있는 배경으로 가정되어있기 때문인 것이다.2)


2) “우리는 앞서 문장의 부분들이 그것들이 나타나는 문장의 의미에 체계적으로 기여한다는 존재론적으로 중립적인 의미에서가 아닌 한, 문장의 부분들이 의미를 갖는다고 가정하지 않기로 결정했다. 의미를 가정하는 것이 텅 빈 곳에 그물을 던지는 일이나 마찬가지였기에, 이제 다시 그 통찰로 되돌아가기로 하자. 그 통찰이 가리키는 한 가지 방향은 의미에 대한 어떤 전체론적 견해이다. 만일 문장들의 의미가 문장들 전체의 구조에 의존하고 있고, 우리가 그 구조 내의 각 항목item들의 의미를 그것이 역할을 하는 문장들의 구조 전체로부터 추출함으로써만 이해한다면, 우리는 그 언어의 모든 문장(그리고 단어)의 의미를 제시함으로써만 하나의 문장(또는 단어)에 의미를 부여할 수 있을 것이다. Frege는 단어가 문장의 맥락에서만 의미를 지닌다고 말했다. 동일한 취지에서 그는 문장(그리고 그에 따라 단어)이 한 언어[전체]의 맥락에서만 의미를 지닌다는 말을 보탤 수도 있었을 것이다.” Donald Davidson, 진리와 의미Truth and Meaning, 진리와 해석에 관한 탐구Inquiries into Truth and Interpretation(1984): 이윤일 , 나남, 2011, 64-5, 번역 일부 수정


이러한 관점에는 다소 불안정한 생각이 도사리고 있다. 전체로서의 의미할당 패턴들은 하나의 관찰된 자료에 비추었을 때 각기 동등한 정도로 그와 부합한다는 것, 즉 각각의 패턴들이 전부 동등한 정도로 합당해 보인다는 점이다. 그 경우 우리는 각기 나름대로의 방식에 따라 경험적으로 입증되는 복수의 이론들을 갖게 될 것이다. 그 이론들 모두 원주민의 언어를 해석하거나 그들의 언어로 대화하는 데에 활용될 수 있다. 하지만 두 이론들은 상호 일치하지 않는바 원주민의 언어와 믿음을 각기 다르게 묘사할 것이다. 이러한 가능성을 인식한다면, 실제로는 한 이론이 경험적으로 입증된 것으로 받아들여졌다 하더라도, 그와는 다른 선택지를 취함으로써도 관찰자료와 잘 부합하는 다른 이론에 도달하게 되었을 수도 있다는 점을 알게 될 것이다.

이것이 바로 해석의 미결정성indeterminacy of interpretation 논제이다. 본디 이는 Quine이 이와는 약간 다른 맥락에서 번역translation이 미결정적indeterminate임을 논증하면서 제시했던 생각이다. 중요한 것은 Davidson이 해석의 미결정성을 자신이 취하는 접근법의 한계점이라 생각하지는 않는다는 점이다. Davidson은 자신의 접근법을 통하더라도 의미에 도달하지 못한다고 생각하지 않는다. 의미에 관한 사실들facts about meaning이라는 것이 있어서, 그것이 우리가 시도하는 원초적 해석을 벗어난다는 식의 그림을 거부하는 것이다. 기실 원초적 해석자의 시야를 벗어나서는 그러한 의미-사실이랄 게 존재하지 않는다. 언어-사용자를 해석하는 한 가지 방식이 존재한다면, 그와는 다른 해석방식 역시 분명 존재할 수밖에 없다.

 

 

자비의 원칙, 믿음과 의미의 상호의존성

 

지금까지 우리는 의미이론을 입증한다는 것이 무엇인지, 그러한 이론이 증거와 부합한다‘fit’ to evidence 는 것이 무엇인지, 의미이론을 입증해줄 그러한 증거란 과연 무엇인지 등에 관해 다소 모호하게 말해왔다.

Davidson에 따르면 원초적 해석자가 이용할 수 있는 주된 증거란 특정 환경에서 특정 문장에 대해 원주민 화자가 나타내는(현시(顯示)하는)manifest 바로서 참이라고-간주하는 태도holding-true이다. 예를 들어 원주민 화자는 앞서 묘사된 것과 같이 한 문장을 긍정하거나 혹은 주장assert함으로써 그러한 태도를 현시한다(긍정을 현시하는 반응이 무엇인지를 해석의 시작단계에서 추측해야 하는 것과 마찬가지로, 해석자는 어떤 종류의 발화utterance가 한 문장에 대한 주장을 구성하는지 역시 추측해야 한다).

다소 거칠게 말해보자면, 해석자가 첫 번째로 해야 할 일은 화자가 한 문장에 대해 참이라고-간주하는 태도와 관찰가능한 환경observable circumstance 간에 성립하는 상관관계를 찾아내는 것이다. 가령 어떤 문장에 대해 참이라고-간주하는 태도와 비가 오고 있는 환경 사이에 상관관계가 있음을 밝혀냈다 해보자. 이때 해석자는 비가 오고 있는 경우 그리고 오직 그 경우에 그 문장이 참이라고 추론할 것이다.

그렇다면 원초적 해석의 증거로서 가장 유용하게 쓰일 수 있는 것은, ‘Gavagai’라든가 지금 비가 오고 있다는 것을 의미하는 문장과 같이, 환경에 따라 진리치가 달라지는 문장들일 것이다. 이러한 문장들은 경우문장occasion sentence이라 칭해진다. 이와 대립되는 개념인 고정문장standing sentence은 원초적 해석의 증거로서는 유용하지 않다. 가령 화강암은 나무보다 단단하므로 원주민 화자는 화강암이 나무보다 단단하다는 것을 의미하는 문장에 대해 언제나 그것을 참이라고-간주할 것이다. 그 경우 우리는 그 문장을 긍정하는 성향disposition의 변화와 그 문장의 진리치 변화 간의 상관관계를 알아낼 수 없을 것이다.

이를 일반화해보자면, 특정 문장을 참이라고-간주하는 태도는 다음과 같이 원주민 화자의 믿음과 그 문장의 의미라는 두 가지 요인이 결합되어 나타나는 산물이다(도식 9.1):

 

믿음

 

 

문장의 의미

참이라고-간주된 문장

도식 9.1 한 문장을 참이라고 간주하는 태도를 결정하는 두 요인

 

예를 들어 원주민 화자가 특정 환경에서 ‘Gavagai’를 참이라고 간주하는 이유는, 그 상황에 토끼가 존재한다고 화자가 믿기 때문[(믿음 요인)]인 동시에 ‘Gavagai’는 토끼가 나타난 경우 그리고 오직 그 경우에 참이기 때문[(문장의 의미(즉 진리-조건) 요인)]이다. 물론 어떤 경우엔 원주민 화자가 잘못된 믿음을 갖고 있음으로 인해 거짓인 사안에 대해 참이라고-간주하고 있을 수도 있다. 이것이 바로 Davidson이 칭한바 믿음과 의미의 상호의존성interdependence of belief and meaning이다. 여기서 믿음 개념은 참이라고-간주하는 태도와 실제로 것 간에 불가피하게 벌어지는 간극을 메우는 역할을 담당한다. p라는 것이 실제로는 거짓임에도, 특정 주체에 의해 참이라고-간주된 어떤 문장이 p를 의미한다고 해석하고자 한다면, 우리는 그 주체에게 p라는 거짓 믿음을 귀속시킴으로써 그렇게 해석할 수 있는 것이다.

이 지점에서 다음과 같은 의문점이 들 것이다: 이런 식으로 거짓 믿음을 귀속시킴으로써 화자의 말을 해석하는 일이 매번 발생하지는 않는다는 것을 우리는 어떻게 알 수 있겠는가? 우리는 원주민 화자가 다음과 같이 토끼에 대해 언제나 잘못된 믿음을 갖고 있다고 가정함으로써 ‘Gavagai’를 관찰자료와 일관되게 해석할 수도 있었다: 가령 원주민 화자들은 토끼가 나타나지 않는 경우 그리고 오직 그 경우에 토끼가 나타났다고 믿고 있을 수 있으며, 그 경우 우리는 ‘Gavagai’토끼가 나타나지 않았다로 해석해야 한다. 또는 ‘Gavagai’를 달이 불타고 있는 경우 그리고 오직 그 경우에 참이라고 해석한 뒤, 원주민들은 토끼가 나타난 경우에만 달이 불탄다고 믿는다는 식으로 가정할 수도 있다. 물론 매우 기이하고 그럴듯하지 않은 가설들이긴 하지만, 대체 무엇이 이러한 가설을 원리적으로 배제할 수 있게 해주는가?

이러한 가능성들을 배제할 수 있게 해주는 것은 이런 식으로 우리가 보기에 다소 기이한 가설들이 도입될 경우 그 어떤 의미이론도 정당화될 수 없다는 사실이다. 또는 다르게 말하자면, 그러한 기이한 가설이 일단 허용될 경우, 그 어떤 의미이론이나 해석도식이 되었든 모든 증거와 부합하게끔 고안될 수 있게 되어버릴 것이다. 그러니 해석 작업을 시작하고 이를 적절히 [즉 받아들여질 만한 것으로서] 뒷받침하기 위해서는, 비가 오는지 여부라든가 토끼가 나타났는지 여부와 같이 상대적으로 명백한 사안에 대해 원주민 화자들이 대체로 올바른 믿음을 갖고 있다고 가정해야만 한다. 어린아이가 말을 배우기 시작할 때 어떤 일이 일어나는지 생각해보라. 아이 주변에 있는 사람들이 참된 것과 거짓된 것을 동등한 빈도로 말하는 습관을 갖고 있다면, 아이는 단어들의 의미를 전혀 추측해내지 못할 것이다. 그런 환경에서는 주변 사람들에 의해 말해지는 것과 아이가 관찰할 수 있는 사안들 간에 그 어떤 관찰가능한 상관관계도 없을 것이기 때문이다.

이것이 바로 Davidson의 유명한 자비의 원칙principle of charity이다. 화자가 주변 환경에 대해 대체로 참인 믿음들을 갖는다고 가정하지 않는 한 그 어떤 언어도 해석되고 이해될 수 없기 때문에, 해석의 대상이 되는 화자에게 너무 많은 거짓 믿음을 귀속시키는 것은 애초에 불가능한 일이다. 화자가 상당히 많은 양의 거짓 믿음을 갖고 있다는 것을 알아내기 위해서라도 우리는 어쨌든 화자의 말을 해석해야 하는데, 이를 위해서는 화자가 대체로 참인 믿음들을 갖는다는 가정이 먼저 요구되는 것이다.3) 따라서 해석의 대상이 되는 주체가 대부분 참인 믿음을 갖는다고 가정하는 것은 해석을 위한 방법론적인 의무methodological imperative이다. 다른 생명체creature를 이해하기 위해서는 그 생명체에게 대체로 참인 믿음들을 귀속시켜야 한다. 다시 말해 그들과 우리가 대단히 많은 점에서 일치할 것이라고 가정해야 한다. (Davidson의 이러한 자비의 원칙을 7, ‘함의에서 살펴본 Grice의 협조준칙들과 비교해보라.)


3) 타인의 발언이나 행위를 이해하는 일은, 심지어 가장 일탈적이고 기이한 행위라 할지라도, 그들에게서 대단히 많은 이성과 진리를 찾아내도록 요구한다. 타인을 너무 지나치게 비합리적이라고 가정하는 것은 그들이 무엇에 관해 그렇게 비합리적인지조차도 이해할 수 없도록 만든다.” D. Davidson, 믿음과 의미의 기초Belief and the Basis of Meaning, 같은 책: 이윤일 , 같은 책, 253, 번역 일부 수정,


이에서 더 나아가 Davidson은 언어를 사용하는 그 어떤 생명체도 체계적으로 틀릴systematically wrong 수 없다는 더욱 급진적인 형이상학적 결론을 제시하기에 이른다(따라서 우리가 가진 믿음들은 대부분 참이며, 이에 따라 회의주의는 논박되는 셈이다!). 상상할 수 있는 한 가지 가능성으로서 전지적(全知的)인 해석자omniscient interpreter OI라는 존재를 생각해보자. OI는 그 어떤 생명체든지 해석할 수 있을 것이다(그렇지 않다면 OI는 전지적이지 않은 셈이기 때문이다). 그런데 OI가 지닌 믿음들은 모두 참이고, Davidson에 따르면 해석에는 자비의 원칙이 요구되기 때문에, OI는 해석되는 주체에게 대체로 참인 믿음들을 귀속시켜야 한다. OI는 전지적이므로 이 과정에서 틀릴 수 없다. 따라서 해석되는 주체의 믿음들은 대체로 참이다.

이 논증은 많은 도전을 받아왔다. 한 가지 떠올릴 법한 비판은, 전지적인 존재로서의 OI는 해석되는 주체가 지닌 믿음들을 이미 다 알고 있기 때문에, OI가 자비의 원칙에 의존해야 할 필요 자체가 없다는 것이다. 다른 한편으로, 해석되는 주체의 믿음들을 OI가 미리 알고 있지는 않다고 가정해보자. 말하자면 OI는 통상적인 해석자들이 알 법한 공적으로 관찰가능한 사실들만을 알고 있다. Davidson의 주장대로 해석을 위해 자비의 원칙에 따른 해석자와 해석자 간의 일치가 요구된다면, 이 경우 OI는 그 주체를 해석할 수 없을 것이다.

 

 

역사적 사항

 

Donald Davidson은 자신의 지적 자서전에서 강조하길, 의미란 무엇이 되었든 간에 공적인 것something public이어야 한다는 생각, 즉 원리적으로 해석자 내지 언어를 배우는 아이가 이용할 수 있는 것이어야 한다는 생각으로부터 일찍이 큰 영향을 받았다고 술회한 바 있다. 그가 이 주제에 관해 깊이 숙고한 뒤 1965년에 의미이론과 학습 가능한 언어Theories of Meaning and Learnable Language(Davidson 1984[진리와 해석에 관한 탐구Inquiries into Truth and Interpretation]에 수록) 1967년에 진리와 의미Truth and Meaning(‘주요 읽을거리참조)를 각각 발표하고 나자, 그에 관한 문헌들이 잇따라 대거 쏟아져 나오면서 많은 파문을 일으켰고, 그의 관점을 적용 및 수정하려는 다양한 시도 역시 이뤄졌다. 머지않아 그는 많은 주목을 받게 되었으며, 그의 관점을 논평했던 유명한 인물들로는 Richard Rorty(1931-2007), Tyler Burge, John McDowell, Ernie Lepore 등이 있다.

언어에 관한 Davidson의 이론은 실질적으로 엄청나게 풍부한 결실을 가져왔다. 그의 이론으로부터 고무된 많은 철학자들 및 언어학자들은 매우 다루기 힘든 골칫거리였던 부사, 서술적 형용사predicative adjective, 지표사 및 지시사, 시제(時制)tense, 서법(敍法)mood, 인과(因果)진술causal statement, 명제적 태도 등의 언어요소들을 Davidson적인 진리-조건이론을 통해 해석할 수 있는 방법을 모색하였다(Davidson 자신 역시 저술활동 초기에 이러한 주제들에 천착하였다). 또한 Davidson[心身문제mind-body problem와 관련하여] ‘무법칙적 일원론anomalist monism으로 알려진 관점을 발전시키면서 심리철학 분야에서도 매우 중요한 업적을 이룩하였다. 이 이론에 따르면 정신상태에 대한 기술mental description은 물리적 기술physical description로 환원되지 않지만not reducible, 그럼에도 양자는 동일한 (물리적) 대상을 기술한다.

후기의 저서들에서 Davidson은 의미이론에 대해 이전보다는 덜 낙관적인 태도를 취하였다. 모든 사람들이 각자 사용하는 언어가 미묘하게 다를 뿐만 아니라 사람들에 의해 사용되는 언어는 항시 변하기 마련이며, 우리는 때때로 (말장난malaprophism을 의도하면서) “잘못 말하는mis-speak경우도 있다. 그렇기 때문에 이른바 의미이론이라는 형식적인 대상들은 단지 이상화된 것일 수밖에 없으며, “여러 이론들을 거치는 과정에서각 이론들은 일정 시점에만 적용될 수 있을 뿐이다. 실제로 Davidson은 단호한 어조로 말하길, 언어가 자신의 의미이론의 진정한 주제라면 언어라 할 만한 그러한 것은 기실 존재하지 않는다고 하였다.

 

 

이번 의 요약

 

Davidson은 의미 현상에 접근하면서 다음과 같은 두 가지 주요 요구조건을 내세운다: 특정 언어 L에 대한 의미이론은 L의 각 문장에 대해, 그 문장의 의미를 어떤 측면에서 진술하거나 보여준다고 할 수 있는 정리를 함축해야 한다. 그리고 우리는 그 이론이 경험적으로 입증되는 방식을 설명해내야 한다. 이러한 의미이론의 일반적인 형식(특정 언어 L의 세부사항과는 무관한 의미이론 일반의 형식)을 성공적으로 기술해낼 수 있다면, 우리는 의미 역시 일반적으로 기술해내는 셈이다. 한 언어에서 구성될 수 있는 문장의 범위는 잠재적으로 무한하기 때문에, 의미이론은 구성적이어야 한다.

Davidson에 따르면 의미이론은 단순히 각 언어표현에 대해 Frege적인 뜻과 같은 모종의 실체를 할당하는 형식을 취할 수는 없다. 이러한 도식은 불가피하게 -정보적uninformative일 뿐만 아니라 목록 문제list problem에 직면하게 되기 때문이다. 이러한 도식을 거부한 뒤 Davidson은 우선 한 언어에 대한 의미이론이 그 언어의 모든 문장 s에 대해 ‘sp라는 것을 의미한다형식의 참인 진술을 도출하는 이론이어야 한다는 생각을 먼저 검토해본다. 이 착상의 문제점은 그러한 진술이 -내포적이라는 데에 있다. 그 진술에는 [-내포적 맥락을 조성하는] ‘라는 것을 의미한다라는 용어가 포함되어 있으며, 따라서 이런 식의 문장은 우리가 기도한바 의미를 순환적이지 않은 방식으로 해명해내고자 하는 이론[즉 의미 개념에 의존하지 않은 채 의미를 설명하는 이론]에서는 사용될 수 없다. 이러한 고찰을 거친 뒤 Davidson진리이론이 의미이론으로 간주될 수 있다는 착상을 제안한다. 진리이론의 목표는 ‘sp인 경우 그리고 오직 그 경우에 참이다형식의 ‘T-문장을 산출하는 것으로서, T-문장 형식에서 s는 임의의 대상언어 문장이고 p는 메타언어 문장이다. 일찍이 Tarski는 단순 단칭용어에 대한 지시-할당referential-assignment 및 단순 술어에 대한 만족-조건satisfaction-condition 등과 같은 유한한 토대로부터 무한한 T-문장을 산출하는 의미론적 진리이론을 구성함으로써, Davidson이 구상한 형식의 진리이론이 어떻게 고안될 수 있는지 보여준 바 있다.

유념할 사항은, 참인 T-문장들이 대상언어 문장들 각각의 의미를 문자 그대로 개별적으로 기술하지는 않는다는 점이다. 그렇다기보다, 한 언어의 문장들이 지닌 의미와 관련하여 기술되어야 할 모든 사안을 전달해주는 것은, 그 언어에 대한 모든 참인 T-문장들 그리고 오로지 참인 T-문장들만을 산출하는 전체로서의 진리이론이다. 요컨대 의미이론은 원자적atomic인 것이 아니라 전체론적homistic이다.

이런 형식을 갖는 의미이론이 실질적으로 입증되는 방식을 설명하고자 한다면, 우리가 전적으로 처음 접하는 언어에 대한 의미이론이 어떻게 고안되는지 설명하는 것으로 충분하다. 이것이 바로 Davidson적인 원초적 해석Davidsonian radical interpretation으로서, 이러한 작업은 언어에 대한 우리의 이해가 토대하고 있는 바를 명료하게 드러내 줄 것이다. 우선 해석자가 작업을 시작해야 할 적절한 지점을 제공하는 것은 나무는 탈[燃燒]burn 수 있다와 같은 고정문장이 아니라 비가 오고 있다와 같은 경우문장이다. 해석자는 원어민 화자의 언어적 행동[즉 특정 환경에서 특정 경우문장을 발화하는 것]을 근거로 삼아 ‘‘Gavagai’는 토끼가 나타난 경우 그리고 오직 그 경우에 Nove-에서-참이다와 같은 가설적인 T-문장을 정립해볼 수 있다. 이러한 가설은 원주민 화자들이 보여주는 일련의 언어행동을 계속 관찰함에 따라 여타 가설들과 더불어 점차 입증될 것이다. 그 과정에서 문장들을 그 이하 단위의 언어표현들로 분석하여 그 각각에 지시조건 및 만족조건을 할당함으로써 또 다른 T-문장이 산출될 것이며, 이 역시 차후의 관찰에 따라 입증될 것이다. 이러한 절차는 온전한 의미이론이 구성될 때까지 계속된다.

토끼가 나타난 상황에서 원주민 화자가 ‘Gavagai’에 대해 긍정하는 반응을 보인다면, 이 경우 우리가 추정해봄 직한 사안은, ‘Gavagai’는 토끼가 나타난 경우 그리고 오직 그 경우에 참이라는 것, 그리고 원주민 화자는 토끼가 나타났다고 믿는다는 것이다. 이를 다르게 말하자면 ‘Gavagai’는 특정 상황 하에서 원주민 화자에 의해 참이라고-간주되며, ‘Gavagai’는 그 상황 하에서 참이다. 이런 식으로 [해석의 과정에서 언어의 진리-조건과 언어-사용자의 믿음 간의 합리적인 상관관계를] 추정하는 것을 Davidson의 자비의 원칙이라 한다. 이러한 가정이 없으면 해석은 전연 불가능하다. 물론 이런 식의 원칙에 따르는 게 언제나 옳은 해석만을 가져다주는 것은 아니다. 우리는 거짓인 문장을 실수로 참이라고 간주하는 경우가 왕왕 있기 때문이다. 인간은 유한한 존재자이기 때문에, 해석에서는 이러한 실수가 허용되어야만 한다. 예를 들어 토끼가 없는데도 이따금 원주민 화자가 ‘Gavagai’에 대해 참이라고-견지하는 태도를 보인다면, 그 경우에도 우리는 그것이 토끼가 나타났다를 의미한다는 이전의 해석을 여전히 고수할 수 있다. [화자가 토끼가 나타났다고 잘못 믿고 있거나 토끼가 아닌 무언가를 토끼로 착각하고 있다는 식으로 화자에게 거짓 믿음을 귀속시킴으로써, 우리가 해석해낸 의미 체계를 유지할 수 있는 것이다.] 다만 이는 제한적으로만 취할 수 있는 선택지이다. 만약 그러한 경우가 너무 자주 발생한다면, 화자에게 거짓 믿음을 귀속시킴으로써 우리의 해석을 고수하려는 시도는 더 이상 유지될 수 없다. 그러한 경우엔 [언어-사용자가 지닌 믿음의 합리성을 대체로 우리와 비슷하게끔 유지시킨 채] 우리의 해석을 수정해야 한다. 믿음과 의미는 이런 식으로 상호침투interpenetration한다.

 

 

탐구문제

 

1. 대개 토착민 부족들은 우리가 보기에 황당무계한 것들을 믿지만, 그들 역시 언어를 사용하며 우리는 그 언어를 충분히 조사 및 연구하여 이해할 수 있다. 이러한 사실은 해석자에게 대체로 참인 믿음을 귀속시켜야 한다는 자비의 원칙과 긴장을 일으키지는 않는가? 자비의 원칙을 가령 다음과 같은 식으로 좀 더 세심하게 다듬는다면 이 사실과 더 잘 부합할 수 있겠는가?: 직접 당면한 환경 하에서 명백한 사안what is evident in the immediate environment에 대해서는 해석자가 대체로 참인 믿음을 갖는다고 가정하라.

 

2. 만약 우리가 항상 참인 믿음만을 갖고 거짓인 믿음을 일절 갖지 않는다면, 분명 Davidson은 자신의 이론에 믿음이라는 개념을 도입할 필요가 전혀 없었을 것이다. 그에 따라 그의 이론 내에서 사용되는 은 참이라고 간주된다is held true는 표현은 단순히 은 참이다is true로 대체될 것이다. 그 경우 도대체 믿음이란 것은 오로지 거짓 믿음이 존재할 경우에만 존재할 수 있는 것처럼 여겨진다. 이는 과연 합당한가?

 

5. 전체론은 한 언어에 존재하는 모든 표현들이 상호의존적이라는 것을 함축한다. 그렇다면 특정 의미를 지닌 단 하나의 문장만으로 이루어진 언어란 불가능한가?

 

6. 전체론은 한 언어에 존재하는 모든 표현들이 상호의존적이라는 것을 함축한다. 따라서 한 언어에 존재하는 모든 표현들의 의미를 배우지 않은 채 개별 표현의 의미를 배우기란 불가능하다. 하지만 그 경우 하나의 언어를 배운다는 것은 도무지 불가능한 일이 돼버린다. 우리는 한 언어 전체를 단번에 학습하지는 않기 때문이다. 그러므로 전체론은 거짓이다. 이에 대해 전체론의 옹호자는 어떻게 답할 수 있곘는가?

 

7. Davidson은 다음과 같은 반론에 무엇이라 답하겠는가?: 우선 다음 문장을 보자: ‘‘Gavagai’는 토끼가 나타난 경우 그리고 오직 그 경우에 참이며, 모든 캥거루는 캥거루이다.’ 이 진술은‘‘Gavagai’는 토끼가 나타난 경우 그리고 오직 그 경우에 참이다와 정확히 동일한 진리-조건을 ‘Gavagai’에 할당한다. 따라서 Davidson의 입장에서 보자면 두 진술의 의미는 동일한 셈이다. 하지만 이는 분명 받아들여질 수 없다.

 

 

주요 읽을거리

 

우선 가장 중요한 문헌을 꼽자면 진리와 의미Truth and Meaning원초적 해석Radical Interpretation이다. 두 글 모두 Davidson진리와 해석에 관한 탐구Inquiries into Truth and Interpretation(1984) Davidson 選集The Essential Davidson(2006)에 실려 있다. 두 단행본 중 전자에는 믿음과 의미의 토대Belief and the Basis of Meaning, Foster에 대한 답변Reply to Foster, 지시 없는 실재Reality Without Reference, 지시 불가해성(不可解性)The Inscrutability of Reference등 언어철학적인 글들이 다수 실려 있으며, 후자에는 비문(碑文)의 미묘한 혼란A Nice Derangrment of Epitaphs이 실려 있다. 「…라고 말하는 것에 관하여On Saying That은유가 의미하는 것What Metaphors Mean은 두 단헹본 모두에 실려 있다.

 

 

추가적인 읽을거리

 

Evnine, S. (1991), Donald Davidson.

Malpas, J. Donald Davidson, 스탠포드 철학 백과사전.


댓글(0) 먼댓글(0) 좋아요(0)
좋아요
북마크하기찜하기
 
 
 

8

명제적 태도

 

외연성에 대한 再考

 

외연성 원리는 대체성 원리와 매우 밀접히 연관되어 있다(2, ‘대체성과 외연성참조). 이 원리는 다음과 같다:

 

() 복합문장의 일부로 나타나는 임의의 부분문장에 대해, 그 문장을 그와 동일한 진리치를 갖는 다른 문장으로 대체replace하여도, 복합문장 전체의 진리치는 변하지 않는다.

() 문장에 나타나는 임의의 술어에 대해, 그 술어를 그와 동일한 외연을 갖는 다른 술어로 대체하여도, 문장 전체의 진리치는 변하지 않는다.

() 문장에 나타나는 임의의 단칭용어에 대해, 그 단칭용어를 그와 동일한 지시를 갖는 다른 단칭용어로 대체하여도, 문장 전체의 진리치는 변하지 않는다.

 

외연성 원리는 매우 직관적이다. 단칭용어에 대해 말하고 있는 ()을 생각해보자: 특정 대상이 특정 술어를 만족한다satisfy는 것을 한 문장이 말하고 있고, 다른 문장 역시 동일한 대상이 동일한 술어를 만족한다 말하고 있다면, 전자가 참인 경우 후자 역시 분명 참이며 전자가 거짓인 경우 후자 역시 분명 거짓이다. 예를 들면 다음과 같다:

 

(1) 파리Paris는 비엔나보다 북쪽에 있다.

(2) ‘파리프랑스의 수도-지시적co-referential이다.

(3) 프랑스의 수도는 비엔나보다 북쪽에 있다.

 

(1)이 실제로 참이든 아니든, (2)에 따르면 (1)(3)은 동일한 진리치를 갖는다. 다음 사례에서 볼 수 있듯 술어에 대해 말하고 있는 () 역시 마찬가지이다:

 

(4) 이 금붕어는 췌장을 갖고 있다.

(5) ‘𝛼는 췌장을 갖고 있다‘𝛼는 비장을 갖고 있다-외연적co-extensive이다.

(6) 이 금붕어는 비장을 갖고 있다.

 

(5)가 참이라고 가정한다면, (4)(6)의 진리치는 반드시 동일해야만 한다(, 진리치 동일성은 의미의 동일성과 다르다는 점에 유의하라).

그런데 우리의 언어에는 -외연적인 표현 혹은 대체성이 성립하지 않는 표현이 존재한다. 가령 ()에 대한 가장 단순하고 명백한 반례(反例)counterexample때문에because라는 문장 연결사이다. 예를 들면 다음과 같다:

 

(7) 그 개가 짖었기 때문에 그 고양이는 가르랑거렸다.

(8) 파리는 프랑스의 수도이기 때문에 그 고양이는 가르랑거렸다.

 

(7)이 참이라고 가정해보자. 따라서 각 부분문장인 그 고양이는 가르랑거렸다그 개는 짖었다는 참이다. 하지만 (7)에서 그 개는 짖었다를 그와 마찬가지로 참인 파리는 프랑스의 수도이다로 대체한 (8)은 거짓이다.

 

 

지시적 불투명성, 그리고 태도에 대한 Frege의 견해

 

이제 다음 문장을 보자:

 

(9) ‘금성은 두 글자로 이루어져 있다.

 

금성개밥바라기는 동일한 대상 금성을 지시하는 각기 다른 이름이다. 하지만 이 사실로부터 다음을 추론하는 것은 타당하지 않다:

 

(10) ‘개밥바라기는 두 글자로 이루어져 있다.

 

(9)와 달리 (10)은 거짓이다. Frege의 설명에 따르면, (9)의 초입에 나타나는 단칭용어는 행성 금성을 지시하는 게 아니라 언어표현 금성을 지시한다. 즉 그것은 행성 금성의 이름이 아니라 금성의 이름의 이름인바, “‘금성’”은 금성이 아니라 금성을 지시한다.1) (9)에서 “‘금성’”이 나타나는 이러한 자리는 지시적으로 투명referentially transparent하지 않고 지시적으로 불투명하다referentially opaque. 이름 금성이 이렇게 지시적으로 불투명한 자리에 삽입되면, 금성을 지시한다는 통상적인 역할[즉 지시적으로 투명한 자리에서 수행했을 역할]을 수행하지 않는다. 다음과 같은 맥락

 

(11) ‘ 은 두 글자로 이루어져 있다.

 

에서 -외연적 표현들 간의 대체가 허용되지 않는 것은 바로 지시적 불투명성에 기인한다. 즉 이러한 맥락은 -외연적non-extensive이다. (9)와 같은 맥락에서 나타나는 금성은 금성을 지시하지 않는다. 어떤 언어표현이 (11)과 같은 맥락의 공란에 삽입되면, 인용부호quotation marks와 결합됨으로써 [그 표현의 지시체에 대한 이름이 형성되는 게 아니라] 그 표현 자체를 지시하는 이름이 형성되며, 문장 전체는 [그 표현의 통상적인 지시체에 대해서가 아니라] 그 표현의 이름에 관해 무언가를 말하게 된다.


1) 본문에서 인용부호가 두 번 사용되었다는 점에 유의하라. (9)는 금성의 이름인 금성사용하고 있는 반면, 본문은 (9)에서 사용된 그 이름을 언급하고 있기 때문이다. ‘금성은 금성을 지시하는 이름이고 “‘금성’”은 금성을 지시하는 이름인 금성을 지시하는 이름이다. 전자는 세계에 존재하는 대상을 지시하는 언어표현이고 후자는 언어표현 자체를 지시하는 언어표현이다.


하지만 (11)과 미묘하게 다른 다음과 같은 맥락은 지시적으로 투명하다:

 

(12) 은 두 글자로 이루어져 있다.

혹은 다음 맥락 역시 마찬가지이다:

 

(13) 𝛼는 두 글자로 이루어져 있다.2)


2) 가령 다음을 보자: 


  (a) ‘미당은 두 글자로 이루어져 있다

  (b) ‘미당’ = 서정주의 호 

  (c) 서정주의 호는 두 글자로 이루어져 있다


(a)는 참이다. 그리고 미당은 미당 즉 시인 서정주를 지시하는 반면, “‘미당’”은 서정주의 호인 미당을 지시한다. 따라서 (b)는 참이다. 이제 (a)에서 인용부호가 포함된 표현 “‘미당’”을 그와 -지시적인 서정주의 호로 대체한 (c) 역시 참이다.

 

Frege는 지시적 불투명성이 (11)과 같은 직접인용 맥락에서뿐만 아니라, 간접인용indirect quotation 맥락 및 믿음과 같은 명제적 태도와 연관된 표현에서도 드러난다고 지적하였다. 2에서 살펴보았던 관점을 잠시 되살려보자. 먼저 다음과 같은 간접인용 맥락의 사례를 생각해보자:

 

(14) Adam은 금성이 행성이라고 말했다said that Venus is a planet.

(15) Adam은 개밥바라기가 행성이라고 말하지 않았다.

 

그런데 다음은 참이다:

 

(16) 개밥바라기 = 금성.

 

하지만 (16)이 참임에도, 만약 Adam개밥바라기에 대해 일절 들어본 바가 없어 (16)이 참임을 모른다면, 그 경우 (14)(15)는 분명 동시에 참일 수 있다. 심지어 Adam(16)이 참임을 알더라도, 어쨌든 그가 (14)에서 기술된 대로 말했다 해서 개밥바라기가 행성이라고 말했다 할 수는 없다. 이러한 -외연적 맥락은 보통 -내포적 맥락hyper-intensional context이라 칭해진다. 그런 명칭이 붙게 된 이유는 잠시 뒤에 살펴보게 될 것이다.

명제적 태도가 나타내어지는 다음과 같은 맥락 역시 -내포적이다:

 

(17) Adam은 금성이 행성이라고 믿는다believe that.

(18) Adam은 개밥바라기가 행성이라고 믿지 않는다.

 

이 두 문장을 참이게 하면서 Adam이 합리적이라고 간주할 수 있으려면 우리는 Adam(16)이 참임을 모른다고 가정해야 한다. 그래야만 (17)(18)이 동시에 참일 수 있게 된다. 이 대목에서 Frege가 제시하는 설명에 따르면

 

(19) 금성이 행성이라는 것that Venus is a planet

 

이라는 일련의 표현은 [명제를 표현하는] 온전한 문장이 아니라, ‘금성은 행성이다라는 문장의 뜻 즉 그 문장이 표현하는 명제를 지칭하는 단칭용어이다. ‘라는 것that이라는 단어가 온전한 문장에 결합되어 형성된 일련의 명사절 전체는 결합된 문장의 뜻을 지시하는 단칭용어로 전환된다. (19)가 지칭하는 뜻은 다음 명사절

 

(20) 개밥바라기가 행성이라는 것

 

이 지칭하는 뜻과 다르다. 왜냐하면 [Frege의 뜻-지시 구분에 따르면 금성개밥바라기-지시적이긴 해도 그 뜻을 달리하기에, 그것들이 나타나는] ‘금성은 행성이다개밥바라기는 행성이다가 각기 다른 명제 즉 각기 다른 뜻을 표현하기 때문이다. 따라서 다음과 같은 맥락

 

(21) Adamα가 행성이라고 믿는다.

 

는 지시적으로 불투명하다. 다르게 말하면 (21)에 단칭용어가 삽입됨으로써 형성된 문장 전체의 진리치는 그 단칭용어에 의해 지시되는 대상에 의해 결정되지 않는다. Frege에 따르면 그러한 문장의 진리치는 삽입되는 단칭용어의 뜻에 의해 결정된다. 이를 일반화하자면, 임의의 문장 S에 대해

 

(22) AdamS라고 믿는다believe that S.

 

라는 맥락은 지시적으로 불투명하다. 이러한 맥락은 -내포적이다. 물론 Adam에게 특별한 초능력이 있어서 (22)-내포적인 것은 아니다. 좀 더 정확하게 일반화하자면, 임의의 믿음-주체 α와 임의의 문장 S에 대해

 

(23) αS라고 믿는다.

 

라는 맥락은 S를 대체하는 문장의 측면에서 지시적으로 불투명하다. 하지만 직접인용의 경우에서와 마찬가지로, 이와 미묘하게 다른 다음 맥락

 

(24) αβ를 믿는다.

 

는 통상적이고 외연적인 관계를 나타내는 단순한 2항관계로서, 그 관계가 단지 믿음관계일 뿐이다. ((23)(24)(11) (12)와 비교해보라.)

따라서 Frege가 보기에 불투명 맥락 (23)(11)과 매우 유사하다. (11)의 공란에 지시적 용어가 채워지면 그 용어의 지시체에 관해서가 아니라 그 용어 자체에 관해 무언가를 말하는 문장이 형성된다. 삽입되는 용어 자체에 관한 문장이 얻어지지 않는다는 점만을 제외한다면 (23) 역시 이와 마찬가지로서, (23)의 각 공란이 채워지면 명제태도의 주체와 하나의 명제 즉 삽입된 문장의 에 관해 무언가를 말하는 문장이 형성된다.

뿐만 아니라 Frege에 따르면 (17)과 같은 문장은 금성 내지 개밥바라기라는 행성을 지시하는 표현을 포함하고 있지 않다. 그 문장은 Adam금성은 행성이다에 의해 표현되는 명제를 지시하며, 전자가 후자를 믿는다는 것을 말하고 있다. 비록 그 명제의 구성요소[금성의 뜻][외연적인 맥락에서라면] 금성을 결정하지만, 그 명제를 가리키는 단칭용어인 (19)에서는 그 어떤 단칭용어도 금성을 지시하지 않는다.

이러한 Frege의 관점은 우리의 직관에 부합한다. 왜냐하면 다음과 같은 문장이 주장되는 경우가 종종 있기 때문이다:

 

(25) Le VerrierVulcan이 수성보다 작다고 믿었다.

 

Vulcan은 실존하지 않지만 그래도 이 문장은 참이다. 따라서 믿음-연산자belief-operator가 포함된 문장의 진리치는, 그 연산자의 범위(영향권)scope 내에 있는 단칭용어가 통상적으로 가리키는 대상customary object[즉 믿음 연산자 외부에서 가리켰을 대상]을 지시하는지 여부와 무관하다. 오직 필요한 것은 명사절(라는 것-)that-clause(또는 문법학자들이 말하는 간접절indirect clause)이 한 명제를 짚어내는가 여부일 뿐이다. (25의 경우 이 요건은 다음과 같이 충족된다: ‘Vulcan’이 대상을 지시하는 데에 실패하기에 ‘Vulcan은 수성보다 작다는 진리치를 결여하지만, 이 문장은 그럼에도 하나의 뜻 즉 명제를 표현하며, (25)Le Verrier가 그 명제를 믿었음을 말하고 있다.

 

 

추가적인 논의: -내포적 맥락의 다중 삽입

 

고양이 한 마리가 매트 위에 있는 모습을 상상해보자. 이 사태를 CM이라 칭하자. 이제 특정 관점에서 CM을 그린 그림을 떠올려보라. 그 그림은 CM을 특정한 방식으로 묘사한다(여기서 방식은 명제의 뜻과 유사하다 [그림이 한 사태를 특정 관점에서 묘사하듯이, 명제 역시 하나의 사태를 특정 관점에서 제시한다. Frege적 뜻이란 지시체가 제시되는 방식 내지 지시-결정 규칙임을 상기할 것]). 이 그림을 P1이라 하자. 이제 P1을 또다시 그린 그림. CM의 그림에 대한 그림인 P2를 떠올려보라. P2P1을 특정한 방식으로 묘사하며 그에 따라 CM을 특정한 방식으로 묘사할 것이다. 이러한 절차는 P3, P4 등으로 계속 이어질 수 있으며 원래의 사태 CM을 묘사하는 방식들은 그에 따라 다층적multiple으로 겹치게 될 것이다. 하지만 P1에 포함된 정보는 그 과정에서 (모든 그림들이 매우 이상적이고 완전하게 그려졌다고 가정한다면) 원리적으로 소실되지 않는다. 모든 그림들의 연쇄에서 그 어떤 것을 골라잡아도, P1에 포함된 정보인 원래의 사태 CM을 찾아낼 수 있는 것이다.

이러한 구조는 Frege가 명제에 관한 명제, 혹은 명제에 관한 명제에 관한 명제 등에 대해 생각했던 방식과 매우 유사하다. Frege가 생각하기에 주어진 어떤 용어가 한 문장의 진리치를 결정하는 데에 기여contribution하는 방식은, 그 용어가 문장에서 내포적인 혹은 -내포적인 연산자의 영향권 내에 삽입되는지 여부에 따라 달라진다. 우선 외연적 맥락의 경우 문장의 진리치에 영향을 미치는 실체entity는 용어의 통상적인 지시체ordinary referent이다. 반면 -내포적인 맥락의 경우 문장의 진리치와 유관한 실체는 Frege가 칭한바 용어의 간접지시체“indirect” referent로서, 이는 용어의 통상적인 뜻ordinary sense[즉 외연적 맥락에서 표현되는 뜻] 이다. 다소 느슨하게 말하자면 용어의 지시는 맥락에 따라 전환shift되며, 지시가 전환되는 단계는 다음과 같이 더욱 높은 수준으로까지 이어질 수 있다:

 

(26) Don Ho가 가수라는 것을 Adam이 믿는다는 것을 Dudley는 믿는다.

Dudley believes that Adam believes that Don Ho is a singer.

 

(17) ‘Adam은 금성이 행성이라고 믿는다와 같은 경우를, 한 용어의 간접적 뜻indirect sense과 간접 지시를 갖는 것으로서 -내포적 삽입hyper-intensional embedding 유형-1이라 칭한다면, (26)2 간접적 뜻과 2차 간접 지시를 갖는 것으로서 -내포적 삽입 유형-2라 할 수 있다.

아래의 8.1-내포적 맥락이 이렇게 다중적으로 삽입되는 단계를 단칭용어 ‘Don Ho’를 예시로 삼아 나타내고 있다. 식별을 용이하게 하기 위해 꺾쇠괄호가 의 뜻을 나타낸다고 하자. 가령 Don Ho= ‘Don Ho’의 뜻이며, 〈〈Don Ho〉〉 = “‘Don Ho’의 뜻의 뜻이다. 따라서 Don Ho[외연적 맥락에서 직접 지시체로서] Don Ho를 결정하며, 〈〈Don Ho〉〉[-내포적 맥락에서 간접 지시체로서] Don Ho를 결정한다(그리고 〈〈〈Don Ho〉〉〉[2-내포적 맥락에서 2차 간접 지시체로서] 〈〈Don Ho〉〉를 결정한다). 화살표는 연결된 각 항목이 동일한 실체로서 반복되고 있음을 나타낸다:

 

8.1 세 층위 맥락에서 한 용어의 뜻과 지시

외연적 맥락

 

-내포적 맥락 유형-1

 

-내포적 맥락 유형-2

(통상적) 지시체

Don Ho

 

 

 

 

(통상적)

Don Ho

(간접) 지시체

Don Ho

 

 

 

 

(간접적)

〈〈Don Ho〉〉

(2차 간접) 지시체

〈〈Don Ho〉〉

 

 

 

 

(2차 간접적)

〈〈〈Don Ho〉〉〉

 

표에서 첫 번째 세로열은 ‘Don Ho는 가수다와 같은 보통의 외연적 맥락을 나타낸다. 이 맥락에서 용어 ‘Don Ho’는 통상적인 뜻 Don Ho를 표현하며, 이 뜻은 그 용어의 통상적 지시체로서 Don Ho를 결정한다(실제로 Don Ho는 히트곡 Tiny Bubbles를 부른 하와이 출신 가수였다). 두 번째 열은 ‘AdamDon Ho가 가수라고 믿는다와 같은 -내포적 맥락 유형-1을 나타낸다. 외연적 맥락에서 ‘Don Ho’의 통상적 이었던 Don Ho가 이 맥락에서는 그 용어의 간접 지시체가 된다. [그리고 이 간접 지시체를 결정하는 것은 간접적 뜻인 〈〈Don Ho〉〉이다.] 세 번째 열은 (26)과 같은 -내포적 맥락 유형-2를 나타내는바, 2차 내포맥락에서 간접적 뜻이었던 〈〈Don H〉〉가 이 맥락에서는 그 용어의 2차 간접 지시체가 된다. [그리고 이를 결정하는 것은 2차 간접적 뜻인 〈〈〈Don Ho〉〉〉이다.] 한 문장 내에서 라고 믿는다와 같은 명제태도 연산자가 무한히 반복되면서 그 영향권 내부에 동일한 용어가 나타날 수 있기 때문에, 한 용어가 -내포적 맥락에 다중적으로 삽입되는 이러한 절차는 원리상 무한히 계속될 수 있다. 따라서 ‘S라고 B가 믿는다는 것을 C가 믿는다는 것을 가 믿는다는 것을 A는 믿는다A believes that B believes that C 등과 같이 계속 이어질 수 있다. 이러한 과정을 이번 의 초입에 살펴보았듯 한 주제를 그린 그림을 그린 그림을 그린 그림처럼 생각해볼 수 있다. 여기서 최초 단계에 그려지는 주제는 통상적인 지시체에 해당할 것이다.

 

 

대물적 필연성과 대언적 필연성

 

한 가지 중요한 종류의 -외연적 맥락은 필연적(必然的)으로necessarily와 같은 양상적(樣相的) 부사modal adverb에 의해 조성된다. 앞서 우리는 이 주제를 간단히 살펴본 바 있다(5, ‘필연성, 가능성, 가능세계6, ‘자연종 용어와 본질에 관한 Putnam의 견해). 앞서와 마찬가지로 여기서도 수학적 진리가 필연적 진리라는, 즉 달리 될 수 없었던could not have been otherwise 진리라는 관점을 택하기로 한다. 이제 다음을 보자:

 

(27) 필연적으로, 23보다 작다.

Necessarily, two is less than three.

 

[수학적 진리는 필연적으로 참이므로] 이는 참이다. 그리고 화성은 PhobosDaimos라는 두 개의 위성을 거느리고 있다. 그러므로 단칭용어 화성의 위성의 개수‘2’는 동일한 지시를 갖는다. 하지만 이러한 사실에 근거하여 (27)로부터 다음을 추론할 수는 없다:

 

(28) 필연적으로, 화성의 위성의 개수는 3보다 작다.

Necessarily, the number of Martian moons is less than three.

 

물론 화성이 셋보다 적은 수의 위성을 지니고 있다는 것은 사실이다. 하지만 이는 단지 우연적(偶然的)인 사실contingent fact로서, 화성은 지금보다 더 많은 수의 위성을 거느리고 있었을 수도 있다. 그렇다면 필연적으로라는 표현은 -외연적인 맥락을 만들어내는 셈이다. [(27)의 일부를 그와 -지시적인 용어로 대체하였는데도 진리치가 유지되지 않기 때문이다.]

한 가지 유의할 사항은, ‘필연적으로와 같은 양상적 부사가 조성하는 맥락이 -내포적이지는 않고 [단순] 내포적이라는 점이다. 이러한 전문적인 구분을 굳이 도입하는 이유는, 양상적 부사가 작동하는 방식이 명제적 태도를 나타내는 동사의 작동방식만큼 까다롭고 섬세하지는 않기 때문이라 할 수 있다. 예시를 통해 양자를 비교해보자: 모든 캥거루가 캥거루라는 것은 필연적으로 참이다. [즉 가능세계 해석을 따르자면 모든 캥거루는 캥거루이다는 모든 가능세계에서 참이다.] 따라서 Samuel Beckett1906년에 태어난 가능세계들과, Beckett1906년에 태어났고 모든 캥거루가 캥거루인 가능세계들은 정확히 동일한 세계들이다. [전자가 참인 모든 세계들에서는 후자 역시 참이기 때문이다.] 그런데 어떤 사람이 Beckett1906년에 태어났는지를 궁금해 한다고 해서 모든 캥거루가 캥거루인지 여부까지 궁금해 하지는 않을 것이다. [요컨대 어떤 두 문장의 양상적 진리치가 일치하는 세계가 동일하다고 해도, 양자에 대한 명제적 태도는 분명 다를 수 있다.] 문장들 간에 나타나는 의미론적 차이가 문장들이 지닌 내포적 진리치intensional truth-value간의 차이에 반영되지는 않는 것이다. [다르게 말하면 단순 내포적 맥락에서의 진리치 동일성이 -내포적 맥락에서의 의미 동일성을 함축하는 것은 아니다.]3)


3) 단순 내포적 맥락과 -내포적 맥락 간의 차이를 요약해보자면 다음과 같다: 진리치 보존적인 대체가 허용되기 위해 전자에서는 양상적으로 동일한 지시(즉 진리치외연지시체)를 갖는 표현들로 대체되어야 하고, 후자에서는 동일한 의미(Frege적인 뜻)를 갖는 표현들로 대체되어야 한다. (반면 외연적 맥락에서는 현실세계에서 동일한 지시를 갖는 표현들로 대체되어야 한다.) 


이렇듯 (28)은 거짓이다. 그런데 이번엔 다음 문장을 보자:

 

(29) 2는 필연적으로 그것3보다 작은 그러한 것이다.

Two is such that necessarily it is less than three.

 

Quine(그에 대해서는 10에서 더욱 자세히 살펴볼 것이다)(27)에서 (29)로의 이행을 수출exportation이라 칭한다. (29)에서 대명사 그것의 지시는 앞서 나타난 ‘2’에 의해 공급되지만, ‘2’와 다르게 그것은 내포적 부사 필연적으로의 범위 내부에 나타난다. (27)(29)는 각각 대언적(對言的)de dicto 양상진술(말해진 것에 관해of things said 양상성을 귀속시키는 진술)대물적(對物的)de re 양상진술(사물에 관해of things 양상성을 귀속시키는 진술) 간의 구분을 예시해준다.4)5)


4) ) “라틴어 어원적으로 ‘dicto’는 말 내지 언어를 뜻하며 ‘re’는 대상 내지 사물을 뜻하므로, 대언적 필연성을 언어적 혹은 명제적 필연성이라 부르고, 대물적 필연성을 존재적 혹은 대상적 필연성이라 부를 수 있을 것이다.” (김영정, 언어, 논리, 존재: 언어철학, 논리철학 입문, 철학과현실사, 1999, 45.)

) “대언적 필연성de dicto necessity 개념은 전통철학에서든 현대철학에서든 한결같이 중요하다고 인정된 구분인 대언적 필연성과 대물적 필연성de re necessity 간의 구분을 환기한다. 전자는 명제가 지닌 속성으로서의 필연성 개념으로서, 명제의 술어로 붙일 수 있는 속성 중에는 양상적 속성 즉 필연적으로 참이 됨이라는 속성이 있다는 것이다. 후자는 대상이 지닌 속성으로서의 필연성 개념으로서, 대상이 어떤 본질적 혹은 필연적 속성을 가진다는 생각이다. 명제 내에서 지시된 대상이 여차여차한 속성을 본질적필연적으로 지닐 때, 그 명제는 대물적 양상성을 표현한다. (中略) 대언적/대물적 양상 간의 구분은 다음과 같이 요약될 수 있다: 대언적 양상을 표현하는 명제는 어떤 다른 명제가 양상적 진리치를 갖는다고 기술한다. 반면 대물작 양상을 표현하는 명제는 어떤 대상이 어떤 속성을 필연적본질적으로 예화한다고 기술한다.” (Anthony Clifford Grayling, 철학적 논리학 입문An Introduction to Pilosophical Logic(3, 1997), 이윤일 , 선학사, 94-5, 볼드체는 원저자의 것.)

* “대언성/대물성 구분을 명확히 보여주기 위해 이따금 Thomas Aquinas가 들었던 예가 사용된다. Aquinas-이교도 大典에서 인간 의지의 자유가 신의 예지(豫知)와 모순되는지 여부를 고찰하면서 이 구분을 도입한다. 신이 시점 t2에 앉아있는 Aristoteles를 그보다 이전 시점인 t1에 본다고 해보자. 이러한 가정 하에서 앉아있는 것으로 보이는 것은 필연적으로 앉아있다가 참이라고 할 경우, Aristoteles는 시점 t2에 반드시 앉아있을 수밖에는 없는 것처럼 여겨진다. [즉 신의 예지를 가정한다면 결정론이 참인 것처럼 보인다. 이러한 결론이 도출되는 것을 막기 위해] Aquinas는 여기서 대물성-대언성 구분을 사용한다. 위 문장이 대언적 양상성을 표현하는 것으로 해석된다면, 앉아있는 것으로 보이는 것이 그 무엇이 되었든 그것이 앉아있다는 것은 필연적으로 참이다로 해석된다면, 이는 참이다. [‘은 필연적으로 참이다를 제외한 명사절로 취해진 문장은 임의의 x에 대해, xF하다면 xF하다[(x)(FxFx)]’ 형식으로서 논리적으로 참이며, 논리적으로 참인 문장은 필연적으로 참이기 때문이다(여기서 αF한 것으로 보인다αF하다가 동일한 술어로 간주되는 것은 신의 전지성에 근거한다).] 반면 위 문장이 앉아있는 것으로 보이는 것이 그 무엇이 되었든 그것은 필연적본질적으로 앉아있다는 속성을 갖는다로 해석된다면, 이는 거짓이다. Aquinas에 따르면 결정론을 옹호하는 논증은 위 문장에 대한 대물적 해석이 참인 경우에만 타당하다. 따라서 결정론이 신의 예지로부터 자동적으로 따라나온다는 논제는 거짓이다.” (A. C. Grayling, 같은 책, 95, 내용 일부 수정.) 

5) 자연언어로 표기된 양상문에서는 필연성 부사가 취하는 영향권이 명확하지 않아 대언적/대물적 양상성 구분이 모호하다. 양화논리에 두 가지 양상 연산자 필연적으로[]’가능적으로[]’가 도입된 양화양상논리의 문장을 비교해보면 대언적/대물적 양상성이 더욱 명료하게 구분될 수 있다.

우선 세 개의 가능세계 W1, W2, W3가 존재하고, 각 세계 내에는 빌딩 a, b, c라는 세 개체들만이 존재한다고 가정하자. 그리고 개체/술어상항에 대한 해석함수 I를 활용하여 다음과 같이 간략화된 해석을 가정하자:


() I(a) = a, I(b) = b, I(c) = c.

() I(H) = {W1|a}, {W2|b}, {W3|c}.

 

()는 모든 가능세계에서 개체상항 ‘a’, ‘b’, ‘c’에 대해 지시체를 할당하고 있고, ()는 술어상항 ‘H’에 대해 각 가능세계에서 그 술어를 만족하는 개체상항을 명시하고 있다. 직관적인 이해를 위해 술어 ‘Hα를 자연언어에서 α는 가장 높은 빌딩이다에 해당한다고 하자. 따라서 위 해석에 따르면 가령 W1에서 1항 원자문장 ‘Ha’는 참이지만 ‘Hb’‘Hc’는 거짓이다. 마지막으로 보편/존재 양화사는 緖論에서 살펴본 대로 통상적인 양화논리의 의미론에 따라 해석하고, 양상성 연산자 5에서 살펴본 다소 간략한 가능세계 의미론에 따라 각각 는 모든 가능세계에서 참이다는 어떤 가능세계에서/적어도 하나의 가능세계에서 참이다로 해석한다.

이제 다음 두 문장을 보자:

 

() (x)Hx.

() (x)Hx.

 

채택된 해석에 따라 각 문장의 진리조건을 명시하면 다음과 같다: 먼저 ()의 경우 에 대한 해석에 따라 존재 양화문 ‘(x)Hx’가 모든 가능세계에서 참이어야 한다. ‘(x)Hx’가 모든 가능세계에서 참이기 위해서는 존재 양화사의 해석에 따라 개방문 ‘Hx’를 만족하는 개체가 각 가능세계에 적어도 하나씩 존재해야 한다. 해석 ()에 따르면 ‘Hx’를 만족하는 개체가 각 가능세계마다 적어도 하나씩 존재하므로(즉 각 가능세계마다 가장 높은 빌딩들이 각기 존재하므로), 최종적으로 이 해석 하에서 ()는 참이다.

그 다음 ()의 경우 존재 양화사의 해석에 따라 개방문 Hx’를 만족하는 개체가 현실세계에 적어도 하나 존재해야 한다. 편의를 위해 W1를 현실세계라 한다면, W1에 존재하는 세 빌딩 중 적어도 하나가 Hx’를 만족해야 하는 셈이다. 이를 위해서는 의 해석에 따라 모든 가능세계에서 ‘Hx’를 만족하는 개체가 W1에 적어도 하나 존재해야 한다. 모든 가능세계에서 ‘Hx’를 만족하는 그러한 개체가 W1에 존재하지 않으므로(즉 모든 가능세계를 통틀어 가장 높은 동일한 하나의 빌딩이란 W1에 존재하지 않으므로), 최종적으로 이 해석 하에서 ()는 거짓이다.

두 문장의 진리조건에서 짐작할 수 있듯이 ()()에서는 각각 대언적/대물적 양상성이 드러난다. 전자는 ‘(x)Hx’라는 문장 내지 명제에 대해 필연적으로 참이다라는 대언적 양상성을 귀속시키고 있고, 후자는 세계 내의 개체와 속성에 대해 ‘x는 필연적으로 H하다라는 대물적 양상성을 귀속시키고 있다. 일반적으로 양상성 연산자가 양화사보다 넓은 범위를 취할 경우 대언적 양상성이 표현되고, 그 반대일 경우 대물적 양상성이 표현된다. 자연언어로 양상문을 표기할 경우 양화사가 없더라도 양화양상논리 형식문의 이러한 구문론적 특성과 대응되도록 대언적 문장은 필연적으로, aF하다, 대물적 문장은 ‘a는 필연적으로 F하다(필연적으로 F한 그러한 것이다)’와 같은 식으로 통일하였다



다시 한 번 말하지만 (29)에서 용어 ‘2’는 필연성 연산자의 영향권 외부에outside the scope 있다. (27)에서 내포적 자리에 있던 ‘2’(29)에서는 [Quine식 수출작용에 의해] 외연적 자리로 옮겨진 것이다. [다르게 말하면 -지시적인 표현에 의해 진리치 보존적으로 대체될 수 있는 자리로 옮겨졌다.] 그리고 2 = 화성의 위성의 개수이므로, (29)로부터 다음을 추론할 수 있다:

 

(30) 화성의 위성의 개수는 필연적으로 3보다 작은 그러한 것이다.

The number of Martian moons is such that necessarily it is less than three.

 

(28)과는 달리, 그리고 (29)와 마찬가지로, 이 문장은 참이다. 이 문장이 말하고 있는 바는, 화성이 거느린 위성의 개수(혹은 Charles 왕자의 귀의 개수와 같이 원하는 그 무엇이라 칭하든)2를 고려하건대, 그 대상이 필연적으로 3보다 작다는 것이기 때문이다. 이렇듯 수출작업은 한 용어가 지닌 개념적 내용conceptual content, 그 용어가 지시하는바 개념적 내용이 제거된 대상 그 자체naked object를 갈라낼 수 있게 해준다.

 

 

대물적 믿음과 대언적 믿음

 

양상 맥락과 같은 단순 내포적 맥락에서 대언성/대물성이 구분되었던 것과 동일하게, 명제적 태도 맥락과 같은 -내포적 맥락의 측면에서도 양자가 구분될 수 있다. 이를 설명 및 예시하기 위해 명제적 태도 진술과 관련하여 Quine이 들었던 유명한 사례를 생각해보자: Ralph는 갈색 모자를 쓴 특정 남자가 수상쩍게 행동하는 것을 여러 차례 목격했다. 한정 기술구 갈색 모자를 쓴 그 남자the man in the brown hat을 축약하여 MBHthe MBH라 하자. 이에 다음은 참이다:

 

(31) Ralph는 그 MBH가 간첩이라고 믿는다.

Ralph believes that the MBH is a spy.

 

이로부터 다음이 얻어진다고 생각해볼 수 있다:

 

(32) MBHRalph가 그를 간첩이라고 믿는 그러한 것이다.

The MBH is such that Ralph believes that he is a spy.

 

(31)로부터 (32)로의 이행은 (27)로부터 (29)로의 추론과 정확히 동일해 보인다. [앞 사례에서 화성의 위성의 개수필연적으로의 범위 내부에서 외부로 이동했던 것과 마찬가지로,] 이번 사례의 경우엔 MBH’믿는다의 범위 내부에서 외부로 이동함으로써, 대언적 믿음 진술로부터 대물적 믿음 진술이 얻어진 것이다. 그러나 일견 문제될 것이 없어 보이는 대언성에서 대물성으로의 이러한 이행은 인식론적으로 중대한 차이점을 야기한다. 이를 살펴보기 위해, 대물적 문장인 (32)를 다음과 같이 존재 양화existential quantification[존재 일반화existential generalization]해보자:

 

(33) 그러한 x가 존재한다 (Ralphx가 간첩이라고 믿는다).

There is an x such that (Ralph believes that x is a spy).

 

그런데 (33)Ralph의 인지적 상태에 대해, 다음과 같이 단순한 대언적 믿음 문장인 (32)에 비해 더욱 흥미로운 무언가를 드러내고 있다:

 

(34) Ralph는 그러한 x가 존재한다고 믿는다 (x는 간첩이다).

Ralph believes that there is an x such that (x is a spy).6)

 

6) 임의의 개체상항 ‘s’와 임의의 폐쇄문 ‘P’에 대해, 믿음 연산자 ‘sP라고 믿는다‘Bs[P]’로 기호화한 뒤, 각주)에서 살펴본 방식대로 (33)(34)를 기호화하자면 다음과 같이 표기될 수 있다:

 

(33) (x)Br[Sx].

(34) Br[(x)Sx].

 

앞서 양상성 문맥에서와 마찬가지로, 존재 양화사가 넓은 범위를 취하고 있는 (33)에서는 대물적 믿음이 드러나고 있고, 믿음-연산자가 넓은 범위를 취하고 있는 (34)에서는 대언적 믿음이 드러나고 있다. 전자는 어떤 대상이 존재해서, 그 대상이 Ralph에 의해 여차여차하다고 믿어진다는 것을 말하고 있는 반면, 후자는 Ralph가 어떤 대상이 여차여차하다고 믿고 있음을 말하고 있다


우선 (34)에 따르면 Ralph는 세상에 간첩이 존재한다고 믿고 있는 셈이다. 그러나 이는 세상 사람들 누구에게나 다 해당되는바, 특별히 Ralph의 인지적 상태에 대해 별다른 무언가를 말하고 있지는 않은 셈이다. 반면 (33)(34)와 미묘하게 다른 점은, 대부분 사람들과는 다르게 Ralph가 특정한 누군가를 간첩이라 의심하고 있음을 말하고 있다는 점이다. [양상성이 나타나는 단순 내포적 맥락에서와 달리, 이렇듯 명제태도가 나타나는 -내포적 맥락에서는 수출이 가해지기 이전과 이후에 인식론적 측면에서 모종의 비대칭성이 발생하는 것이다.]

그 자체로 보자면 (32)에서 (33)으로의 이행[즉 존재 양화]은 명백히 타당하다. (32)에서 양화되는quantified into 자리인 MBH’가 나타나는 맥락은, 단칭용어가 온전히 통상적으로 나타나는 자리 즉 지시적으로 투명한 자리이기 때문이다. (32)에서 'MBH'라고 믿는다에 의해 속박bind되어 있거나 그 연산자의 영향권 내부에 있는 게 아니라 외부에서 나타나고 있다(그리고 존재양화가 이뤄지면서 (33)에서 두 번째로 나타나는 변항 ‘x’ [즉 양화사를 제외한 부속문 ‘Ralphx가 간첩이라고 믿는다‘x’](32)에서 나타나는 단어 he’ [‘Ralph가 그를 간첩이라고 믿는’]를 대체한 셈이다. (32)에서처럼 사용되는 경우 대명사 는 변항과 동일한 기능을 수행한다).

하지만 Frege적인 관점에서 보자면 (31)에서 (32)로의 추론부터가 애초에 타당하지 않았던 것처럼 보인다. 앞서 태도에 관한 Frege의 견해와 지시적 불투명성말미에서 살펴보았듯, 믿음 맥락 내부에 있는 용어는 그것의 통상적인 지시체가 아니라 통상적인 뜻을 지시하기 때문이다. 그렇기에 (31)Ralph가 그 MBH라는 대상과 어떤 관계를 맺고 있음을 진술하고 있는 게 아니라, 어떤 명제와 관계를 맺고 있음을 진술하고 있다. 기실 (31)은 그러한 남자가 존재한다는 것조차도 함축하지 않는다. [(31)에서 단칭용어 MBH’-외연적인 맥락에 나타나는바 그 지시체에 대한 존재함축이 가정되어있지 않기 때문이다.] 이는 Le VerrierVulcan의 사례에서살펴본 바와 같다. [Le VerrierVulcan이 뜨거울 것이라 믿었다 해서, 그 믿음이 Vulcan의 존재를 보증해주지는 않는다.] 따라서 (31)은 그 MBH의 존재를 함축하는 (32)를 함축할 수 없다. (32)와 그에 따라 (33)은 특정한 한 남자 즉 그 MBH의 존재로 인해 참인 반면, (31)은 엄밀히 말해 그 MBH에 대해 말하고 있지 않은 문장이다. 이러한 관점에 따르면 (31)과 같이 지시적으로 불투명한 맥락은 양화사와 같은 장치에 대해 봉쇄되어있는 듯하다. 무언가 추가적인 도움이 없는 한, 지시적으로 불투명한 맥락 외부에 있는 양화사는 그 맥락 내부의 자리를 양화할quantify into 수 없는 것이다. (31)을 둘러싼 이러한 의미론적 여건을 그림에 대한 태도에 빗대어 이해해볼 수 있겠다: Ralph가 어떤 그림이 실제 사건을 묘사한 것이라 여긴다 해서, 그 그림에 묘사된 사건이 실제로 발생했다거나 그려진 사물들이 실재한다는 것을 함축하지는 않는다.

(31)에서 (32)를 추론했던 절차에는 한 가지 더 추가적인 가정이 숨어있는 듯하다. 바로 MBH’가 대상을 결정하는 뜻을 표현한다는 것, 간단히 말해 무언가를 지시한다는 것이 전제되어 있었다. 이러한 가정을 다소 간략히 표현하자면 다음과 같다:

 

그러한 x가 존재한다: ‘MBH’의 뜻은 x를 결정한다.

 

 

궁지에 몰린 Ralph

 

QuineRalph 이야기를 이어간다. RalphBernard J. Ortcutt라는 사람이 매우 정직한 사회구성원으로서 확실히 간첩이 아니라 믿고 있다. 따라서 다음은 참이다:

 

(35) RalphOrtcutt가 간첩이 아니라고 믿는다.

 

에서 그 MBH가 존재한다는 것을 근거로 (31)로부터 (32)를 추론하고 그에 따라 (33)을 추론했던 것과 마찬가지로, Ortcutt가 존재함을 근거로 (35)로부터 다음을 추론할 수 있다:

 

(36) 그러한 x가 존재한다(Ralphx가 간첩이라고 믿는다).

 

그런데 Ralph는 모르지만 다음은 참이다:

 

(37) MBH = Ortcutt.

 

앞선 (33) ‘그러한 x가 존재한다 (Ralphx가 간첩이라고 믿는다)’에 따르면 Ralph는 어떤 사람에 대해 그가 간첩이라 믿고 있다. 그 사람은 그 MBH인데, (33)에 따르면 이는 곧 Ortcutt와 동일인이다. 따라서 Ortcutt에 관해 Ralph는 그가 간첩이라 믿고 있는 셈이다. 하지만 이와 정확히 평행하게 진행되는 추론에 의해서, Ortcutt에 관해 Ralph는 그가 간첩이 아니라 믿고 있다는 것이 따라나온다. 이 남자에 대한 Ralph의 태도는 둘 중 과연 어느 것인가? Ralph의 입장에서 보자면 이는 그가 그 남자를 어떻게 생각하고 있느냐에 달렸다. Ralph가 그 남자를 그 MBH로 여기는 한 그 남자를 간첩이라 믿는 것인 반면, 그 남자를 Ortcutt로 여기는 한 그 남자를 간첩이 아니라 믿는 것이다. 하지만 우리가 보기엔 어떠한가? 분명 그 남자에 대한 Ralph의 태도는 우리가 그 남자를 생각하는 방식과 무관하다. 도대체 Ortcutt에 관해 Ralph는 그가 간첩이라 믿고 있는 것인가, 간첩이 아니라 믿고 있는 것인가?

샛별 사례에 대한 Frege의 가르침에 따르자면, Ralph의 태도는 둘 다라고 할 수 있다. 그렇다고 해서 Ralph일관적inconsistent이거나 합리적irrational인 사람이 되는 것은 아니다. 즉 하나의 명제 P에 대해 RalphP라고 믿고 있는 동시에 P가 아니라 믿고 있는 것은 아니다. Frege의 착상에 따라 말해보자면, (31) ‘Ralph는 그 MBH가 간첩이라고 믿는다에서 믿는다의 범위 내부에 나타나는 용어에는, RalphOrtcutt에 관해 생각하는 방식, Ralph가 그 남자를 MBH로서 생각하고 있다는 사실이 반영되어야 한다. 이는 (35)에서도 마찬가지로서, 다만 (35)에서는 Ralph가 그 남자를 Ortcutt로서 생각하고 있다는 점만이 다를 뿐이다. (31)(35)에서 각각 MBH’‘Ortcutt’가 나타나는 자리는 지시적으로 불투명한바 -지시적 용어에 의한 대체가 허용되지 않는다. ‘Ralph 가 간첩이라고 믿는다라는 맥락에 대체성 원리가 적용될 수 있는지 여부는, 공란에 삽입되는 용어가 지시하는 대상이 아닌 다른 무언가에 따라 결정된다. Frege에 따르면 그러한 맥락을 지닌 문장은 그 대상에 관해 말하고 있지 않다.

반면 (32) ‘MBHRalph가 그를 간첩이라고 믿는 그러한 것이다에서 나타나는 MBH’는 이러한 기능을 수행하지 않는다. [(32)에서 MBH’에는 Ralph가 그 남자에 관해 생각하는 방식이 결부되어 있지 않다.] (32)에서 MBH’가 차지하는 자리는 명백히 지시적으로 투명한바 -지시적 단칭용어(일테면 ‘Ortcutt’)에 의한 대체가 허용된다. (31)로부터 (32)로의 수출작용은 대언적 문장으로부터 대물적 문장으로 이행하는 절차라 할 수 있다. 한편, 존재 양화문인 (33) ‘그러한 x가 존재한다 (Ralphx가 간첩이라고 믿는다)’는 술어를 만족하는 특정 대상이 존재한다는 것을 명시적으로 말하고 있다.

 

대물성/대언성 구분의 인식론적 중요성은 매우 여러 가지가 있지만, 가장 두드러진 것으로 다음 두 가지를 들 수 있다.

 

() 대물성의 필수불가결성. Rusell나는 네 요트가 그보다 훨씬 길다longer than it is고 생각했어라는 예시를 든 적이 있다. 여기서 말해지고 있는 그 요트를 A라 해보자. Russell이 과거 특정 시점에 자신이 지녔던 믿음에 대해 말하는 바가 분명 다음과 같은 것은 아니다:

 

(38) BertranAA보다 길다고 믿는다.

 

그는 이와 같은 명백한 모순을 믿었다고 주장하고 있지 않다. 우리는 그의 말을 BertrandA의 길이가 A의 길이보다 길다고 생각했다는 식으로 해석해서는 안 된다. 직관적으로 생각해보자면 그가 의도했던 의미는, A의 실제 길이가 존재하는데, Bertrand[자신이 생각한] A의 길이가 그것보다 길다고 믿었다는 것이다. 이러한 의미를 나타내기 위해서는 A의 실제 길이를 지시하는 표현을 다음과 같이 라고 믿는다의 범위 내부로부터 그 외부로 수출시켜야 한다:

 

(39) 그러한 xy가 존재한다 (x = A의 길이 yx보다 길다 BertrandyA의 길이라고 믿는다).

 

다시 말해 Bertrand가 착각한 내용을 정확하게 기술하기 위해서는 믿는다의 범위 외부로부터 그 내부로 양화할 필요가 있는 셈이다. 이렇듯 경우에 따라서는 대물적 문장이 반드시 요구된다.

 

() 수출작용에 대한 추가적인 제한사항. Ralph의 친구 Leo에게는 Ralph와 달리 특정한 누군가가 간첩이라고 믿을 이유가 전혀 없다고 해보자. 다만 매우 일반적인 통념에 따라 Leo는 키가 가장 작은 두 간첩의 키가 정확히 동일하지는 않다고 믿는다. 그의 생각이 맞다고 해보자. 따라서 우리는 다음을 받아들여야 한다:

 

(40) Leo는 키가 가장 작은 간첩이 간첩이라고 믿는다.

 

키가 가장 작은 간첩은 존재하기 때문에, (40)에 수출작용을 가하여 다음을 얻을 수 있다:

 

(41) 키가 가장 작은 간첩은 Leo가 그를 간첩이라 믿는 그러한 것이다.

The shortest spy is such that Leo believes that he is a spy.

 

여기서 키가 가장 작은 간첩을 존재 양화하여 다음을 도출한다:

 

(42) 그러한 x가 존재한다 (Leox가 간첩이라 믿는다).

 

하지만 어딘가 잘못된 듯하다. (42)에 따르면 Leo는 특정 누군가가 간첩이라 믿고 있는 셈인데, 앞선 가정에 따르면 이는 사실이 아니기 때문이다. 다시 한 번 강조하지만 (41)로부터 (42)를 도출하는 과정에는 아무런 문제가 없다. 이는 단지 ‘B는 여차저차하다로부터 여차저차한 어떤 것이 존재한다를 도출하는 형식을 지니고 있을 뿐이다.

그렇다면 문제는 (40)으로부터 (41)을 얻어내는 수출절차에 있을 수밖에 없다. 단순히 대언적인 믿음을 갖는 것만으로는 그에 상응하는 대물적 믿음을 갖는 데에 충분하지 않다. 다시 말해 지시적이거나 기술적descriptive인 단칭용어가 포함된 한 명제에 대한 믿음을 갖는다고 해서, 그 단칭용어에 의해 지시되는 사물에 관한 믿음 역시 갖고 있다는 게 반드시 보증되지는 않는다. 그 특정 대상과 연관된 단칭명제에 대한 믿음을 갖는 것만으로는 충분하지 않은 것이다. 따라서 대언적 문장으로부터 대물적 문장으로 이행하는 수출작용에는 지금까지 고찰해온 것 이상의 무언가가 더 요구된다.

직관적으로 생각해보면 Leo와 키가 가장 작은 간첩 사례에서 결정적인 문제점은 키가 가장 작은 바로 그 간첩이 누구인지Leo모른다는 점에 기인하는 듯하다. 따라서 만약 키가 가장 작은 간첩이 누구인지를 Leo가 안다는 전제가 추가된다면 (41)에는 문제가 없게 된다.

David Kaplan은 이러한 착상을 더욱 정교하게 다듬고 발전시켰는데, 여기서는 그 핵심만 간략히 살펴보고자 한다. Kaplan에 따르면 수출작용을 통해 대언성에서 대물성으로 이행하는 추론이 성공적이기 위해서는, 수출되는 용어(이 사례의 경우 (40)키가 가장 작은 간첩’)가 믿음의 주체(이 경우 Leo)를 믿음의 대상과 인식론적으로 부합하게 되는 위치에 두어야 한다.7) 여기서 Kaplan생생한 지시어vivid designator라는 핵심 개념을 도입한다. 생생한 지시어란 한 사람의 내적 이야기inner story를 이루는 단칭용어이다. 이는 정신적 이미지mental images, 부분적인 기술구partial descripsion, 일상적인 이름ordinary name 등이 뒤섞인 복합체conglomeration로서, 만약 이를 만족하는 대상이 존재한다면 생생한 지시어는 그것을 지닌 사람의 마음에 그 대상을 불러일으키는 작용을 한다. 심지어 대상이 존재하지 않더라도 주체의 관점에서는 마치 그 대상이 존재하는 것처럼 여겨지도록 작용한다. 생생함vivacity이라는 개념은 보통 심리철학philosophy of mind에서 내적 현상internal phenomenon에 속하는 것으로 간주된다. 그렇기 때문에 생생한 이름vivid name이 반드시 지시체를 가져야 할 필요는 없으며,8) 어떤 동일성 진술의 양변이 생생한 지시어로 이루어져 있다 해도 그 진술의 진리치에 관해 실수할 가능성이 언제나 열려 있다.


7) 이 문장은 언뜻 이해하기 어렵다. 논의 맥락상 추정해보자면, 믿음-주체가 언어표현 e를 통해 그 지시체를 인식론적으로 투명하게 떠올릴 수 있는 경우에만, e에 대한 수출과정이 타당하다는 의미인 듯하다. (40)키가 가장 작은 간첩과 같이 막연한 개념적대언적 믿음만을 환기하는 용어를 Leo가 받아들인다 해서 그로부터 Leo가 그 간첩에 대한 대물적 믿음을 갖고 있다고 추론할 수는 없다. 이것이 타당하기 위해서는 키가 가장 작은 간첩을 통해 Leo가 바로 그 간첩을 인지적으로 명료하게 표상할 수 있어야 한다. 본문에서 곧 이어지듯이, 수출되는 용어가 믿음-주체의 입장에서 생생한 이름이어야 한다는 것이 이 문장의 요지인 듯하다.

8) 심리철학에서 내적 현상 내지 심적 상태의 두드러진 특징 중 하나로 여겨지는 것이 바로 지향성intentionality 개념이다. 상식적으로 알 수 있듯이 믿음바람욕구함혐오함사랑함 등의 정신적 상태가 존재한다고 해서 그렇게 믿어지거나 욕구되는 대상이 반드시 존재할 필요는 없다. 우리는 존재하지 않는 대상에 대해서도 정신적 태도를 취할 수 있는바 이러한 특성을 심적 상태의 지향성이라 칭한다. 물리적 상태는 대체로 이런 종류의 지향성을 갖지는 않는다. 내가 무언가를 발로 찼다면(찼다고 착각한 게 아니라 정말로 찼다면) 내 발에 치인 대상이 분명 존재한다.

Kaplan에 따르면 생생한 지시어는 이런 특징을 갖는 내적 현상에 속하는 사안이므로, 어떤 표현 e가 태도-주체 s에게 생생한 이름이라고 해서 그 지시체의 존재가 보증되는 것은 아니다. 반면 표현 e의 지시체가 보증된다고 해서 e가 임의의 주체에게 반드시 생생한 이름인 것도 아니다. 따라서 MBH’Ralph에게 생생한 이름이어도 그 MBH가 실제로는 존재하지 않을 수 있으며, 키가 가장 작은 간첩이 분명 존재한다고 해도 키가 가장 작은 간첩Leo에게는 생생한 이름이 아닐 수 있다



Kaplan 이론의 요지를 간추려보자면, 어떤 사람이 생생한 지시어를 사용하여 무언가를 생각하고 있고, 그에 더해 믿음주체의 입장에서 그 지시어와 적절하게 연관된다고 여겨지는 대상이 존재할 경우, 생생한 지시어는 그 대상을 믿음주체에게 드러낸다(표상시킨다)represent. [즉 믿음주체는 생생한 지시어를 통해 대상을 명시적으로 표상한다.] 수출작용은 이러한 경우에만 허용된다. 반면 (40)키가 가장 작은 간첩과 같이 생생하지 않은 지시어가 개입된 경우엔 단지 희미하고 막연한 개념적 내용이 표현될 뿐으로서. 이는 특정 [대언적] 믿음을 형성하는 데에는 충분하겠으나 그로부터 [대물적 믿음으로 이행하는] 수출작업을 진행할 수는 없다.

 

 

믿음 귀속과 명시적 지표사: 대자적 믿음

 

칵테일 파티에서 Jones가 방 건너편에 있는 Brown을 가리키며 다음과 같이 험담을 늘어놓고 있다 해보자:

 

(43) 대학 총장은 그가 돌팔이라고 믿는다.

The president of the university believe that he’s a charlatan.

 

Jones의 발화를 어떻게 해석해야 하는가? Frege의 도식을 철저히 따라본다면, 그의 발화를 해석하기 위해서는 Brown을 짚어내는 데에 Jones가 사용하는 뜻이 아니라 총장이 사용하는 뜻이 필요하다. 하지만 (43)은 그러한 뜻이 무엇인지에 대해 아무런 단서도 제공하지 않는다. 게다가 분명 (43)에서 (지표사가 [특정 화자에 의해 특정 맥락에서] 사용될 때 모종의 뜻이 표현된다고 한다면) Jones를 짚어내기 위해 총장이 이용하는 뜻이 아니라 Jones에 의해 이용되는 뜻을 표현한다. [요컨대 Jones는 발화맥락에 따라 화자의 뜻만을 표현하는 지표사를 사용하고 있음에도, 총장의 믿음 내용을 성공적으로 기술하고 있다. Frege적인 도식은 이런 식의 믿음 귀속이 어떻게 가능한지 설명하는 데에 어려움을 겪는다.]

JonesBrown에 대한 험담을 다음과 같이 일반화할 경우 문제는 한층 더 복잡해진다:

 

(44) 대학 사람들은 전부 그가 돌팔이라고 믿는다.

Everyone in the university believes that he is a charlatan.

 

이 발화에서도 JonesBrown을 지시하기 위해 지표사 를 사용하고 있다. Frege주의자라면 (44)에 있는 그 지표사가 표현하는 뜻이 Brown을 지시하기 위해 대학 구성원들이 각기 이용하는 다양한 뜻들이 아니라 Jones가 표현하는 뜻이라는 데에 동의해야만 한다.

앞서 6, ‘지표사의 필수불가결성에서 지표사가 단순한 지시적 장치만은 아니라는 점을 살펴본바 있다. 지표사는 매우 특이하면서도 필수적인 방식으로 화자 자신을 특정 공간 및 시점에 위치시키는 역할을 할 수 있다. 우리는 지도를 보면서 여기가 어딘지를 알고 싶어 하기도 하며, 달력을 보면서 지금 혹은 오늘이 무슨 날인지를 알고 싶어 하기도 있다. 그 경우 일반적인 개념들로 이뤄진 단순한 기술구만으로는 궁금증이 해소되지 않는다. 이는 ’, ‘나를’, ‘나 자신과 같은 1인칭 대명사가 사용되는 경우에 더욱 분명하게 드러난다. 예를 들어, 연주가 한창인 클래식 공연장에서 그 F(F한 그 사람)의 핸드폰 벨소리가 울린다면, 이를 처음 들었을 때 내가 느끼게 될 짜증스러움의 정도와, 알고 보니 그 핸드폰이 다름 아닌 의 것이었을 경우 느끼게 될 당혹스러움의 정도는 천양지차일 것이다. 1인칭 지표사가 사용된 믿음 귀속의 사례로는 다음과 같은 문장을 들 수 있다: 앞서와 마찬가지로 이 문장의 화자가 Jones라고 가정하자.

 

(45) 대학 총장은 내가 돌팔이라고 믿는다.

The president of the university believes that I am a charlatan.

 

이런 식의 자기-귀속적인 믿음self-ascribing belief대자적(對自的) 믿음belief de se이라 칭해진다. (45)에서 Jones는 총장이 Brown이 아니라 Jones 자신을 좋지 않게 생각하고 있음을 말하고 있다. 여기서도 앞서와 마찬가지로 지표적 대명사 Jones에 관해 생각하기 위해 총장이 사용하는 뜻을 표현할 수 없다. (45)에서도 Jones가 자신에 대해 생각하고 있는 그만의 방식을 표현하고 있다. 따라서 나는 돌팔이이다라는 명사절을 사용하여 Jones가 표현하고 있는 명제는, 총장이 아니라 오로지 Jones 그 자신에 의해서만 이용되거나 생각될 수 있는 것처럼 여겨진다. [그럼에도 Jones는 그 명사절을 사용하여 자신의 믿음이 아닌 총장의 믿음을 성공적으로 기술하고 있다.]

이런 난점을 해결하기 위해 (43)Kaplan의 도식에서 분석해보자면, 다음과 같이 지표사가 결합된 존재 양화문으로 나타낼 수 있다:

 

(46) 예상 가능한 맥락상 의 지시체 = Brown이다 & 다음과 같은 생생한 이름 y가 존재한다 (y는 총장에게 Brown을 표상해준다 & 총장은 ‘y는 돌팔이이다에 의해 표현되는 명제를 믿는다).9)


9) (原註) 여기서 인용부호 내부가 양화되었는데, 엄밀히 말해 이는 형식논리적으로 무의미하다. 하지만 이에 대해 올바른 논리적 관점까지 설명하는 것은 지금의 맥락상 논의를 너무 복잡하게 만들 듯하다.

 

(44)(46) 역시 이와 마찬가지의 방식으로 분석될 수 있다. 물론 이와는 다른 방식의 접근법들도 제시되어왔다. 그중 한 가지를 다음 에서 살펴보고자 한다.

 

 

암묵적인 지표적 요소

 

앞서 5장에서 살펴보았듯이, 고유명에 대한 Kipke식의 직접지시적 관점은 Frege로 하여금 뜻이라는 것을 상정하게끔 추동했던 생각 자체를 단순히 거부하기에 이른다. 왜냐하면, 만약 고유명이 직접지시적인 용어로서 내포connotation를 갖지 않고 오로지 외포denotation만을 갖는다면, 다음과 같은 상황이 만족스럽게 설명될 수 없기 때문이다:

 

(47) AliceMarilyn Monroe가 유명한 여배우라고 믿는다.

(48) AliceNorma Jean Baker가 유명한 여배우가 아니라고 믿는다.

(49) Marilyn Monroe = Norma Jean Baker.

 

직접지시론의 관점에서 보자면 단칭용어 ‘Marilyn Monroe’‘Norma Jean Baker’는 동의어이다. 따라서 Alice합리적이게도 한 명제와 그 부정을 동시에 믿는 셈이다. 하지만 직관적으로 생각하기에 (47)-(49)에 제시된 상황은 분명 있음직한 일로서 반드시 합리적인 것만은 아닌 듯하다. 반면 Frege의 관점에서 ‘Marilyn Monroe’‘Norma Jean Baker’는 각기 다른 뜻을 표현하기에 Alice의 믿음을 모순 없이 설명해낼 수 있다.

하지만 Frege의 노선이 정말로 만족스러운지 의심할 만한 강력한 이유가 있다. 당신이 Pederewski가 폴란드의 두 번째 수상이었다는 점만을 제외하고는 그에 대해 아는 바가 거의 없다고 해보자.10) 그런데 당신이 월광 소타나라는 영화에서 한 피아니스트를 얼핏 보게 되는데, 그 사람 역시 Pederewski라 불린다(실제로 Pederewski는 그 영화에 출연하여 자기 자신을 연기하였다). Pederewski에 대해 잘 모르는 당신으로서는 두 사람이 동일인이라고는 전혀 생각하지 않을 것이다. 하지만 둘은 같은 이름을 가진 동일인이다. 따라서 당신은 Pederewski가 정치인이라는 명제와, 그 부정인 Pederewski가 정치인이 아니라는 명제를 동시에 믿고 있는 듯하다. 여기서 이 문제가 고유명에 대한 직접지시론 논제와는 무관하다[즉 고유명이 직접지시적인지 여부에 대한 쟁점과는 무관하다]는 점에 유의해야 한다. ‘Pederewski’라는 용어가 어떤 식으로 설명되든, Frege의 관점에 따라 설명되든 Kripke의 관점에 따라 설명되든 그와 무관하게, 당신은 Pederwski가 정치인이이라는 것과 정치인이 아니라는 것을 동시에 받아들이고 있는바 논리적으로 모순된 믿음을 지니고 있는 듯하다.


10) (原註) 이 예시는 Kripke (1979)에서 차용하였다.


Frege는 이 문제를 다음과 같이 교묘하게 피해간다: 당신의 개인어(個人語)idiolect 내에는 하나가 아니라 두 개의 ‘Pederewski’라는 이름이 있어서, 당신은 두 이름이 각기 다른 사람의 것이라 믿고 있다. 이런 종류의 현상은 그다지 특별할 것도 없이 일상에서 매우 흔한 일이다. 당신이 동일한 하나의 사물 내지는 사물의 한 가지 종류라 여겼던 것이 실제로는 둘 혹은 그 이상이었던 일은, 자주 있을 법하지는 않지만 적어도 상상가능하다conceivable. 당신은 당신의 왼손이 늘 동일한 하나의 신체 일부라 생각해왔겠지만, 그게 실제로는 당신 모르게 매일 새벽마다 사악한 악마에 의해 새로 만들어진 일련의 다수의 실체들이었을지도 모른다. 그것들에게 내 왼손1’, ‘내 왼손2와 같은 식으로 이름을 붙여줄 수도 있다. 그 경우 분명 당신의 왼손1 당신의 왼손2이다. 전혀 있을 법하지 않은 일이지만 그렇다고 절대적으로 배제될 수도 없는 가능성이기 때문에[즉 현실적으로는 불가능하지만 논리적으로는 가능하기 때문에], 단순히 약정에 의해 의미론적으로 결정할 수 있는 종류의 사안은 아닌 듯하다. 다르게 말해 이러한 가능성이 분석적으로 거짓이라 판정할 수는 없다. 이보다 훨씬 더 그럴듯한 사례를 들 수도 있다. 당신이 창밖을 보니 어떤 버스의 앞부분이 비스듬한 각도로 보인다 해보자. 같은 벽면에 있는 다른 창문을 통해 보니 거기에서는 어떤 버스의 뒷부분이 보인다. 버스가 아주 길다는 점을 감안하건대 당신이 저 버스가 이 버스인가?’ 하고 궁금해 하는 것은 충분히 있을 수 있는 일이다.

이런 사례들에서 나타나는 단칭용어들은 고유명이나 자연종 용어라기보다는 차라리 지표사라고 해야 한다. 그중에서도 특히 지시사demonstrative라 할 수 있는바, 이 경우 단칭용어는 사용될 때마다 각기 다른 (암묵적인) -지시화demonstration가 동반되면서 여러 번 사용되는 동일한 지시사와 같이 사용되고 있는 셈이다. 그런데 우리는 앞서 6에서 과 같은 자연종 용어가 암묵적인 혹은 숨겨진 지표적 요소를 포함하고 있음을 꽤 설득력 있게 주장하는 Putnam의 논증을 살펴본 바 있다. 또 다른 예를 들자면, 19세기까지는 옥()이 한 가지 광물인 것으로 알려져 왔지만, 이후 광물학자들에 의해 두 가지 다른 광물인 경옥(硬玉)과 연옥(軟玉)으로 구분된다는 것이 밝혀졌다. 암묵적인 지표사 내지 암묵적인 지시사를 다음과 같이 명시적이게 만듦으로써 두 광물이 구분될 수 있다: (통상 옥이라고 불려온 물질의 표본을 가리키며pointing at) 이것은 경옥이고, (또 다른 물질 표본을 가리키며) 저것은 연옥이다.

 

 

직접지시, 태도, 의미론적 대물성

 

앞선 들에서 논의된 바에 따르면 명제적 태도는 Frege가 주장했을 법한 방식으로 의미론과 상호 연관되지는 않는 것처럼 여겨진다. 기실 샛별개밥바라기는 전형적인 고유명과는 거리가 멀다. 왜냐하면 이 경우 두 이름에는 공적인 의미로서 각기 독특한 제시방식mode of presentation이 결부되어 있다고 보는 편이 합당하기 때문이다. [반면 사람의 이름과 같은 일상적인 고유명은 그렇지 않다.] 그리고 아무리 그러한 경우라도 정작 중요한 사안은, 한 용어에 특정 속성이 결부되는 것은 파기 가능한데도, 그렇게 결부됨으로써 그 속성이 그 용어와 긴밀하게 결부되어 있는 듯한 착각을 불러일으킨다는 것이다. 즉 특정 속성이 한 용어의 고유한 의미 내지 뜻으로 결부되어 있는 것처럼 여겨지게 된다. 경옥과 연옥 사례, Pederewski 사례, 창밖으로 보이는 리무진 사례 등은 매우 희소한 경우로서 사실 그다지 놀라울 것도 없다. 언어는 다른 무엇보다도 의사소통을 목적으로 사용된다. 그런데 우리가 마주치는 모든 대상이나 대상의 모든 부분들에게 굳이 별도의 이름을 일일이 붙이려 한다면, 그것들이 실제로도 일일이 구분되는 사물이나 물질인 경우가 아닌 한, 정상적인 의사소통은 도저히 이뤄질 수 없을 것이다. 만약 어떤 것이 오리처럼 소리를 낸다면, 그것에 오리라는 이름을 붙여주면 그만이다.

그렇다면 이제 Frege를 제쳐두고 직접지시론의 입장에 서보자. 이 관점에서 고유명, 자연종 용어, 지표사 등의 의미론적 값semantic value 혹은 정보적 값informational value은 단지 그 용어들의 지시체에 불과하다. 이것이 바로 Mill주의적Millianism 관점으로서, 이에 따르면 고유명과 지표사는 기술적(記述的) 내용descriptive content을 표현하지 않으며 단지 대상을 지시하기만 할 뿐이다. (이러한 관점은 Russell주의적Russellianism이라 칭해지기도 한다. 고유명이 한정 기술구가 축약된 표현이라는 주장을 제외한다면 Russell의 관점 역시 대체로 이와 일치하기 때문이다.)

이에 [직접지시론자인] Nathan Salmon은 태도에 대해 다음과 같이 대안적인 설명을 제시한다. ‘슈퍼맨‘Clark Kent’가 직접지시적인 고유명이라 해보자. Salmon의 관점에 따르면 이 두 이름의 의미론적 값 내지 정보적 값은 동일하다. 그럼에도 불구하고 Lois Lane11)슈퍼맨은 영웅이야!’‘Clark Kent는 영웅이야!’를 듣는다면 각기 다르게 반응할 것이다. Salmons의 용어법을 따르자면 Lois는 동일한 유형에 속하는 두 명제-개항들12)을 각기 다른 외양(外樣)guise을 통해 파악한다grasp. 두 진술은 정확히 동일한 명제를 표현하지만, Lois는 어떤 외양 하에서는 슈퍼맨은 영웅이다를 긍정하는 성향을 보이고dispose to assent, 그와 다른 외양 하에서는 ‘Clark Kent는 영웅이다를 부정하는dissent 성향을 보일 것이다.


11) 슈퍼맨 시리즈의 등장인물로서 Clark Kent가 근무하는 언론사의 동료이자 슈퍼맨의 연인.

12) 직접지시론의 관점에서 보자면, ‘슈퍼맨‘Clark Kent’의 지시체가 동일하므로, 임의의 술어 ‘F’에 대해 슈퍼맨은 F하다‘Clark KentF하다는 동일한 명제를 표현한다.


중요한 것은 Lois가 그 명제를 파악하는 각기 다른 외양이라는 것이 이름 슈퍼맨‘Clark Kent’에 관한 의미론에 속하는 사안은 아니라는 점이다. 그 외양들은 슈퍼맨은 영웅이다Clark Kent는 영웅이다라는 진술의 의미의 일부가 아니다. 오히려 그 외양들은 Lois 내부에in Lois 있는 인지적cognitive 혹은 심리적 특질psychological feature이다. 그렇기에 이 사례에서 결정적 용어인 슈퍼맨‘Clark Kent’를 혹여 다른 어떤 사람이 충분히 능숙하게 사용하더라도, Lois가 지닌 외양이 그 사람에게 반드시 떠올라야만 하는 것은 아니다. [Lois가 한 명제를 파악하는 외양들은 Lois에게만 고유한 인지적 특성이다.] 가령 Clark Kent와 슈퍼맨이 동일인임을 잘 아는 사람이라면 두 용어가 상호대체가능하다고 생각할 것이다. (이 사례가 별로 만족스럽지 않게 여겨진다면, 한 사람이 여러 이름으로 불리는 여타 비근한 사례를 떠올려보라.)

이제 이러한 착상을 명제적 태도를 나타내는 용어가 명시적으로 포함된 문장에 적용해보자. Salmon의 이론에서 결정적인 사안은, 태도에 대한 분석 내지 믿음 문장에 대한 분석에서 태도 주체의 외양이 드러나야 한다는 것이다. 가령 Lois가 슈퍼맨이 영웅이라고 믿는다면, 그 경우 한 외양과 한 명제가 존재해서 Lois는 그 외양 하에서under that guise 그 명제에 대해 특정한 인지적 관계cognitive relation를 맺는다. 이러한 관계를 ‘BEL-관계라 칭해보자. 통상적인 2항 믿음-관계와 유사하게 BEL-관계는 3항 관계이다. ‘p’가 한 명제를 나타내고 ‘g’가 한 외양을 나타낸다고 한다면, 믿음에 대한 Salmon 식의 분석은 다음과 같이 일반화된다:

 

A는 다음의 경우 그리고 오직 그 경우에 p믿는다: 그러한 g가 존재한다 Ag에 의해서 p를 파악한다 Ag 하에서 p에 대해 BEL-관계를 맺는다.

 

이를 Lois에게 적용하기 위해, 앞서와 마찬가지로 슈퍼맨은 영웅이다‘Clark Kent는 영웅이다[직접지시론의 입장에 따라] 동일한 명제를 표현한다고 가정한 채 그 명제를 ‘P’라 하자. Salmon의 관점에 따르면 두 문장은 각기 다른 외양을 통해 Lois와 연관된다. 이에 Lois가 맺고 있는 두 믿음-관계를 분석하면 각각 다음과 같다:

 

Lois슈퍼맨은 영웅이다가 포함된 외양을 통해 P를 믿는다 Lois는 그 외양 하에서 P에 대해 BEL-관계를 맺는다.

 

Lois‘Clark Kent는 영웅이다가 포함된 외양을 통해 P를 믿는다 Lois는 그 외양 하에서 P에 대해 BEL-관계를 맺는다.

 

따라서 최종적으로 Lois는 명제 P와 그 부정을 동시에 믿고 있는 것으로 드러난다. 그녀는 슈퍼맨이 영웅이라 믿는 동시에 영웅이 아니라고도 믿는다는 것이다. 물론 후자에 대해 그녀는 그러한 단어로 표현된 믿음을 갖고 있다는 것을 부인할 것이다. 하지만 어떤 사람이 한 문장을 부정한다고 해서 그 문장에 의해 표현된 명제마저 부정한다고 할 수는 없다. 그러므로 Lois의 믿음이 이렇게 분석된다고 해서 그녀가 합리적이게 되는 것은 아니다. 그녀가 합리적이게 되는 경우란 오로지 동일한 하나의 외양 하에서 한 명제를 믿으면서 믿지 않는 경우뿐이다. 다른 사례들 역시 이와 마찬가지로 분석될 수 있다. 어떤 사람이 ‘Mark TwainSamuel Clemens이다를 부정하면서도 그와 동일한 명제를 믿는다는 게 반드시 불합리한 일은 아니다. 이 명제는 단순히 ‘a=a’의 형식을 지니고 있지만, 이 명제를 부정하는 사람이 의존하는 외양 하에서라면 ‘a=b’의 형식으로 파악될 수도 있기 때문이다.

7에서 우리는 가령 파리Paris가 독일에 있다면 파리는 모로코에 있다와 같이, 의미론적으로 올바른 문장들이 왜 화용론적으로는 부적절한지에 대해 Grice가 제시한 설명을 살펴보았다. 조건문에 대한 고전적인 진리-함수적 설명에 따르면 이 문장은 참이지만, 이 문장에 대한 발화 즉 그 문장에 대한 진술은 통상 화용론적으로 부적절하다고 받아들여진다. [이러한 착상을 Lois 사례에 차용해볼 수도 있다.] Lois Lane[순전히 의미론적인 관점에서만 보자면] 슈퍼맨이 Clark Kent라는 것, 혹은 위 사례의 경우 Clark Kent가 영웅이라는 것을 엄연히 믿고 있긴 하지만, 그녀의 믿음을 그러한 단어들로 기술하는 것은 화용론적으로 부적절할 것이다.

Salmon이 제시한 외양 개념은 언어표현의 의미론적 속성에 의해서는 충분히 해결되지 않는 인지적 측면의 잔여물을 포착하여 이와 얽힌 문제를 만족스럽게 해결해낸다. 따라서 외양에 관한 온전한 이론은 인지과정 내지 정보처리절차에 대한 경쟁하는 이론들의 일부를 이룬다고 할 수 있다. 물론 Salmon 역시 어떤 대상에게 명제적 태도를 귀속시킬 때 태도-주체가 지닌 특정 인지적 관점이 드러나도록 분석해야 한다는 데에 동의한다. 종종 우리는 어떤 사람이 자신의 태도를 명시하면서 긍정할 법한 것과 동일하거나 혹은 거의 유사한 단어들을 사용하여 그 주체의 태도를 기술하고자 한다. 그 경우 우리는 추정컨대 모종의 Grice적인 준칙에 따라서, 주체의 믿음을 기술하기 위해 사용된 단어들에 의해 순전히 의미론적으로 전달되는 내용을 명시하는 데에 몰두하기보다는, 믿음 주체의 인지적 상태에 관한 사안들을 전달하는 데에 더 주의를 기울여야 할 것이다. 다만 이는 어디까지나 Salmon이 칭한바 화용론적으로 전달되는 정보pragmatically imparted information에 속하는 사안일 뿐이다. 그러므로 고유명이 Frege적인 뜻을 표현하지 않는 직접지시적 표현이라는 논제를 유지한 채로도, Lois의 경우와 같은 변칙적인 사례들을 여전히 설명해낼 수 있다.

따라서 다음 도식은 타당하지 않다(즉 이 도식의 대입례는 경우에 따라 거짓일 수 있다):

 

BS라고 믿는다면, B가 진실될sincere 경우 B‘S’를 긍정할 것이다.

 

이러한 관점에서 보자면, 고유명이 연관되어 있는 한 대물성/대언성 간의 구분은 무너지게 된다. Frege에서 비롯된 고전적인 사고방식에 따르면, 대물적 믿음 진술에서는 믿음의 대상을 명시하는 용어(앞 사례들의 경우 개밥바라기‘Marilyn Monroe’)가 믿음 연산자 내지 믿음 술어belief predicate의 영향권 외부에 있어야 한다. 따라서 이는 곧 대물성이라는 속성being de re을 구문적인syntactical 사안 내지는 언어의 구조적인structural 사안으로 취급하는 셈이다. 반면 단칭용어가 직접지시적이라는 점을 받아들인다면 이러한 생각은 지지될 수 없다. 왜냐하면, ‘Marilyn Monroe’와 같은 단칭용어가 대상을 직접지시한다면 그 용어가 문장에 기여하는 바는 오로지 지시체밖에 없으며, 그 경우 ‘AliceMarilyn Monroe가 유명한 여배우라고 믿는다‘Marilyn Monroe에 대해, Alice는 그녀가 유명한 여배우라고 믿는다의 내용 내지 진리-조건 간에는 아무런 차이가 없을 것이기 때문이다. 따라서 이러한 관점에서는 구문론적 대물성syntactic de re 개념과 대조되는 의미론적 대물성semantic de re이라는 개념을 받아들여야 할 듯하다.

 

 

역사적 사항

 

명제적 태도에 대한 철학적 관심이 2차 세계대전 이후 극적으로 대두된 이래 이러한 기조는 지난 70년간 좀체 수그러들 기미를 보이지 않았다. 물론 세계대전이 발발하기 훨씬 이전에 FregeRussell이 명제적 태도에 관한 논의의 초석을 다지긴 하였지만, 그 주제에 본격적으로 착수한 인물은 1940년대 후반 및 1950년대에 대략 Frege적인 체계 내에서 작업하였던 Rudolf CarnapAlonzo Church였다. Carnap1947(21956에 출간)년에 출간된 의미와 필연성Meaning and Necessity에서, Church1951년의 뜻과 지칭에 관한 논리학A Formulation of the Logic of Sense and Denotation1954년의 내포적 동형성과 믿음 동일성Intensional Isomorphism and Identity of Belief에서 각각 명제적 태도에 대한 분석을 시도하였다.

이후 Quine1955년 논문 양화사와 명제적 태도Quantifiers and Propositional Attitudes(Quine 1975[역설의 길The Ways of Paradox, 개정판]에 수록)에서, Frege가 말하는 지시적 불투명성이 명제적 태도에 관한 논의의 전부는 아니며, 태도는 태도-주체와 Frege적인 명제 간에 성립하는 단순한 2항 관계가 아니라고 주장하였다. 당시 Quine이 취했던 관점에 따르면, 명제적 태도 동사의 범위 내부를 양화하는 것은 대체로 올바르다고 받아들여지기 때문이라는 것이다. 다만 최종적으로 Quine은 그러한 상식적 직관에 반대하기에 이른다. 하지만 이후 많은 인물들이 Quine의 도전에 응수하였으며, 대표적인 인물로 1968년의 획기적인 논문 내부로의 양화Quantifying In(Kaplan 1969 [단어와 반대: W. V. Quine의 저작들에 관한 小論Word and Objections: Essays on the Works of W. V. Quine]에 수록)를 쓴 David Kaplan을 들 수 있다.

명제적 태도에 관한 논의의 흐름은 1970년대에 접어들면서 Frege주의에서 벗어나 Mill주의적 관점 내지는 직접지시론의 관점으로 향하게 된다. Kaplan은 비교적 후기 논문들에서 이러한 방향전환을 보여주었으며, Kripke1979믿음에 관한 퍼즐A Puzzle About Belief을 출간하였다. 그 글에서 Kripke, 설사 양화와 연관된 쟁점들을 차치하더라도 어쨌든 Frege주의 의미론은 믿음-퍼즐을 처리하기 위해 자연스럽게 떠올릴 법한 해결책과는 거리가 멀다고 주장한다. Nathan Salmon1986년에 Frege의 퍼즐Frege’s Puzzle을 출간하였는데, 이 책은 직접지시론의 관점에서 명제적 태도 문제에 접근하고자 했던 다양한 시도들 중의 하나이다. 이 방향에서 작업한 좀 더 최근의 인물들로는 Mark RichardScott Soams를 들 수 있다.

Salmon의 접근법은 Jerry Fodor (1975)[사고언어The Language of Thought]에 제시된 사고언어Language of Thought이론과 부합하는 측면이 있다. 그 책에 제시된 Fodor의 이론에 따르면 인지활동cognition은 정신적mental이면서 개별적인individual 언어에 의해 발생하는바, 이 정신적 언어란 사실상 Frege가 제시한 바와 다소 유사한 구조more Frege-like structure를 지니고 있되 다만 공적언어public language의 층위가 아니라 개인어idiolect 층위에 있는 언어이다.13) 여기서 우리가 살펴보지 않았던 또 다른 줄기로는 Donald Davidson병렬적paratactic접근법을 들 수 있다. 이에 따르면 태도를 귀속시키는 문장은 명제와 태도-주체 간의 관계를 나타내는 게 아니라, 믿음 귀속자에 의해 제시된 문장-개항과 태도-주체 간의 관계를 나타낸다.


13) Salmon이 제시하는 외양 개념이 Fodor의 이러한 정신언어에 속하는 개념이라 할 수 있는 듯하다.

 

 

이번 의 요약

 

Frege의 도식에 따르면 -내포적 맥락에서 나타나는 표현들은 [그것이 외연적 맥락에서 가리켰을] 통상적인 지시체를 지시하는 게 아니라 [외연적 맥락에서 표현했을] 통상적인 뜻을 지시한다. 예를 들면 금성은 행성이다에서 금성은 행성 금성을 지시하지만, ‘Bob은 금성이 행성이라고 믿는다에 있는 금성은 그 지시체인 금성이 아니라 그 용어 자체의 뜻을 지시한다. 이러한 관점은 ‘Vulcan’이 존재하지 않음에도 ‘Le VerrierVulcan이 태양을 공전한다고 믿었다와 같은 문장[즉 믿음 연산자의 범위 내에 지시체가 없는 표현이 나타나는 믿음-귀속 문장]이 참이 될 수 있는 이유를 잘 설명해낸다는 이점을 갖는다. 또한 이 관점은 ‘Bob은 금성이 행성이라고 믿는다‘Bob은 개밥바라기가 행성이 아니라고 믿는다가 일관된 진리치를 가질 수 있다는 명백한 직관을 잘 설명해내기도 한다. -내포적 맥락이란 지시적으로 불투명한 맥락으로서, 이러한 맥락 내에 있는 표현들은 그와 -지시적인 여타 표현으로 대체될 수 없다. -내포적 맥락은 가령 ‘p라고 Fred가 믿는다는 것을 Bill이 믿는다는 것을 Jim은 믿는다에서처럼, -내포적 맥락, -내포적 맥락 내에 있는 -내포적 맥락, -내포적 맥락 내에 있는 -내포적 맥락 내에 있는 -내포적 맥락 등과 같은 식으로 무한히 반복될 수 있다.

이러한 -내포적 맥락과 달리, 대언적인 단순 내포적 맥락에서 진리치-보존적인 대체가 허용되기 위해서는 대체되는 표현들의 뜻이 동일해야 할 필요는 없으며, 다만 대체되는 표현들의 양상적 특징이 보존되어야 한다. 가령 대언적 내포맥락을 갖는 필연적으로, 2<3이다에서 ‘2’‘1+1’으로 대체될 수 있지만, 그와 -지시적인 화성의 위성의 개수로는 대체될 수 없다. [후자의 지시체가 ‘2’와 동일한 것은 우연적으로만 성립하기 때문이다.] 반면 온전한 지시적 투명성은, 가령 화성의 위성의 개수는, 필연적으로 3보다 작은 그러한 것이다와 같이 대물적 필연성 맥락이 갖는 특징이다. 이 대물적 필연성 진술은 그에 대응하는 앞서의 대언적 필연성 진술 필연적으로, 2<3이다를 함축하지만, 은 성립하지 않는다.

명제적 태도 문장에서의 수출작용은 수출되는 용어가 무언가를 지시하는 데에 성공하는 한 일견 타당해 보인다. a가 지시적 단칭용어라면 ‘BobFa라고 믿는다aBob이 그것에 대해 F하다고 믿는 그러한 것이다를 함축하는 듯하다. 그런데 이러한 함축관계는 다른 단칭용어 b가 있어서 b=a이고 BobFb라고 믿는 경우에도 성립한다. 왜냐하면 이 상황에서 bBob이 그에 대해 F하지 않다고 믿는 그러한 것인데, 그 경우 a 역시 Bob이 그에 대해 F하지 않다고 믿는 그러한 것이기 때문이다. 요컨대 동일한 하나의 특정 대상에 대해 Bob은 그것이 F하다고 믿는 동시에 F하지 않다고 믿고 있기도 하다. 그렇다고 해도, 가령 FaFa를 동시에 믿는 것과 같은 식의 합리성[즉 모순되는 두 대언적 명제를 믿는 합리성]Bob에게 귀속되는 것은 아니다.

하지만 수출작업이 성공적이기 위해서는 추가적인 무언가가 더 필요한 듯하다. 예를 들어, 설사 키가 가장 작은 간첩이 존재한다고 하더라도 ‘Ralph는 키가 가장 작은 간첩이 간첩이라고 믿는다와 같은 문장에서는 수출작업이 진행될 수 없기 때문이다. 일반적으로 제안되는 해결책에 따르면, 이러한 경우 수출이 진행되기 위해서는 키가 가장 작은 그 간첩이 누구인지를 Ralph가 알고 있어야 한다는 전제가 필요하다. David Kaplan은 이러한 착상을 더 명료하게 다듬어서, 수출되는 용어가 생생한 지시어여야 한다는 주장을 펼친다. 생생한 지시어는 수출 및 그 에 해당하는 수입importation작용의 대상이 된다는 점에서 고정 지시어와 유사하지만, 하나의 지시어가 생생한지 여부는 사람에 따라 각기 달라질 수 있는바 주관적으로만 생생하다는 점에서는 고정 지시어와 다르다.

지금까지 살펴본 형태의 Frege적인 그림은 지표사 및 지시사를 고려할 때 제기되는 사항들에 많은 제약을 받는 편이었다. Jones대학 총장은 내가 돌팔이라고 믿어와 같이 말하는 경우에서처럼, 어떤 명제적 태도 문장들은 명시적으로 지표사를 포함하고 있다. John Perry에 따르면 이러한 대자적 믿음은 1인칭 대명사가 나타나지 않는 그 어떤 진술과도 인식론적으로 동등하지 않지만, ‘나는 돌팔이이다Jones의 진술에서와 같은 방식으로 사용될 경우 그 문장은 총장만이 이용 가능한 명제를 표현하지 않는다. 뿐만 아니라 Frege 식의 퍼즐은 지시되는 그 어떤 대상과 관련해서든 발생할 수 있다. 예를 들어 어떤 사람이 엄청 느릿느릿하게 저 태양 = 저 태양이다하고 말한다면, 그 말을 들은 사람은 정말로 저 태양 = 저 태양인지 궁금해 할 것이다. 여기서 그 진술은 실제로 참이며, 첫 번째로 나타나는 저 태양과 두 번째로 나타나는 저 태양에는 각기 다른 -지시화 행위가 동반된다. 이러한 사안들을 감안하건대 명제적 태도는 Frege의 이론에서 즉각 환기될 법한 언어의 모양새와는 그다지 잘 부합하지 않는 듯하다. 명제적 태도라는 것은 생각보다 훨씬 섬세하고 까다로우며, 명제적 태도와 연관된 인지적 상태는 통상적으로 공적 언어에서 즉각 표현될 수 있는 것보다 훨씬 다양한 것이다.

Nathan Salmon은 명제적 태도 연산자의 영향권 내에서 -지시적 고유명들 간의 대체를 제한 없이 허용하는 관점을 펼친다. Salmon은 고유명이 직접지시적이라고 간주하기 때문에, 가령 ‘Hesperus’‘Phosphorus’가 명제적 태도 맥락 내에서도 상호교환될 수 있다고 주장한다. 다만 명제적 태도를 설명해내는 것[즉 태도-주체에 대한 믿음 귀속과 같은 문제를 설명하는 것]은 언어철학적 문제라기보다는 심리철학에서 다뤄져야 할 문제가 된다. 좀 더 자세히 말하자면, 어떤 사람이 한 명제를 믿지 않을지라도 다른 외양 하에서는 그 명제를 믿을 수 있으며, 이 경우 그러한 외양의 구성에 관한 논의들은 궁극적으로는 심리학 내지 심리철학에 속하는 사안이라는 것이다. 분명 우리는 외양과 연관된 정보에 관해 의사소통하며 이는 많은 경우 뜻에 관한 Frege의 이론과 부합하는 측면이 있긴 하지만, 그러한 정보는 Grice적인 방식에 따라 화용론적으로만 전달되거나 함의되는바 용어에 관한 의미론의 일부는 아니라는 결정적인 차이점이 있다, 예컨대 Lois‘Clark Kent는 용감하지 않다를 긍정하더라도, 그녀는 슈퍼맨이 용감하다고 믿기 때문에 그에 따라 Clark Kent가 용감하다는 것을 믿고 있기도 하다.

 

 

탐구문제

 

1. 다음을 생각해보자:


Hob은 마녀가 Bob의 암말들이 전염병에 걸리게 만들었다고 믿으며, Nob은 그녀(그와 동일한 마녀)Cob의 암퇘지들을 죽였는지 궁금해한다.

 

마녀가 존재하지 않더라도 이 문장은 참일 수 있다. 이를 어떻게 설명할 수 있는가? HobNob의 믿음을 한데 묶어주는 그러한 사물이 실존하지 않는데도, 두 사람의 믿음 사이에 성립하는 연관성을 어떻게 재구성해볼 수 있겠는가? (Geach 1967[내포적 동일성Intensional Identity]에서 차용)

 

2. Frege주의 의미론의 관점에서 볼 때, -속의-brain-in-a-vat는 당신과 정확히 동일한 믿음을 가질 수 있겠는가? 그 경우 그 고양이는 배고프다와 같은 전형적인 믿음은 거짓이겠는가, 아니면 참도 거짓도 아니겠는가? 혹은 통-속의-뇌가 그러한 믿음을 지닐 수나 있겠는가? 직접지시론의 관점에서는 어떻겠는가?

 

3. 언어표현들의 동의성 조건, 즉 용어가 표현하는 뜻의 동일성에 대한 조건으로서 한 가지 그럴 법한 것은 용어들이 (인용되는 맥락을 제외한) 모든 맥락에서 상호대체가능해야 한다는 것이다. 그런데 사다구매하다가 동의어라고 가정한 채, 다음을 생각해보자:

 

(a) x가 핫도그를 산다고 믿는 그 누구든 x가 핫도그를 산다고 믿는다는 것을 아무도 의심하지 않는다.

 

앞서 제안된 동의성 조건에 따르면 이 문장은 다음을 함축한다:

 

(b) x가 핫도그를 산다고 믿는 그 누구든 x가 핫도그를 구매한다고 믿는다는 것을 아무도 의심하지 않는다.

 

과연 이것이 올바른가? (b)(a)정확히 동일한 명제를 표현하는가? (Mates 1952[동의성Synonymity]에서 차용)

 

4. 지금까지 우리는 이 책에서 다음과 같은 사안을 그저 무비판적으로 가정해왔다: 예를 들어

 

(a) 그 고양이가 하얗다는 것

 

은 다음을 의미한다

 

(b) ‘그 고양이는 하얗다의 뜻

 

하지만 뭔가 잘못된 듯하다. 분명 번역translation에서는 언어표현의 뜻이 보존되어야 한다. 우선 (a)를 프랑스어로 번역하면 다음과 같을 것이다:

 

(c) que le chat est blanc

 

그리고 (b)를 프랑스어로 번역하면 다음과 같을 것이다:

 

(d) le sens du 그 고양이는 하얗다

 

이 과정에서 뭔가 잘못된 것처럼 여겨지는 이유는, (b)가 특정 한국어 문장과만 연관되기 때문이다. (b)는 한국어 문장의 제시방식[Frege적인 뜻]을 포함하고 있다. 앞서 말했듯 뜻은 지시결정규칙이므로, 이 번역과정에서는 한국어 문장 그 고양이는 하얗다에 대한 지시가 보존되어야 한다. 따라서 (d)(b)와는 동의적이지만 (c)와 동의적이지는 않다. 그런데 (c)(a)와 동의적이므로, 결국 [우리의 직관과는 다르게] (b)(a)와 동의적이지 않게 되어 버린다. 하지만 (b)(a)와 엄밀히 동등하지 않다면, 대체 어떤 표현이 (a)와 동의어일 수 있겠는가? 아니면 위의 추론과정 어딘가에 결함이 있는 것인가? (Curch 1950[주장진술 및 믿음진술에 대한 Carnap의 분석에 관하여On Carnap’s Analysis of Statement of Assertion and Belief])

 

 

주요 읽을거리

 

Kripke, S. (1979), 믿음에 관한 퍼즐A Puzzle about Belief: A. Margalit , 사용에서의 의미Meaning in Use에 수록.

Salmon, N. (1986, 1991), Frege의 퍼즐Frege’s Puzzle.

다음은 좀 더 어렵다:

Kaplan, D. (1969), 내부로의 양화Quantifying In.

Quine, W. V. (1975), 양화사와 명제적 태도Quantifiers and Propositional Attitudes, 개정판.

 


댓글(0) 먼댓글(0) 좋아요(0)
좋아요
북마크하기찜하기
 
 
 
처음 처음 | 이전 이전 | 1 | 2 | 3 |다음 다음 | 마지막 마지막